OPTHA

You might also like

Download as pdf or txt
Download as pdf or txt
You are on page 1of 297

Only Study Notes / Only for Study purposes / Personal notes/ AMC MCQ

Recall sessions
EYE

1.Patient with red eye, decrease in vision, normal red reflex, what is
the most likely diagnosis?
a. conjunctivitis
b. blepharitis
c. glaucoma…
d. keratitis
e. Uveitis

• glaucoma…red and light reflex


lost / Chronic close angle does
not cause red eye.
• keratitis…normal vision with
grittiness
• conjunctivitis vision is no
decrease
• blepharitis is also vision
normal

Uveitis is defined as inflammation of the uveal tract, which is further subdivided into
anterior and posterior components. The anterior tract is composed of the iris and
ciliary body, while the posterior tract includes choroid. Hence, uveitis is inflammation
of any of these components and may also include other surrounding tissues such as
sclera, retina, and optic nerve.[1] Uveitis is often idiopathic but may be triggered by
genetic, traumatic, immune, or infectious mechanisms.

Only for Study purpose / Only Notes / AMC MCQ


Only Study Notes / Only for Study purposes / Personal notes/ AMC MCQ

Only for Study purpose / Only Notes / AMC MCQ


Only Study Notes / Only for Study purposes / Personal notes/ AMC MCQ

2- Neonatal conjunctivitis tx after iv abs


- Ans: Sulphacetamide drops
no option

There is a difference Neonatal


conjunctivitis is after one week its
chlamydia or if it's in between two days
Gonorrhoea.

3- Orbital cellulitis scenario, what invg to find


causative organism?
1. eye swab
2. blood culture
3. CT brain
Ans: So blood culture to determine organism and CT for diagnosis

Orbital cellulitis is an uncommon complication of bacterial rhinosinusitis, but rhinosinusitis is the


source of most cases of orbital cellulitis; coexisting rhinosinusitis is present in 86 to 98 percent of
cases of orbital cellulitis [2-5]. Ethmoid sinusitis and pansinusitis are the forms of rhinosinusitis most
likely to lead to orbital cellulitis.
As has been noted above, the ethmoid sinuses are separated from the orbit by the lamina
papyracea, a thin structure with many fenestrations. Computed tomography (CT) scanning often
shows the predominant site of inflammation to be the medial aspect of the orbit, adjacent to the
ethmoid sinuses, and subperiosteal abscesses most often occur in the same location.
The causative organisms of orbital cellulitis are often difficult to identify. Cultures from the orbit are
only obtained if surgical intervention is needed, usually to drain an abscess. Sinus cultures obtained
during surgery can be used to guide antimicrobial therapy [18], but may not accurately reflect the

Only for Study purpose / Only Notes / AMC MCQ


Only Study Notes / Only for Study purposes / Personal notes/ AMC MCQ

pathogen(s) in the orbit. Blood cultures are sometimes positive in children, the percentages ranging
from 0 to 33 percent in various retrospective series [19-23]. They are rarely positive in adults.

Bacteria — The most commonly identified pathogens in orbital cellulitis are Staphylococcus aureus
and streptococcus

Microbiologic studies — Despite the low yield, we


recommend obtaining blood cultures in all patients with
suspected orbital cellulitis before the administration of
antibiotics. In patients with concomitant acute bacterial
sinusitis who are examined by an otolaryngologist, any
purulent sinus drainage seen on endoscopic
examination should be cultured. If surgery is
performed, the material obtained should be examined
by Gram stain and, in patients with risk factors for a
fungal and/or mycobacterial ethology, by special stains
for fungi and mycobacteria. Cultures should be done for
ordinary bacterial pathogens and, depending on the
circumstances, fungi and mycobacteria.

Only for Study purpose / Only Notes / AMC MCQ


Only Study Notes / Only for Study purposes / Personal notes/ AMC MCQ

Only for Study purpose / Only Notes / AMC MCQ


Only Study Notes / Only for Study purposes / Personal notes/ AMC MCQ

4- Another clinical scenario mentioning painful red eye but not much increase
in lacrimation and eye lids are not sticky (most likely trying to rule out
purulent cause). Pic given showing marked conjunctival vessel injection.
Asked for next appropriate step. Almost similar pictures.
a. Topical Streroid
b. Topical Acyclovir
c. Topical Chloramphenicol
d. Other antibiotics I don’t remember

Episcleritis is defined by the abrupt onset of inflammation in the episclera of one or both eyes,
typically presenting as redness, irritation, and watering of the eye with preserved vision. Most
patients with episcleritis have a mild, isolated problem that responds readily to topical therapy alone
and does not pose a threat to vision.

Episcleritis is not sight-threatening and, in most patients, is an episodic, self-limited process; thus,
symptomatic relief should be the goal of therapy. Indeed, many patients with episcleritis require no
treatment per se since the condition typically resolves over a short course. For the achievement of

Only for Study purpose / Only Notes / AMC MCQ


Only Study Notes / Only for Study purposes / Personal notes/ AMC MCQ

symptomatic relief, there are four levels of therapy that we successively employ in the following
sequence, depending upon the adequacy of the response to the initial and subsequent intervention:
●Topical lubricants
●Topical nonsteroidal anti-inflammatory drugs (NSAIDs)
●Topical glucocorticoids
●Oral NSAIDs

Only for Study purpose / Only Notes / AMC MCQ


Only Study Notes / Only for Study purposes / Personal notes/ AMC MCQ

Only for Study purpose / Only Notes / AMC MCQ


Only Study Notes / Only for Study purposes / Personal notes/ AMC MCQ

5- Picture of red eye with complain of severe pain, no


discharge.
Treatment asked -- (Looks like Episcleritis)
a) Topical hydrocortisone
b) Topical methylprednisolone
c) Chloramphenicol

Treatment of episcleritis usually involves the following:


1. Topical corticosteroids eye drops given several times per
day.
2. Topical lubricant eye drops such as artificial tears.
3. Cold compresses 3 to 4 times per day.
4. Non-steroidal anti-inflammatory medications given by mouth
are prescribed in more severe cases.

Scleritis is a painful, destructive, and potentially blinding


disorder that may also involve the cornea, adjacent episcleral,
and underlying uveal tract. Up to 50 percent of patients with
scleritis have an underlying systemic illness, most often a
rheumatic disease.
Treatment of scleritis always requires systemic therapy with nonsteroidal antiinflammatory drugs
(NSAIDs), glucocorticoids, or other immunosuppressive drugs.

Extra info:
Pinguecula is a yellowish, raised growth on the conjunctiva. It's usually on the side of the eye near your nose,
but can happen on the other side too. A pinguecula is a deposit of protein, fat, or calcium. Pterygium is a
growth of fleshy tissue (has blood vessels) that may start as a pinguecula.

Only for Study purpose / Only Notes / AMC MCQ


Only Study Notes / Only for Study purposes / Personal notes/ AMC MCQ

6- A patient presents with a very painful red eye with watery


discharge.
Normal disc on fundoscopy Eye muscle movements were all
normal No
picture given Investigation asked
A)Ct (CT is done if extraocular movements are affected)
B)Tonometry
C)Gonioscopy
D)slit lamp examination

Red eye pain


discharge maybe
any foreign body or
due to irritation
in glaucoma there
will be visual field
and optic disc
change

What Is Gonioscopy
Used For?
Our eyes constantly make aqueous humor. As new
aqueous flows into your eye, about the same amount
should leave the eye through the drainage angle. This
process keeps pressure in your eye (called intraocular

Only for Study purpose / Only Notes / AMC MCQ


Only Study Notes / Only for Study purposes / Personal notes/ AMC MCQ

pressure or IOP) stable. If the drainage angle is not working properly, fluid builds up. Pressure in the
eye rises, damaging the optic nerve. This is often how glaucoma develops.

Tonometry is the procedure eye care professionals perform to determine the intraocular pressure
(IOP), the fluid pressure inside the eye. It is an important test in the evaluation of patients at risk from
glaucoma. Most tonometers are calibrated to measure pressure in millimeters of mercury (mmHg).

Glaucoma is diagnosed in patients with characteristic nerve damage on fundus examination (picture
1A-B) and visual field testing, typically in the presence of elevated intraocular pressure (IOP).

Glaucoma is a group of eye diseases traditionally characterized by elevated intraocular pressure


(IOP). However, glaucoma is more accurately defined as an optic neuropathy than a disease of high
pressure. In open-angle glaucoma, optic nerve damage results in a progressive loss of retinal
ganglion cell axons, which is manifested initially as visual field loss and, ultimately, irreversible
blindness if left untreate
Open-angle glaucoma is an optic neuropathy characterized by progressive peripheral visual field loss
followed by central field loss in a typical pattern. It is usually but not always in the presence of
elevated intraocular pressure (IOP). Increased aqueous production and/or decreased outflow are
possible mechanisms for elevated
intraocular pressure (figure 2). The optic
nerve or "disc" takes on a hollowed-out
appearance on ophthalmoscopic
examination, which is described as
"cupping." Cupping is associated with the
loss of ganglion cell axons.
Individuals with open-angle glaucoma rarely
experience symptoms. Thus, open-angle
glaucoma is generally detected incidentally
during comprehensive ophthalmic
examination. This is in contrast to angle-
closure glaucoma in which patients present
with symptoms and signs including loss of
visual acuity, pain, conjunctival erythema,
and corneal edema.
High elevations of intraocular pressure
(IOP), up to 40 mmHg in patients with open-
angle glaucoma, generally cause no pain,
redness, or visual symptoms. There is no
loss of visual acuity as long as central vision
is preserved.

Only for Study purpose / Only Notes / AMC MCQ


Only Study Notes / Only for Study purposes / Personal notes/ AMC MCQ

7- Patient was digging ground and suddenly developed left


eye pain. Can’t open eye
due to photophobia. ask what you will do to diagnose.
(Short scenario)
a. Apply topical atropine
b. Ct scan of head and eye
c. Apply topical fluorescein
d. Visual acuity
Cause of photophobia is corneal abrasion

● Digging fround
● Some dust particle..
● Apply topical fluoresin
● Slit lamp examination

Fluorescein
Fluorescein is used to detect any epithelial defect
associated with the foreign body. It can be
instilled via a moistened strip or as drops, and the ocular
surface viewed with a cobalt blue filter
from an ophthalmoscope or a Wood’s lamp. Multiple
vertically orientated linear abrasions should raise suspicion
of a tarsal plate foreign body. The Seidel’s test using 2%
fluorescein can
detect an aqueous leak
that will occur in non–
self sealing corneal
perforations.

Only for Study purpose / Only Notes / AMC MCQ


Only Study Notes / Only for Study purposes / Personal notes/ AMC MCQ

8- Pic of red eye.. H/o cataract surgery with painful loss of vision ..
1. acute glaucoma.
2. conjunctivitis.
3. Scleritis
he file did not mention but says the cause is hypopyon
So hypopeon mean >uveitis > means > glucoma
Hypopyon is a medical condition involving inflammatory cells in the anterior chamber of the eye.

It is an exudate rich in white blood cells, seen in the anterior chamber, usually accompanied by
redness of the conjunctiva and the underlying episclera. It is a sign of inflammation of the anterior
uvea and iris, i.e. iritis, which is a form of anterior uveitis.

Post operative complication of cataract surgery >>>


1) cloudy vision
The main problem that can occur after
cataract surgery is a condition called
posterior capsule opacification (PCO).
2) inflammation (swelling and redness)
in the eye
3) swelling of the retina (cystoid macular
oedema) – where fluid builds up
between layers of the retina at the back
of
the eye, sometimes affecting vision
4) swelling of the cornea – where fluid
builds up in the cornea at the front of
the eye; this usually clears itself
5) Dislocated Intraocular Lenses
6) retinal detachment – a rare
complication where the retina (layer of nerve cells inside the back of the eye) becomes
separated from the inner wall of the eye
7) infection in the eye, such as endophthalmitis (a rare bacterial infection)

For Glaucoma:
CLINICAL PRESENTATION
The rapidity and degree of the intraocular pressure (IOP) elevation from angle-closure determines
whether symptoms occur. If the IOP rises quickly, as is typical of acute primary angle-closure
glaucoma, patients may experience some or all of the following symptoms:

●Decreased vision

●Halos around lights

●Headache

●Severe eye pain

●Nausea and vomiting

Only for Study purpose / Only Notes / AMC MCQ


Only Study Notes / Only for Study purposes / Personal notes/ AMC MCQ

Secondary angle-closure — Secondary angle-closure results when the anterior chamber angle
becomes occluded as the result of conditions that push the iris or ciliary body forward ("pushing"
conditions) or deform the iris so that it is retracted into the angle ("pulling" conditions).
Pushing conditions include:

●Fibrosis of the pupil to the anterior surface of the lens.

●Choroidal swelling or hemorrhage (due to types of ophthalmic surgery, retinal laser

treatment, posterior scleritis, or drug reactions). Topiramate is the most common of the sulfa-

based medications to cause this rare, idiosyncratic reaction.

●Plateau iris syndrome (a developmental anomaly).

●A large or anteriorly displaced lens.

●Aqueous misdirection (in which aqueous humor fills the vitreous cavity instead of flowing

through the pupil, most commonly as a result of ophthalmic surgery).

●Choroidal hemorrhage or effusion.

●A space-occupying lesion in the posterior segment of the eyeball (such as a tumor or a gas

bubble placed during retinal surgery).

●Developmental syndromes causing fibrosis in the posterior segment of the eyeball

Pulling conditions include:

●Inflammation or blood in the angle itself that fibroses and contracts

●Neovascularization of the iris with a resulting fibrovascular membrane (most commonly seen

in poorly controlled diabetes mellitus or ophthalmic artery insufficiency)

●Abnormal corneal endothelial cell proliferation

●Prolonged shallowing of the anterior chamber resulting in iris-angle contact due to trauma or

surgery

Only for Study purpose / Only Notes / AMC MCQ


Only Study Notes / Only for Study purposes / Personal notes/ AMC MCQ

●Epithelial cell or fibroblast invasion of the angle

Saw this in an article. Angle


closure glaucoma is an
important consideration in a
patient with painful acute visual
loss. It is often distinguished
from entities such as optic
neuritis by the severity of pain
(which can be excruciating)
and a red eye with an
enlarged, nonreactive pupil.

Only for Study purpose / Only Notes / AMC MCQ


Only Study Notes / Only for Study purposes / Personal notes/ AMC MCQ

9- Old age Pt. with cataract had improved vision after


surgery, in post op. day 4 he wake at morning with
painful eye & blurred vision, what happened to this pt. ?
A) Conjunctivitis
b) Uveitis ans best ans is Endophthalmitis
C) suture infection & abscess
d) hypopyon (due to endophthalmitis) (if asking about
picture sing)
E) Acute glaucoma

Hypopyon
Layering out of White Blood Cells in anterior chamber
III. Causes
• Endoophthalmitis
• Anterior Uveitis
• Corneal Ulcer
• Behcet's Disease

Only for Study purpose / Only Notes / AMC MCQ


Only Study Notes / Only for Study purposes / Personal notes/ AMC MCQ

Endophthalmitis
An intra-ocular bacterial infection (both anterior and posterior chambers) that may complicate any
penetrating injury, including intro-ocular surgery. Can be endogenous spread (bacteraemia,
fungaemia).
CF:
-red painful eye, may be with hypopyon (pus in the anterior chamber)

Anterior Uveitis: Hyphema:

Endophthalmitis refers to bacterial or fungal


infection within the eye, including involvement
of the vitreous and/or aqueous humors. It is not
caused by viruses or parasites; by convention,
infections due to these organisms are included
in the term "uveitis" (eg, cytomegalovirus
[CMV] retinitis, toxoplasma chorioretinitis).
Most cases of endophthalmitis are exogenous,
resulting from inoculation of organisms from
the outside, via trauma, eye surgery, or as an
extension of keratitis (corneal infection). In
such cases, the aqueous humor may be
seeded first before extension into the vitreous.
The remaining cases are endogenous,
resulting from bacteremic or fungemic seeding
of the eye. In endogenous endophthalmitis,
organisms usually seed the highly vascular
choroid first then extend anteriorly into the
vitreous.

Most cases of endophthalmitis are due to


bacteria and present acutely. Acute bacterial
endophthalmitis is a vision-threatening
condition and must be managed as an
emergency. The clinical outcome depends
upon both the virulence of the infecting
organism and the speed with which
appropriate therapy is initiated.

Only for Study purpose / Only Notes / AMC MCQ


Only Study Notes / Only for Study purposes / Personal notes/ AMC MCQ

Bacterial endophthalmitis can be divided into several categories:

●Acute postoperative (primarily postcataract surgery) endophthalmitis

●Chronic pseudophakic endophthalmitis

●Post-intravitreal injection endophthalmitis (eg, after anti-vascular endothelial growth factor or

corticosteroid injections)

●Bleb-related endophthalmitis

●Post-traumatic endophthalmitis

●Endogenous endophthalmitis

Symptoms and signs — The onset of symptoms occurs within one week of surgery in 75 percent
of cases. Patients usually give a 12 to 24 hour history of decreasing vision and eye "ache" (they
may deny eye "pain"). Patients feel otherwise well and are afebrile.
On physical examination, findings are confined to the affected eye. The lids often appear normal,
although they may be slightly swollen. The conjunctiva may be injected or edematous
(conjunctival chemosis), although these findings can also represent residual postoperative
changes. Visual acuity is decreased, and a hypopyon (layering of white blood cells in the
anterior chamber) is often present (picture 1). The view of the retina is usually hazy, and, in 80
percent of patients, no retinal vessels can be seen [18]. Slit lamp examination reveals intraocular
white blood cells and protein (called "cells" and "flare," respectively, by ophthalmologists).
Diagnosis — Endophthalmitis is a clinical diagnosis that is confirmed by positive aqueous or
vitreous culture. However, a negative culture does not exclude the diagnosis.
Acute postcataract, post-intravitreal injection, and post-traumatic endophthalmitis are the most
common types of endophthalmitis
Cataract surgery is commonly performed, with over 3 million surgeries performed each year in the
United States alone. As a consequence, acute postcataract endophthalmitis is a common type of
endophthalmitis. Worldwide incidence of acute postcataract endophthalmitis ranges from 0.04 to 0.1
percent of cataract surgeries [3-5]. Most cases are caused by bacteria.

10- A 77-year-old woman presents with pain and decreased vision in her right eye. She states that
it started yesterday and has been rapidly worsening. Her temperature is 99.0°F (37.2°C), blood
pressure is 133/81 mmHg, pulse is 88/min,
respirations are 15/min, and oxygen saturation is
98% on room air. Physical exam is notable for the
finding in Figure A. Which of the following is most
likely to be found in this patient's history?

1. Atopic dermatitis
2. Blunt trauma
3. Cataract surgery
4. Chronic steroid use
5. Recent viral illness

Only for Study purpose / Only Notes / AMC MCQ


Only Study Notes / Only for Study purposes / Personal notes/ AMC MCQ

This patient is presenting with pain and decreased vision with a hypopyon on physical exam which
is concerning for endophthalmitis. Recent cataract surgery is a risk factor for this condition.

Symptoms of endophthalmitis include eye pain, vision loss, swollen eyelids, and conjunctivitis.
Physical exam reveals the presence of a hypopyon, which is a layer of leukocytes in the anterior
chamber. Initial management involves aspiration for culture and intravitreal antibiotics (vancomycin
and ceftazidime or amikacin). Prognosis depends on the cause and severity of endophthalmitis; a
good indicator of outcome is visual acuity. Risk factors for endophthalmitis include anything that
penetrates the eye that could inoculate bacteria including recent surgery or penetrating trauma.

Figure/Illustration A shows an example of endophthalmitis with a hypopyon present (blue arrow).

Incorrect Answers:
Answer 1: Atopic dermatitis and eczema could be found in the history of a patient with allergic
conjunctivitis which presents with itchy and watery eyes with edema. Symptoms are bilateral and
worsened with allergen exposure.

Answer 2: Blunt trauma may cause a hyphema to form in the eye which would present with a
collection of blood in the inferior portion of the eye in the anterior chamber.

Answer 4: Chronic steroid use is a risk factor for cataracts which presents with bilateral and
progressive loss of vision (worse at night) with a cloudy lens seen on slit-lamp exam.

Answer 5: Recent viral illness presents with unilateral conjunctival injection and watery/itchy eyes
with symptoms that typically spread from one eye to the other in addition to other symptoms of a
viral infection such as a runny nose and cough.

Bullet Summary:
Penetrating trauma and cataract surgery are risk factors for endophthalmitis.

11- A 67-year-old man presents to the emergency room with eye pain. He notes that his right eye
has become progressively more painful over the past day and that his vision has been worsening.
He had cataract surgery in the same eye 2 weeks ago. He has a past medical history of diabetes,
hypertension, obesity, and erectile dysfunction. He is currently sexually active and engages in
anal, oral, and vaginal intercourse. His temperature is 100°F (37.8°C), blood pressure is 164/104
mmHg, pulse is 85/min, respirations are 12/min, and oxygen saturation is 98% on room air. Visual
acuity is 20/400 in the affected eye. Physical exam is performed as seen in Figure A. Which of the
following is the most appropriate treatment for this patient?

1. Intravitreal ceftazidime and amikacin


2. Intravitreal vancomycin and
ceftazidime
3. IV ceftriaxone and topical erythromycin
4. IV vancomycin and amikacin
5. IV vancomycin and piperacillin-
tazobactam

This patient is presenting with blurry vision


and a hypopyon after cataract surgery which
is concerning for endophthalmitis which
should be treated with intravitreal vancomycin
and ceftazidime.

Only for Study purpose / Only Notes / AMC MCQ


Only Study Notes / Only for Study purposes / Personal notes/ AMC MCQ

Infectious endophthalmitis is a severe complication of cataract surgery. Patients may present up to


6 weeks following surgery with a red, painful eye, and on slit-lamp examination, a collection of pus
may be seen filling the anterior chamber. The most common pathogens are gram-positive agents
such as streptococcal species and staphylococcal species. Physical exam may demonstrate a
hypopyon and decreased visual acuity. Management involves a vitreous tap for culture and
injection of intravitreal antibiotics with appropriate combinations being vancomycin and either
ceftazidime or amikacin. Vitrectomy (removal of the vitreous humor) is only indicated in refractory
or severe cases.

Figure A shows a hypopyon, an accumulation of pus in the anterior chamber which is seen in
endopthalmitis.
Incorrect Answers:
Answer 1: Intravitreal ceftazidime and amikacin are incorrect as the appropriate antibiotics are
vancomycin and either ceftazidime or amikacin.
Answer 3: IV ceftriaxone and topical erythromycin may be appropriate management of
gonnococcal conjunctivitis which presents with a purulent discharge from the eye. It would not
present with blurry vision and a hypopyon.
Answer 4: IV vancomycin and amikacin are inappropriate as this combination of antibiotics must
be given intravitreally.
Answer 5: IV vancomycin and piperacillin-tazobactam are an inappropriate combination of
antibiotics and is an inappropriate route of administration. This is a broad-spectrum regimen that
could be appropriate in septic shock.
Bullet Summary:
The treatment of endophthalmitis is intravitreal vancomycin and ceftazidime (or amikacin).

12- Pic of red eye... post op case of cataract... pt woke up on day 3 with

pain and lid swelling plus little hypopyon plus red conjunctiva. pupil
seemed to me small... also visual acuity decreased. most likely Dx?

a. Hypopyon
b. acute iritis
c. Glaucoma (acute glaucoma has dilated pupil)

Only for Study purpose / Only Notes / AMC MCQ


Only Study Notes / Only for Study purposes / Personal notes/ AMC MCQ

Iritis — Inflammation of the anterior uveal tract is called iritis or anterior uveitis; when the adjacent
ciliary body is also inflamed, the process is called iridocyclitis. (See "Uveitis: Etiology, clinical
manifestations, and diagnosis".)
Patients with iritis may present in a similar fashion to those with an active corneal process,
but there is no foreign body sensation per se. The patient may choose to keep the eyes closed
to block out light but, in a dimly lit environment, the patient is able to keep the affected eye open
spontaneously. Patients with an active corneal process and iritis will display an aversive response
when the penlight is shined in the affected and in the uninvolved eye.
The cardinal sign of iritis is ciliary flush: injection that gives the appearance of a red ring around
the iris. Typically, there is no discharge and only minimal tearing. The pupil is typically very small.
Corneal abrasion should be ruled out with fluorescein staining, and angle closure should be ruled
out by confirming that the pupil is not fixed in mid-dilation. The diagnosis is presumptive until
presence of inflammatory cells or exudative “flare” is confirmed by slit lamp examination.
Iritis can be caused by any one of many infections, inflammatory, and infiltrative processes. These
include tuberculosis, sarcoidosis, syphilis, toxoplasma, toxocara, and reactive arthritis. Many
cases are idiopathic.
Patients with iritis should be seen by an ophthalmologist within a matter of days. The
ophthalmologist will initiate treatment, typically with topical steroids, and monitor for side effects
and response to therapy. Cases that are bilateral, recurrent, sight-threatening, or non-responsive
to therapy will require extensive evaluation for etiology.

Anterior uveitis – Anterior uveitis may produce pain and


redness, although these symptoms are minimal if
inflammation begins insidiously (eg, in juvenile idiopathic
arthritis [JIA]). In anterior uveitis (iritis), the redness, if
present, is primarily noted at the limbus (the junction
between the cornea and the sclera); such patients often
have a constricted pupil and pain. The degree of visual
loss associated with anterior uveitis is variable.
The presence of leukocytes in the anterior chamber of the
eye on slit lamp examination is characteristic of anterior
uveitis, but is nonspecific (see 'Differential diagnosis'
below). Leukocytes are not normally found in the aqueous
humor that fills the space between the cornea and the
lens. A haze, described by ophthalmologists as "flare,"
may also be appreciated by slit-lamp examination and
reflects protein accumulation in the aqueous humor
secondary to disruption of the blood aqueous barrier.

Posterior and intermediate uveitis – In contrast to anterior


uveitis, posterior or intermediate uveitis is more likely to be
painless but may result in nonspecific visual changes such as
floaters and/or reduced visual acuity. Redness of the eye is
not a prominent feature of posterior inflammation unless there
is an accompanying anterior uveitis.

Only for Study purpose / Only Notes / AMC MCQ


Only Study Notes / Only for Study purposes / Personal notes/ AMC MCQ

Only for Study purpose / Only Notes / AMC MCQ


Only Study Notes / Only for Study purposes / Personal notes/ AMC MCQ

13- A 7-year-old girl presents to the emergency department with swelling around her eye. This has
happened to her before but has never been this severe. The lesion started a few days ago but has
become increasingly red and painful. Physical exam reveals the lesion in Figure A which is tender to
palpation. Her cranial nerves are intact and there is no pain with extraocular movements. Which of the
following is the most likely diagnosis
1Chalazion
2Dacryocystitis
3Hordeolum
4Orbital cellulitis
5Periorbital cellulitis

• This patient is presenting with inflammation and tenderness around


the nasal aspect of her lower lid and medial canthus suggesting a
diagnosis of dacryocystitis.

• Dacrocystitits is an infection of the lacrimal gland caused most


commonly by Staphylococcus aureus as well as Streptococcus pneumoniae, Haemophilus
influenzae, and Streptococcus pyogenes. Treatment is indicated to prevent the spread to the
cavernous sinus resulting in thrombosis or meningitis. Treatment involves clindamycin,
decongestants, warm compresses, and IV antibiotics if the patient is ill or symptoms are
worsening.

• Figure A demonstrates inflammation and erythema around the patient's lower lid and medial
canthus suggesting a diagnosis of dacryocystitis.

Incorrect Answers:
• Answer 1: Chalazion presents with eyelid swelling and erythema that progresses to a
painelss/rubbery nodular lesion.

• Answer 3: Hordeolum or a stye presents with a tender, swollen eyelid and occurs secondary to
blockage and infection of a sebaceous/sweat gland.
• Answer 4: Orbital cellulitis presents with a swollen eye, a fever, a leukocytosis, and pain with
extraocular movements.

• Answer 5: Periorbital cellulitis presents with swelling and erythema surrounding the eye without
pain with extraocular movements.

• Bullet Summary:
• Dacryocystitis presents with tender inflammation over the medial canthus

14- Pic of a red eye, old man came 4th pod of cataract surgery, most likely dx?
a.acute iritis
b.hyphema
c.conjunctivitis
d.glaucoma

Hyphema — Hyphema refers to the finding of red blood cells layered out in the anterior chamber.
This finding warrants same-day evaluation by an ophthalmologist as it can be associated with
significant trauma, inflammation, or pathologic neovascularization.

I think that we got from the questions that endophthalmitis was common after cataracts and it can give
us hypopyon or ant uveitis(iritis)

Only for Study purpose / Only Notes / AMC MCQ


Only Study Notes / Only for Study purposes / Personal notes/ AMC MCQ

Post operative complication of cataract surgery >>>


1) cloudy vision
The main problem that can occur after cataract surgery is a condition called posterior capsule
opacification (PCO).
2) inflammation (swelling and redness) in the eye
3) swelling of the retina (cystoid macular oedema) – where fluid builds up between layers of the retina
at the back of
the eye, sometimes affecting vision
4) swelling of the cornea – where fluid builds up in the cornea at the front of the eye; this usually clears
itself
5) Dislocated Intraocular Lenses
6) retinal detachment – a rare complication where the retina (layer of nerve cells inside the back of the
eye) becomes
separated from the inner wall of the eye
7) infection in the eye, such as endophthalmitis (a rare bacterial infection)
Swelling of the cornea or retina.
9) Increased pressure in the eye (ocular hypertension).
10) Droopy eyelid (ptosis).

15-. red eye, decreased visual acuity, normal pupil response


1. Iritis
2. keratitis
3. angle-closure glaucoma

• Iritis (constricted and irregular pupil)


• angle-closure glaucoma (non-reactive dilated pupil)

16- A patient with painful red eye and irregular pupil, best long term
management?
A. Acetazolamide
B. Steroids (no long term)
C. Iridotomy (primary/ angle closure glaucoma treatment)

Dx:Angle closure glaucoma

Acetazolamide It may be used long term for the treatment of open angle glaucoma and short term for
acute angle closure glaucoma until surgery can be carried out.

Laser peripheral iridotomy (also described as ‘laser iridotomy’ or simply termed 'iridotomy') is a medical
procedure which uses a laser device to create a hole in the iris, thereby allowing aqueous humor to traverse
directly from the posterior to the anterior chamber and, consequently, relieve a pupillary block.[1][2][3] It is
commonly used to treat a wide range of clinical conditions, encompassing primary angle‐closure glaucoma,
primary angle closure (narrow angles and no signs of glaucomatous optic neuropathy

Open Angle:

• All patients with glaucoma should receive treatment upon diagnosis. Pharmacologic therapy, laser therapy
(trabeculoplasty), and/or surgery (trabeculectomy) have been shown to lower intraocular pressure (IOP) in
randomized trials (discussed below)
• Glucocorticoids — Glucocorticoid preparations (ie, ocular, oral, inhaled, and periocular dermatologic preparations) can
raise intraocular pressure (IOP) in open-angle glaucoma patients. For example, 90 percent of primary open-angle
glaucoma patients have been reported to develop an elevation in pressure after one month of topical ocular use [48].
Intranasal steroids (despite carrying a warning about use in glaucoma) typically do not cause an increase in IOP [49].
• In general, a two-week (or longer) course of glucocorticoids is required before an increase in IOP is seen.

Only for Study purpose / Only Notes / AMC MCQ


Only Study Notes / Only for Study purposes / Personal notes/ AMC MCQ

Only for Study purpose / Only Notes / AMC MCQ


Only Study Notes / Only for Study purposes / Personal notes/ AMC MCQ

Iritis/Ant Uveitis Treatment


Medical
• antiviral therapy
o indication
▪ first-line for uveitis caused by a viral disease
• topical corticosteroids
o indication
▪ anterior uveitis
▪ first-line for noninfectious anterior uveitis
• systemic corticosteroids
o indication
▪ first-line for posterior uveitis
▪ bilateral uveitis unresponsive to topical corticosteroids
Glaucoma
Acetazolamide
Closed-angle (acute congestive) glaucoma
• 500 mg PO/IV, followed by 125-250 mg PO q4hr
• Sustained-release: 500 mg PO q12hr
Open-angle (chronic simple) glaucoma
• 250 mg-1 g PO/IV qDay or divided q6-12hr
• Sustained-release: 500 mg PO q12hr
It may be used long term for the treatment of open angle glaucoma and short term for acute angle closure
glaucoma until surgery can be carried out.

Only for Study purpose / Only Notes / AMC MCQ


Only Study Notes / Only for Study purposes / Personal notes/ AMC MCQ

Open Angle

Laser therapy — Laser therapy (trabeculoplasty) increases aqueous


outflow by improving drainage of aqueous humor through the trabecular
meshwork. The procedure is performed as an outpatient in the
ophthalmologist’s office or at a surgicenter or hospital outpatient clinic.
Options include argon laser trabeculoplasty or selective trabeculoplasty,
which uses a low-level laser that only affects selected pigmented tissue
in the eye. There is a trend toward preference for selective laser
trabeculoplasty over argon laser trabeculoplasty due to fewer adverse
events such as pressure rise attributed to scarring, ease of use, and the
fact that the procedure can be repeated.

Only for Study purpose / Only Notes / AMC MCQ


Only Study Notes / Only for Study purposes / Personal notes/ AMC MCQ

Only for Study purpose / Only Notes / AMC MCQ


Only Study Notes / Only for Study purposes / Personal notes/ AMC MCQ

Only for Study purpose / Only Notes / AMC MCQ


Only Study Notes / Only for Study purposes / Personal notes/ AMC MCQ

17- Pic given with right red eye. Pt has seasonal hay fever and also
complaining of lower back pain. Diagnosis? (depend on picture)
a. Conjunctivitis
b. Uveitis
c. Iritis
d. Keratitis

One eye - as-iritis

Ankylosing Spondylitis May Cause Anterior Uveitis


About 30% of people with ankylosing spondylitis develop
anterior uveitis sometime in the course of their disease.
Anterior uveitis is an inflammation of the front part of the
eye called the uvea, including the iris and ciliary body.

Allergic conjunctivitis is caused by airborne


allergens contacting the eye, which leads to
immunoglobulin E (IgE)-mediated local mast cell
degranulation and allergic inflammation. It
typically presents as bilateral ocular pruritus,
redness, and watery discharge.
Thus, the acute form of allergic conjunctivitis is
characterized by intense episodes of itching,
hyperemia, tearing, chemosis, and eyelid edema

Eye pain is not characteristic of allergic


conjunctivitis. Ocular pain should raise concern for
more serious disorders, such as angle-closure
glaucoma, scleritis, or episcleritis.

Only for Study purpose / Only Notes / AMC MCQ


Only Study Notes / Only for Study purposes / Personal notes/ AMC MCQ

18- A 78-year-old woman presents to the ophthalmologist with complaints of painless, blurry vision
that has worsened in the past year. She says that she sees halos around lights, and that she
particularly has trouble driving at night because of the glare from headlights. On a physical exam,
the patient has an absence of a red reflex. What is the most likely pathology that is causing this
patient’s visual symptoms?
1Corneal edema
2Degeneration of the retina
3Hardening of the lens
4Neovascularization of the retina
5Optic nerve head damage

THE RED REFLEX. The single most important piece of


clinical information that a paediatric ophthalmologist
requires is the presence or absence of a red reflex. An
absent red reflex usually suggests sight-threatening
pathology (cataract) and may mean life-threatening
pathology (retinoblastoma).

Cataracts develop slowly with a gradual decline in vision that is not corrected with glasses. Symptoms
include blurry vision, difficulty with night vision, and seeing glare and halos around lights. Types of
cataracts include age-related (the most common type), as well as traumatic and metabolic cataracts (such
as in patients with uncontrolled diabetes, galactosemia, Wilson disease, and myotonic dystrophy). Due to
the hardening and yellowing of the crystalline lens (Illustration A), there is an absence of a red reflex
(reflection of light from the retina) in patients with cataracts. The treatment for cataracts is surgical removal
and replacement of the natural lens with an artificial intraocular lens.

Only for Study purpose / Only Notes / AMC MCQ


Only Study Notes / Only for Study purposes / Personal notes/ AMC MCQ

Illustration A shows a an eye with a cataract, as seen on slit


lamp examination.

Incorrect Answers:
Answer 1: Corneal edema also causes symptoms of blurred
vision and halos around lights; however, corneal edema most
commonly develops after damage to the corneal endothelium
from eye surgery, corneal infections, or Fuchs corneal
endothelial dystrophy. The lack of a trigger for corneal damage
in this patient makes cataracts the more likely diagnosis.

Answer 2: Degeneration of the retina is the pathology behind


age-related macular degeneration (AMD), which presents with
progressive decreased visual acuity, blurred vision, and metamorphopsia (visual distortion). The absence
of a red reflex in this patient indicates that there is opacification of the lens, making cataracts the more
likely diagnosis.

Answer 4: Neovascularization of the retina is a feature of proliferative diabetic retinopathy, an eye disease
that can manifest in patients with long-standing diabetes. Symptoms of diabetic retinopathy include
decreased vision, floaters and flashes of light, and defects in the field of vision.

Answer 5: Optic nerve-head damage is the pathology behind glaucoma, which is typically asymptomatic
until late disease when patients become aware of a constricted visual field or blurred vision. The absence
of a red reflex in this patient indicates that there is opacification of the lens, making cataracts the more
likely diagnosis.

Bullet Summary:
Cataracts present with painless, progressive loss of visual acuity, lens opacification, and an absent red
reflex.

19. Picture of cherry red spot on fundoscopy given. But scenario said vision
loss in my right eye improved after some time. Cause?
a. CRAO (if asking for diagnosis)
b. Carotid Disease (if asking for cause)
c. Retinal detachment

Central retinal artery occlusion (CRAO) is a


disease of the eye where the flow of blood through
the central retinal artery is blocked (occluded).
There are several different causes of this
occlusion; the most common is carotid artery
atherosclerosis.

A cherry-red spot is a finding in the macula of the


eye in a variety of lipid storage disorders and in
central retinal artery occlusion.[1] It describes the
appearance of a small circular choroid shape as
seen through the fovea centralis. [2] Its appearance
is due to a relative transparency of the macula;
storage disorders cause the accumulation of
storage material within the cell layers of the retina,
however, the macula, which is relatively devoid of
cellular layers, does not build up this material, and
thus allows the eye to see through the macula to
the red choroid below.

Only for Study purpose / Only Notes / AMC MCQ


Only Study Notes / Only for Study purposes / Personal notes/ AMC MCQ

CRAO can be classified based on it pathogenesis,


as arteritic versus non-arteritic.[5][1][6] Non-arteritic
CRAO is most commonly caused by an embolus
and occlusion at the narrowest part of the carotid
retinal artery due to plaques in the carotid artery
resulting in carotid retinal artery
atherosclerosis.[1][5][6] Further causes of non-arteritic
CRAO may include vasculitis and chronic systemic
autoimmune diseases.[5] Arteritic CRAO is most
commonly caused by giant cell arteritis.[5][6] Other
causes can include dissecting aneurysms and
arterial spasms, and as a complication of patient
positioning causing external compression of the
eye compressing flow to the central retinal artery
(e.g. in spine surgeries in the prone position)

Amaurosis fugax: With respect


to embolic and hemodynamic
causes, this transient monocular
visual loss ultimately occurs due
to a temporary reduction in
retinal artery, ophthalmic artery,
or ciliary artery blood flow,
leading to a decrease in retinal
circulation which, in turn,
causes retinal hypoxia.[10]
While, most commonly, emboli
causing amaurosis fugax are
described as coming from an
atherosclerotic carotid artery,
any emboli arising from
vasculature preceding the
retinal artery, ophthalmic artery,
or ciliary arteries may cause this
transient monocular blindness.

CRVO

Only for Study purpose / Only Notes / AMC MCQ


Only Study Notes / Only for Study purposes / Personal notes/ AMC MCQ

CRAO
Risk factors :
• Carotid emboli
• TIA
• Atherosclerosis
• Symptoms
• Sudden loss of vision
• It may be transient initially "Amaurosis
Fugax"
• Shows cherry red spot if not treated
immediately may led to permanent
damage
• Treatment
• Urgent referral
• Inhale Co2
• And after 2 hours IV acetazolamide
treatment myst ne in less than by hour
• Complication
• Permanent damage

20- Picture of very red eye with subconjunctival


haemorrhage (no scenario given)asking for next step:
A- CT (if eye trauma history)
B- Warm dressing…could be done acc to medscape
C- Antibiotic
D- Local anesthetic

In JM, it can happens spontaneously and if follows


trauma, it can be orbitak fracture. But in this pt no
trauma history
Subconjunctival hemorrhages are common in the elderly
(over 80 years of age), in whom they are most often
associated with systemic disease, primarily
hypertension. In patients under 40 years of age, ocular
rather than systemic conditions, primarily minor trauma
or complications from contact lens use, are seen [1].
Other causes at any age include coagulopathy.

Only for Study purpose / Only Notes / AMC MCQ


Only Study Notes / Only for Study purposes / Personal notes/ AMC MCQ

(especially in patients receiving anticoagulation therapy), diabetes mellitus, and elevated venous
pressure (Valsalva maneuver, coughing, vomiting).
Subconjunctival hemorrhage — Patients with subconjunctival hemorrhage may have a history of
trauma or contact lens use or may report no history of trauma and note the hemorrhage after
waking from sleep. A typical subconjunctival hemorrhage appears as a focal, flat, red region on
the ocular surface representing a collection of blood between the sclera and the conjunctiva.

In elderly patients, the clinician should perform


a complete history to determine if any trauma
occurred and measure the blood pressure.
Coagulation studies should be performed to
assess for a bleeding diathesis or
overmedication (in patients who are
anticoagulated) and if subconjunctival
hemorrhages are recurrent or other signs of a
bleeding disorder are present.
Subconjunctival hemorrhage resulting from
trauma (or cases when trauma cannot be
ruled out in patients who are poor
historians), particularly in the setting of bullous
elevation of the conjunctiva, warrants urgent
ophthalmology consultation to evaluate for
additional retinal trauma and definitively rule out
open globe injury.

Subconjunctival hemorrhage is the most


common ocular complication of leptospirosis,
occurring in as many as 92% of patients. Renal
manifestations include hematuria. Oliguric or
anuric acute tubular necrosis may occur during the second week due to hypovolemia and
decreased renal perfusion.1

Only for Study purpose / Only Notes / AMC MCQ


Only Study Notes / Only for Study purposes / Personal notes/ AMC MCQ

Eye Trauma:
Although it is not always possible to identify the source of the problem, some potential causes of
subconjunctival hemorrhage include: Eye trauma. A sudden increase in blood pressure that can
result from heavy lifting, coughing, sneezing, laughing and constipation.

The initial approach to eye injury must first address life-threatening injuries prior to assessing
periocular and ocular damage. Once life-threatening injuries are excluded or identified and treated,
the clinician should identify threats to vision using a focused history and an organized approach
(algorithm 1). Rapid recognition and treatment of caustic eye exposures, orbital compartment
syndrome (OCS), open-globe injuries, or orbital entrapment is essential to preservation of vision.

Evaluate periocular structures for injuries requiring imaging and/or specialty


consultation – Patients with normal vision or no change from baseline should undergo
careful evaluation of the lids and extraocular movement. Important findings indicating the
need for specialty consultation include:
•Protruding foreign body involving the orbit and/or globe – These patients warrant
orbital CT and urgent ophthalmology consultation.
•Limited extraocular movement – Patients with limited extraocular movement,
especially pain on lateral gaze and/or diplopia on upward gaze suggesting an orbital
fracture with muscle entrapment (picture 8 and image 2), should undergo orbital CT,
urgent ophthalmology consultation, and involvement of other specialists depending upon
the site of the fracture. (See "Orbital fractures".)
•Lacerations warranting orbital CT, specialty consultation, or both – The following
lacerations warrant specialty consultation.
Lacerations with orbital fat prolapse (picture 6) or full thickness injuries of the eyelid
– These findings suggest penetration beyond the orbital septum Patients with these
injuries warrant orbital CT and repair should be performed by an ophthalmologist or
plastic surgeon.
-Lacerations through the lid margin (picture 5), wounds which are poorly aligned, or
lid avulsions - these wounds require careful approximation by an ophthalmologist or
plastic surgeon.
-Lacerations involving the tear drainage system .

Only for Study purpose / Only Notes / AMC MCQ


Only Study Notes / Only for Study purposes / Personal notes/ AMC MCQ

21- A 42-year-old woman presents to her primary care provider with vision loss. She reports that twice
over the last 2 weeks she has had sudden “black out” of the vision in her right eye. She notes that both
episodes were painless and self-resolved over approximately a minute. The patient’s past medical
history is significant for hypertension, diet-controlled diabetes mellitus, and hypothyroidism. Her family
history is notable for coronary artery disease in the patient’s father and multiple sclerosis in her
mother. Ophthalmologic and neurologic exam is unremarkable. Which of the following is the best next
step in management?
1Check serum inflammatory markers
2Emergent referral to ophthalmology
3Intravenous dexamethasone
4MRI of the brain
5Ultrasound of the carotid arteries

This patient presents with transient monocular vision loss and vascular risk factors, which
suggests a diagnosis of amaurosis fugax of an ischemic etiology. The best next step in
management is ultrasound of the carotid arteries.

Amaurosis fugax is a general term for transient loss of vision. The most common cause of
transient monocular visual loss is ischemia to the retina or optic nerve, which may be due to retinal
emboli from the carotid artery, giant cell arteritis, or retinal vein occlusion. All patients with
transient monocular visual loss who are older than age 50 or have vascular risk factors should first
be evaluated for carotid artery disease with bilateral ultrasounds of the carotid arteries.

Incorrect Answers:

Answer 1: Checking serum inflammatory markers such as erythrocyte sedimentation rate (ESR) is
an appropriate initial screening test for patients greater than 50 years of age to exclude the
diagnosis of giant cell arteritis (GCA). This patient is under age 50 and has vascular risk factors
that make an ischemic etiology more likely. GCA-initial rest-E -Confirmatory-biops-Next step-
high dose steroid

Answer 2: Emergent referral to ophthalmology would be the appropriate initial step for intraocular
etiologies of vision loss, including retinal detachment. Patients with retinal detachment classically
describe their vision loss as a “curtain falling” over their eyes.

Answer 3: Intravenous dexamethasone is the best initial step in management of suspected GCA
(giant cell arteritis). GCA more commonly presents in patients over the age of 50 and is often
accompanied by symptoms of jaw claudication and headache

Answer 4: MRI of the brain plays a role in the evaluation of optic neuropathy or neuritis as MRI can
demonstrate other evidence of demyelinating disease. While this patient has a family history of
multiple sclerosis, optic neuritis usually presents with pain on eye movement.

Bullet Summary:

Only for Study purpose / Only Notes / AMC MCQ


Only Study Notes / Only for Study purposes / Personal notes/ AMC MCQ

Patients with transient monocular visual loss and vascular risk factors should be evaluated for
carotid artery disease with ultrasound of the carotid arteries.

Only for Study purpose / Only Notes / AMC MCQ


Only Study Notes / Only for Study purposes / Personal notes/ AMC MCQ

Only for Study purpose / Only Notes / AMC MCQ


Only Study Notes / Only for Study purposes / Personal notes/ AMC MCQ

22-5. Picture given of red eye, acute & painful, his brother had ankylosing spondylitis. What is the most
appropriate action to reach Dx?
A. Xray spine
B. Bone scan
C. HLA haplotype

STEP ONE: A patient with chronic low back pain (at least three months), with onset before age 45,
should have an anterior-posterior (AP) plain radiograph of the pelvis to examine the sacroiliac
joints. A diagnosis of AS can be made if, in this patient with back pain, the image meets criteria
for sacroiliitis (at least grade 2 bilaterally or grade 3 unilaterally)
The diagnosis of axial spondyloarthritis (axSpA), which includes ankylosing spondylitis (AS) and
nonradiographic axSpA (nr-axSpA), should be considered in patients with an onset before age 45
of continuous chronic back pain, although few patients with chronic back pain and only a minority
of the fraction with the features typical of inflammatory back pain (IBP) are likely to have axSpA
[10]. Many other conditions may cause back pain in clinical practice; most often such pain is of
shorter duration than is typical of axSpA.
Review:
Axial spondyloarthritis (axSpA), including both ankylosing spondylitis (AS) and nonradiographic
axSpA (nr-axSpA), can generally be diagnosed based upon the recognition of a pattern of clinical,
laboratory, and imaging findings characteristic of axSpA.

STEP ONE: A patient with chronic low back pain (at least three months), with onset before
age 45, should have an anterior-posterior (AP) plain radiograph of the pelvis to examine
the sacroiliac joints. A diagnosis of AS can be made if, in this patient with back pain, the
image meets criteria for sacroiliitis (at least grade 2 bilaterally or grade 3 unilaterally) (see
'Plain radiography' above) and the patient has at least one other SpA-typical parameter.
Further steps are not required for diagnosis, but the information obtained in the following step
may help to better characterize the patient's illness and may be important in management.
●STEP TWO: In patients who are not positive for sacroiliitis by plain radiography of the

pelvis, the presence or history of each of 11 features of SpA should be ascertained; a

patient with at least 4 of the 11 SpA features can usually be diagnosed with nr-

Only for Study purpose / Only Notes / AMC MCQ


Only Study Notes / Only for Study purposes / Personal notes/ AMC MCQ

axSpA, but preferably such patients should have positive imaging and/or positive test

for HLA-B27; the absence of both of these two findings makes SpA less likely in such

patients. (See 'History' above and 'Physical examination' above and 'Laboratory testing'

above.)

These 11 features characteristic of SpA are:


•IBP
•Heel pain (enthesitis)
•Dactylitis
•Uveitis
•Positive family history for SpA
•Inflammatory bowel disease
•Alternating buttock pain
•Psoriasis
•Asymmetric arthritis
•Positive response to nonsteroidal antiinflammatory drugs (NSAIDs)
•Elevated acute phase reactants (ESR or CRP)
●STEP THREE: Patients with fewer than four SpA features and without radiographic sacroiliitis

should undergo HLA-B27 testing, if it has not already been performed.

•Patients without radiographic sacroiliitis but with two to three SpA features and positive
HLA-B27 testing may generally be diagnosed with nr-axSpA.
•A diagnosis of axSpA is relative unlikely, and a diagnosis other than SpA should
generally be sought in those with two or three SpA features who are negative for HLA-
B27 and lack radiographic sacroiliitis. However, in patients in whom the clinical suspicion
of axSpA remains high, the SI joint should be evaluated for evidence of sacroiliitis by
MRI, which can support the diagnosis of nr-axSpA. In addition, an MRI of the SI joints
may be useful to document active inflammation that may benefit from treatment or to
provide further prognostic information.
●STEP FOUR: Patients without radiographic sacroiliitis and with zero or only one SpA feature, but a
positive test for HLA-B27, should be evaluated for sacroiliitis by MRI. An MRI that is positive for sacroiliitis
supports the diagnosis of nr-axSpA in such patients but is itself not diagnostic and should be interpreted in
the context of the patient's symptoms and other findings.

Only for Study purpose / Only Notes / AMC MCQ


Only Study Notes / Only for Study purposes / Personal notes/ AMC MCQ

Only for Study purpose / Only Notes / AMC MCQ


Only Study Notes / Only for Study purposes / Personal notes/ AMC MCQ

Only for Study purpose / Only Notes / AMC MCQ


Only Study Notes / Only for Study purposes / Personal notes/ AMC MCQ

MRI of sacroiliac joints — In patients with obvious sacroiliitis documented by plain radiography,
additional imaging, such as an MRI, is not necessary for diagnosis; however, in patients in
whom plain radiographs do not reveal sacroiliitis or findings are uncertain, and when there is a
high index of clinical suspicion of SpA, abnormal findings on MRI are an important part of making
the diagnosis of nr-axSpA (see 'Diagnosis' below). This is because MRI, unlike plain radiographs,
can reveal inflammatory changes, fatty changes, and subtle structural abnormalities.
However, the usefulness of MRI is limited by the difficulty of interpreting these images, especially
without special training, and also that the MRI needs to be interpreted in the context of the degree
of clinical suspicion. In addition, active SI joint inflammation is often no longer evident in patients
with more advanced AS, not all patients with nr-axSpA have abnormal MRI findings, and false-
positive findings may occur in individuals without axSp.

Plain radiography — A standard AP plain radiograph of the pelvis should be obtained to evaluate
the SI joints. In a patient with chronic back pain, the presence of obviously significant
abnormalities of the SI joint(s) on plain radiographs, including erosions, ankylosis, changes in joint
width, or sclerosis, strongly supports the diagnosis of AS.

23-Contro a pic of eye with conjunctival injection, very red eye. . .headache with photophobia
A:uveitis ( - photophobia + - none watery - ciliary flush very small pupil )
B:episcleritis - (no photophobia )
C:conjunctivitis -( no photophobia )
D:glaucoma - (Photophobia + - none/watery discharge + fixed, mid dilated pupil)

Only for Study purpose / Only Notes / AMC MCQ


Only Study Notes / Only for Study purposes / Personal notes/ AMC MCQ

24- Patient came with painful red eye with blurred


vision for last 2 hours, cornea was hazy, and
peri-corneal injection. Which of the following will
give long term solution of the problem?
1. Laser therapy
2. Trabeculectomy
3. Iridotomoy

Closure Angle Closure glaucoma review


Acute primary angle-closure glaucoma
Medical therapy — There are no available trials
comparing medical options for treatment of acute
angle-closure glaucoma, and treatment
recommendations are based on clinical
experience [22]. When an ophthalmologist is
available for consultation within one hour of
patient presentation, patients with signs or
symptoms suggesting possible acute angle-closure should be referred for emergency assessment
and treatment.
When there is likely to be an hour or more delay before a patient can be seen by an
ophthalmologist, and the suspicion of an acute attack is high, empiric treatment should be initiated.
If the vision is normal, but other symptoms and signs suggest an acute angle-closure attack,
empiric treatment should only be given if the intraocular pressure is significantly elevated (eg, >40
mmHg). For an acute primary angle-closure attack, initial management involves prompt
administration of pressure-lowering eye drops. A possible regimen would be one drop each, one
minute apart, of [22]:

●0.5% timolol maleate;

●1% apraclonidine; and

●2% pilocarpine

Only for Study purpose / Only Notes / AMC MCQ


Only Study Notes / Only for Study purposes / Personal notes/ AMC MCQ

We also suggest giving the patient 500 mg of oral or intravenous (IV) acetazolamide. The eye
pressure should be checked 30 to 60 minutes after giving pressure-lowering drops and
acetazolamide. If the eye pressure is still significantly elevated, the same drops could be given
again, but the patient should also be examined immediately by an ophthalmologist. Systemic
medications other than acetazolamide (such as IV mannitol) should be administered under the
guidance of an ophthalmologist, since angle-closure should be confirmed before they are given.
If medical treatment is successful in reducing IOP, as is most often the case, corneal edema and
eye pain will typically lessen or resolve. In refractory cases, the ophthalmologist may consider
performing an anterior chamber paracentesis to remove some aqueous humor and immediately
lower the eye pressure temporarily, which may help to break the attack. Once the attack is broken,
the treatment of choice is a peripheral iridotomy. If laser peripheral iridotomy fails to remain patent,
or the cornea is too cloudy to visualize the iris, surgical peripheral iridectomy may be necessary.
Laser peripheral iridotomy — This procedure creates a tiny hole in the peripheral iris through
which aqueous humor can flow and reach the angle [23]. Once the iridotomy is patent, pupillary
block is bypassed. A peripheral iridotomy is usually created with a laser.
The IOP is rechecked 30 to 120 minutes after the iridotomy is performed, and mild steroid drops
are given for several days. Repeat gonioscopy is then performed to determine if the angle is wider.
The pupil is dilated to ensure that the IOP does not rise significantly and to better assess any
glaucoma damage to the optic nerve.
Possible complications of laser peripheral iridotomy include:

●Increased IOP

●Inflammation

●Laser burns to the cornea, lens, or retina

●A ghost image in the vision

●Increased rate of cataract formation [24,25]

●Need for repeat treatment if the hole were to shrink or close spontaneously

The fellow eye should be examined. If a narrow angle is found, prophylactic laser peripheral
iridotomy should be performed to prevent future attacks of angle-closure glaucoma [24].
Untreated, approximately 50 percent of fellow eyes in acute angle-closure glaucoma patients will
have another attack within five years [24,26].
Surgical peripheral iridectomy — In this procedure, the ophthalmologist makes an incision into
the anterior chamber and manually removes a small amount of iris tissue to create a passage for
aqueous humor to reach the angle.
Other surgery — In a few cases of angle-closure glaucoma, goniosynechialysis may be
performed in the operating room at some point after a peripheral iridotomy is in place. In
goniosynechialysis, adhesions that result in scarring of the angle are mechanically lysed in an
attempt to restore some drainage function. Results tend to be better if the scarring has been
present for less than one year [27].
Cataract surgery (phacoemulsification) with an intraocular lens implant may resolve the issue of
acute or chronic primary angle-closure glaucoma in some patients by removing the lens that may
be crowding the angle [24,28-30]. A randomized trial of 62 Chinese patients with cataracts who
had been medically treated for acute primary angle-closure glaucoma found that early
phacoemulsification was more effective than laser peripheral iridotomy in preventing recurrence of
IOP rise [31]. In a 2017 meta-analysis, patients with angle-closure glaucoma experienced a 6.4
mmHg (95% CI, -9.4 to -3.4 mmHg) lowering of IOP at 12 months’ or longer follow-up [32]. A
multicenter study randomizing angle-closure patients to either clear lens extraction (ie, lens
removal without significant cataract) or laser peripheral iridotomy found that clear lens extraction

Only for Study purpose / Only Notes / AMC MCQ


Only Study Notes / Only for Study purposes / Personal notes/ AMC MCQ

was more efficacious, with IOP 1.18 mmHg lower than after iridotomy (95% CI -1.99 to -0.38), as
well as more cost-effective [7].
Chronic angle-closure glaucoma — Laser peripheral iridotomy is the first step in treatment of
patients with chronic angle-closure glaucoma, to relieve any pupillary block component. The IOP
may remain elevated, however, if scarring has already damaged the drainage angle. In this case,
the remaining glaucoma is treated medically and surgically much as in open-angle glaucoma. (See
"Open-angle glaucoma: Epidemiology, clinical presentation, and diagnosis".)
Secondary angle-closure glaucoma — Secondary angle-closure glaucoma is managed by
treating the cause if possible. Controlling the IOP medically and surgically afterwards is the next
step, much as in open-angle glaucoma. (See "Open-angle glaucoma: Epidemiology, clinical
presentation, and diagnosis".)

25 - . your in a rural area and 4 cases of trachoma come to you then you find out extra 20 how can you
treat acute indexes ?
a-hand washing
b-azithromycin
c-doxycyclin
d-don’t do anything
e-penicillin

Trachoma is the leading infectious cause of blindness worldwide [1]. It is a chronic


keratoconjunctivitis caused by recurrent infection with Chlamydia trachomatis (a small, gram-
negative obligate intracellular bacterium); humans are the only hosts. Trachoma is caused almost
exclusively by C. trachomatis serotypes A, B, Ba, and C; genital infection is caused by serotypes
D through K
Treatment of trachoma consists of community-based antibiotic therapy; in addition, patients with
trichiasis warrant surgery tailored to individual physical findings.
Antibiotic therapy — Antibiotic therapy for trachoma is best managed within an entire region
rather than on an individual basis.
Antibiotic selection — We suggest azithromycin (single dose 20 mg/kg orally, maximum 1 g) for
community-wide distribution

A single dose of an antibiotic (azithromycin) is the recommended treatment for all people with active
trachoma.
This antibiotic is also recommended for the contacts of the person being treated. A contact is anyone
who is living and/or sleeping in the same house as a person with trachoma. If the person lives or
sleeps in multiple households, then all members of each household are regarded as contacts.
If there is a high rate of trachoma in a community, then all Aboriginal members of the community
should be treated with this antibiotic.
Aboriginal adults over 40 years of age in communities where trachoma is usually present (endemic)
should be screened annually for trichiasis. Adults who complain of a sore eye also need to be
examined for trichiasis. Health services need to ensure timely treatment and referral to an eye surgeon
for people with trichiasis.

Transmission — C. trachomatis is highly infectious and transmitted rapidly in areas of poor hygiene. C.
trachomatis is transmitted between individuals via spread of ocular and nasal secretions on fingers and use
of contaminated fomites. Within a family, spread of infection can recur up to six months after azithromycin
treatment; among families, spread of infection can occur in 12 months after antibiotic treatment

Only for Study purpose / Only Notes / AMC MCQ


Only Study Notes / Only for Study purposes / Personal notes/ AMC MCQ

26- Rural indigenous community. 5 kids have trachoma. Contact tracing resulted in 20 people.
Community population is 250. What to do apart from treating index patient?
a. Nothing more
b. Treat contact
c. Treat community
d. Prophylactic drops to contact
e. Prophylactic drops to community

250 -- 100% / 5 --- x = 5x100/250 = 2% of population with Trachoma


<5% Treat cases and contacts.

Only for Study purpose / Only Notes / AMC MCQ


Only Study Notes / Only for Study purposes / Personal notes/ AMC MCQ

Important review of Trachoma:


Acute chlamydial conjunctivitis

Chlamydia trachomatis conjunctivitis (adult inclusion conjunctivitis) usually presents as an acute or subacute
unilateral conjunctivitis with mucopurulent discharge, or as a chronic conjunctivitis. It should be considered as a
differential diagnosis of bacterial or viral conjunctivitis.
Chlamydial conjunctivitis usually occurs in neonates, and in adults with exposure to sexually transmitted
infections. Infection in a neonate, infant or child may reflect mother-to-child transmission, accidental transmission
or sexual abuse (see also STIs in infants and children).
Conjunctival swabs for nucleic acid amplification testing (NAAT) (eg polymerase chain reaction [PCR]) are
recommended in all neonates, and in patients with persistent conjunctivitis when C. trachomatis is suspected.
Systemic treatment is necessary. There is no evidence that concomitant topical therapy improves outcomes. For
adults and children older than 1 month, use:

azithromycin 1 g (child: 20 mg/kg up to 1 g) orally, as a single


dose.

For neonates, use:

azithromycin 20 mg/kg orally, daily for 3 days.

Approximately 50% of neonates with chlamydial conjunctivitis have associated chlamydial pneumonia (see
Pneumonia caused by Mycoplasma pneumoniae or Chlamydophila (Chlamydia) species).
Treat the mother of the infected neonate for C. trachomatis infection—for treatment regimens, management of
sexual contacts and other information, see Approach to Chlamydia trachomatis infection.
Refer patients with chlamydial conjunctivitis who have pain, photophobia or reduced vision to an ophthalmologist
because corneal complications can occur.

Trachoma

Trachoma is a form of chronic C. trachomatis conjunctivitis caused by repeated infections with C. trachomatis

serotypes A, B, Ba or C. It is the leading cause of preventable infectious blindness in the world, especially in

developing countries, and is still common in remote Aboriginal and Torres Strait Islander communities in

Australia. Without treatment, recurrent infection can lead to scarring of the eyelids, corneal ulceration, corneal

scarring and loss of vision. Suspect trachoma in all cases of conjunctivitis in endemic areas. Treatment is with

azithromycin (see Acute chlamydial conjunctivitis above for dosage).

In areas where trachoma is prevalent, regular face washing and treatment of household contacts is
recommended. Community-wide treatment may be required in areas where prevalence is high. For further
information on public health management of trachoma, see the Communicable Diseases Network Australia
National guidelines for the public health management of trachoma
https://www1.health.gov.au/internet/main/publishing.nsf/Content/cdna-song-trachoma.htm

Only for Study purpose / Only Notes / AMC MCQ


Only Study Notes / Only for Study purposes / Personal notes/ AMC MCQ

Glossary
Active trachoma

Chronic inflammation of the conjunctiva caused by infection with Chlamydia trachomatis. The World Health
Organization (WHO) simplified trachoma grading scheme defines active trachoma as TF and/or TI, where TF
(trachomatous inflammation follicular) is the presence of 5 or more follicles in the central part of the upper tarsal
conjunctiva, each at least 0.5mm in diameter, and TI (trachomatous inflammation intense) is pronounced inflammatory
thickening of the upper tarsal conjunctiva that obscures more than half of the normal deep vessels.1

At-risk communities

Remote and rural communities are classified as being at risk of trachoma based on 1) no recent data, but historical
evidence of endemicity; 2) Data of active trachoma prevalence ≥ 5% in Aboriginal and Torres Strait Islander children
aged 5-9 years in the last five years; or 3) Data < 5% active trachoma prevalence but with a recorded prevalence of
active trachoma ≥ 5% in the past 5 years.

Blinding trachoma

The indicators for blinding trachoma being eliminated as a public health problem are when TF is sustained at < 5% in
children aged 1-9 years and the prevalence of trichiasis is < 0.1% in a community where management for trichiasis is
in place.2

Community

For the purpose of this National Guideline, a "community" is defined as a specific location where people reside and
where there is at least one school.

Community-wide

Antibiotic administration to all Aboriginal and Torres Strait Islander people in the treatment community who weigh
>3kg and who live in houses with children < 15 years.

Endemic trachoma

Prevalence of active trachoma of ≥5% in Aboriginal and Torres Strait Islander children aged 5-9 years or a prevalence
of trichiasis of ≥ 0.1% in the Aboriginal and Torres Strait Islander adult population.

Facial cleanliness

Absence of nasal and ocular discharge on the face.

Hyperendemic trachoma

Prevalence of active trachoma of ≥ 20% in Aboriginal and Torres Strait Islander children aged 5-9 years.

Contact

Anyone who is living and/or sleeping in the same house as a person with trachoma. If the active case lives or sleeps
in multiple households, then all members of each household are regarded as contacts.

SAFE strategy

Surgery, Antibiotics, Facial cleanliness, Environmental health

Screening coverage

Defined as the proportion of 5-9 year old Aboriginal and Torres Strait Islander children in a community who were
screened for trachoma at the time of community screening. The denominator used to calculate screening coverage is
based on the best information available from school enrolments, community health service databases, or other
sources. Children who reside in places that do not have a school should be included in the screening coverage
calculation for the community of the school which they attend.

Treatment Coverage

Only for Study purpose / Only Notes / AMC MCQ


Only Study Notes / Only for Study purposes / Personal notes/ AMC MCQ

Defined as the proportion of Aboriginal and Torres Strait Islander people in a community who weigh >3kg and live in a
house with 1 or more children aged <15 years and who were treated for trachoma during each episode of community-
wide treatment. The denominator used to calculate treatment coverage should be based on the number of Aboriginal
and Torres Strait Islander people in such houses in the community at the time that treatment is being administered.
For most communities this will include people in the community over the several days it takes to complete the
distribution of treatment; in the case of large communities treatment may take longer. People who reside in places that
do not have a school should be included in the treatment coverage calculation for the community of the school their
children attend.

27-You see many trachoma cases in indigenous community. What to do to prevent trachoma cases?

A) Give eye drops to community

Give eye drops to contact

C) Give treatment to community

D) Give treatment to contact

SAFE - Atb to contacts and community

28- 50 kids in a community found with Trachoma...no clustering of cases.. it was a small community of
total 500 ppl. How to manage this outbreak?

a- treat kids

b- treat kids and household

c- treat whole community

d- prophylactic abx for household

e- prophylactic abx for community

Only for Study purpose / Only Notes / AMC MCQ


Only Study Notes / Only for Study purposes / Personal notes/ AMC MCQ

500 -- 100% / 50 --- x = 50x100/500 = 10% of population with Trachoma


5% - 20% No clustering - Treat community.

30- In a village,4 cases of trachoma and 20 household contacts. After treating the index case what to
do?-

A) Prophylactic drops to whole village

B) to household contacts

C)treat household contacts

D)treat whole village

E)do nothing

Need more info - with only this info treat household

31. Pic of uveitis with oral and genital ulcers.T/t asked


1. Oral prednisone
2. Hydroxychloroquine
3. NSAIDs
4. Chloramphenicol

Dx: Behcet syndrome


Behcet's (beh-CHETS) disease, also called Behcet's syndrome, is a rare disorder that causes
blood vessel inflammation throughout your body. The disease can lead to numerous signs and
symptoms that can seem unrelated at first. They can include mouth sores, eye inflammation, skin
rashes and lesions, and genital sores.

Reiters: The manifestations of reactive arthritis include the following triad of symptoms: an inflammatory arthritis
of large joints, inflammation of the eyes in the form of conjunctivitis or uveitis, and urethritis in men or cervicitis in
women. Arthritis occurring alone following sexual exposure or enteric infection is also known as reactive arthritis.
Patients can also present with mucocutaneous lesions, as well as psoriasis-like skin lesions such as circinate
balanitis, and keratoderma blennorrhagicum. Enthesitis can involve the Achilles tendon resulting in heel pain.[3]
Not all affected persons have all the manifestations.
The clinical pattern of reactive arthritis commonly consists of an inflammation of fewer than five joints which
often includes the knee or sacroiliac joint. The arthritis may be "additive" (more joints become inflamed in

Only for Study purpose / Only Notes / AMC MCQ


Only Study Notes / Only for Study purposes / Personal notes/ AMC MCQ

addition to the primarily affected one) or "migratory" (new joints become inflamed after the initially inflamed site
has already improved)

32- A 30-year-old woman presents with a bump on her eyelid. She reports that it started as a small lesion that
has been progressively increasing in size over several weeks. It is not painful, but she wants to know if it needs
to be treated. The patient is generally healthy and smokes marijuana frequently. Physical exam is notable for the
finding in Figure A. You instruct the patient to use a hot compress on her eye for several days. After 3 weeks,
she returns with persistence of the lesion. Findings on physical exam demonstrate a lack of eyelashes around
the lesion. What is the most appropriate next step in the management of this patient?
1Biopsy
2Cephalexin
3Excision
4Observation
5Warm compresses

This patient most likely has a chalazion given the painless


swelling on her eyelid that progresses to a nodular, rubbery
lesion. Given the lack of response to warm compresses and
the eyelash loss, excision is necessary to rule out
malignancy.
Key features that should prompt further investigation
include gradual enlargement, central ulceration or induration, irregular borders, eyelid margin
destruction or loss of lashes, and telangiectasia.
Eyelid margin destruction or loss of lashes and progresses increasing size---consider malignancy

A chalazion is the result of meibomian gland obstruction and presents with a firm/rubbery and non-tender
nodule. The nodule forms on the conjunctival surface of the eyelid and is the result of granulomatous
inflammation. This condition is self-limiting and does not require treatment. If persistent or recurrent, it
may be a sign of meibomian gland carcinoma or sebaceous cell carcinoma. If there is any concern for
malignancy (such as eyelash loss), excision should be performed as it will remove the lesion for cosmetic
purposes and will allow for confirmation of the diagnosis.

Figure A displays a chalazion with a firm, rubbery nodule on the eyelid.

Answer 1: Biopsy to rule out malignancy would be suboptimal care. If this were a meibomian cyst, it is easily
removed (thereby ameliorating symptoms) and is preferred over biopsy which is roughly just as invasive yet
does nothing for symptom management. Given the concern for malignancy, excision is a better answer as it
gives tissue for diagnosis and removes the lesion.

Answer 2: Cephalexin is an oral antibiotic that covers for most skin flora; however, a chalazion is not an
infectious condition and will not respond to antibiotics.

Answer 4: Observation is not correct as this persistent lesion is concerning for malignancy warranting tissue
diagnosis and treatment.

Answer 5: Warm compresses can be used to treat a hordeolum which presents with a firm and tender eyelid. It
would not be appropriate management of a possible malignancy and this patient's symptoms already did not
responded to this treatment.

Bullet Summary:
A chalazion that is persistent or is associated with hair loss should be excised as there is concern for
malignancy.

Review:
Eyelid Lesions in general practice:

Only for Study purpose / Only Notes / AMC MCQ


Only Study Notes / Only for Study purposes / Personal notes/ AMC MCQ

Background
Patients with eyelid lesions often
present in the primary healthcare
setting. Although most eyelid lumps
are benign, accurate diagnosis and
early recognition of sinister lesions
leads to improved patient outcomes.

Objective
The aim of this article is to provide
an overview of common eyelid
lesions presenting to the general
practitioner.

Discussion
The majority of eyelid lesions are benign, ranging from innocuous cysts (cysts of Moll, Zeis and
epidermoid cyst) and chalazion/hordeolum to naevi and papillomas. Key features that should
prompt further investigation include gradual enlargement, central ulceration or induration, irregular
borders, eyelid margin destruction or loss of lashes, and telangiectasia. The presence of these
features should prompt referral to an ophthalmologist for further evaluation.

Patients with eyelid lesions often present in the primary care setting, with the majority of eyelid
lesions indicating benign pathology. Rarely, eyelid lesions can indicate malignant disease, which
has the potential for metastatic spread and associated mortality. Pattern recognition plays a key
part in accurate diagnosis and, combined with careful history-taking and clinical examination,
allows for prompt management or further investigation as required. This article will cover common
eyelid lesions and the approach to investigation and management, as well as key red flag signs
and rare conditions not to miss.

Only for Study purpose / Only Notes / AMC MCQ


Only Study Notes / Only for Study purposes / Personal notes/ AMC MCQ

Types of benign lesions


Lesions arising from glands

There are three types of glands located in the eyelids: Meibomian, Zeis and Moll. Blockage of any
of these glands results in corresponding focal collections/cysts, and these are commonly
encountered in general practice. Figure 1 shows some common benign lesions that occur around
the eyelid.

Figure 1. Common benign eyelid lesions: Translucent


cyst of Moll (upper left), cyst of Zeis filled with sebaceous
material (upper right), epidermal inclusion cyst filled with
keratin (lower left) and molluscum contagiosum (lower
right)

• Cysts of Moll arise from blocked apocrine


sweat glands found on the margin of the
eyelid. They are solitary dome-shaped
papules or nodules filled with clear fluid,
making transillumination a key feature.
• Apocrine hidrocystomas are common smooth cysts that are considered adenomas of the
secretory cells of Moll rather than classic retention cysts but clinically look similar and may
have a bluish colour.
• Cysts of Zeis arise from blocked sebaceous glands, also found on the eyelid margin. These
tend to be filled with yellow oily secretions and do not transilluminate.
• Chalazions represent focal granulomatous inflammation due to retained Meibomian gland
secretions from a blocked duct and are the most common lid lesion. Meibomian glands are
modified sebaceous glands present throughout the upper and lower eyelid, in and around
the tarsal plate. Chalazions present as painless nodules within the tarsal plate (posterior
aspect of lid) or at the lid margin and can vary in size. Associated blepharitis is common,
and clinical signs include eyelid margin telangiectasia, erythema and lash crusting.
Symptoms tend to reflect ocular surface discomfort and can include dryness, gritty
sensation and/or epiphora (watery eyes). Treatment involves hot compress, massage and
eyelid margin hygiene at least twice per day. Antibiotics are not required unless there is
evidence of superimposed bacterial infection. Conservative treatment is recommended for
at least 1–2 months prior to referral for consideration of incision and drainage.
• Hordeolums (styes) are acute bacterial infections of any of the above blocked glands and
are classified as either internal (Meibomian gland) or external (Zeis or Moll). Associated
preseptal cellulitis is common and can be treated with a course of oral antibiotics in addition
to hot compress and massage. Orbital cellulitis is a rare but potentially sight-threatening
and even life-threatening complication that occurs when infection breaches the orbital
septum. Differentiating preseptal from orbital cellulitis involves examination of visual acuity,
pupils, extraocular movements and the eyelids. A drop in visual acuity, relative afferent
pupillary defect or sluggish pupils, painful and limited eye movements, proptosis and
chemosis are suggestive of orbital cellulitis and require prompt referral to a tertiary centre
emergency department for imaging and antibiotic treatment.

Only for Study purpose / Only Notes / AMC MCQ


Only Study Notes / Only for Study purposes / Personal notes/ AMC MCQ

Epidermal lesions

Cysts of the epidermis are the second most common type of benign periocular lesion encountered.

• Epidermal inclusion cysts arise from the infundibulum of the hair follicle and are primarily
due to occlusion. These lesions are slow-growing, firm, elevated, round and often have a
central pore. Epidermal inclusion cysts are filled with keratin (despite being sometimes
called sebaceous cysts), and rupture can incite an inflammatory foreign body reaction.
Secondary infection is also possible. Referral for surgical excision is generally
recommended.
• Molluscum contagiosum is caused by the molluscum contagiosum virus, a member of the
Poxviridae family. Immunocompetent young children and patients who are
immunosuppressed are more commonly affected.1 Lesions are characteristically 1–3 mm
white, pink or flesh-coloured nodules with a central umbilication. Molluscum contagiosum
may also cause a follicular conjunctivitis because of shedding of the molluscum virus into
the tear film. Treatment is usually not required as the lesions will spontaneously resolve
over time. For rare cases that are very symptomatic (eg those that cause local irritation),
cryotherapy or curettage can be used, but referral to an ophthalmologist may be warranted
given the locally destructive treatment modalities.
• Xanthelasma are lesions in the superficial dermis and subdermal tissue containing lipid-
laden macrophages. These are sometimes associated with raised cholesterol or congenital
disorders of lipid metabolism requiring further investigation and management. Surgical
excision can be considered for cosmetic reasons.

Benign tumours

Benign tumours are also known as epidermal proliferations. The term papilloma refers to a group
of various benign epithelial proliferations that can affect the eyelid skin. Figure 2 shows common
benign eyelid lesions.

Figure 2. Benign proliferative lesions: Amelanotic


naevus (upper left) in comparison to pigmented
naevus (upper right), seborrheic keratosis (lower left)
and squamous papilloma (lower right)

• Seborrheic keratosis is the most


common benign lesion that affects elderly
patients and carries no risk of malignant
transformation.2 It is most commonly
described as a well-demarcated, waxy,
pigmented lesion with a ‘stuck on’
appearance. However, on the eyelid, seborrheic keratoses may be lobulated or
pedunculated. Excision can be considered for cosmetic purposes or if the seborrheic
keratosis is large enough to interfere with vision. If there are no concerns about the eyelid

Only for Study purpose / Only Notes / AMC MCQ


Only Study Notes / Only for Study purposes / Personal notes/ AMC MCQ

structure or function being affected, excision may be performed by the general practitioner
(GP).
• An acrochordon is also known as skin tag, fibroepithelial polyp and squamous papilloma.
These lesions are clinically in appearance: classical narrow-based pendunculated; broad-
based sessile; hyperkeratotic similar to cutaneous horn.
• Verruca vulgaris (viral wart) is caused by epidermal infection with human papillomavirus,
typically type 6 or 11. These lesions are most commonly found at the eyelid margin as non-
pigmented papules with digitations. Cryotherapy can be used to prevent viral spread, but
this should be performed cautiously around the eyelid area.

Benign melanocytic lesions

Pigmentation can be seen in both benign and malignant lesions and does not necessarily imply
melanocytic origin. Conversely, lesions of melanocytic origin may not be visibly pigmented (eg
amelanotic naevus).

• Naevi are flat or raised lesions that arise from melanocytes and are common on the eyelid.
Naevi progress through three stages, starting in childhood as a junctional naevus that is a
flat, pigmented macule located in the dermoepidermal junction. It then becomes an
elevated compound naevus in the second decade, involving both the dermoepidermal
junction and dermis, before involuting later in life and losing pigmentation by the seventh
decade. Third-stage naevi are known as dermal naevi and involve the dermis only.
Transformation to melanoma is rare, but can occur in junctional or compound naevi.

Vascular lesions
• Infantile haemangiomas, previously known as strawberry or capillary haemangiomas, are
lesions with a classic red/pink appearance that are common in children. The vast majority of
these lesions involute and self-resolve by the age of 10 years. Rarely, they can cause
ptosis, refractive error and amblyopia. All children with eyelid infantile haemangiomas
should be referred to a paediatric ophthalmologist to assess for potential refractive error
and monitor for amblyopia.3
• Port wine stains are permanent capillary malformations that are present from birth. They
are variably sized macular lesions that can be dark red to blue in colour. The lesion grows
in proportion with the child and may also gradually become raised and thicker. Port wine
stains involving the eyelid may be associated with glaucoma; approximately 18% of children
are affected,4 particularly patients with bilateral port wine stain or involvement of both upper
and lower eyelids. The prevalence is further increased when associated with Sturge-Weber
syndrome,5 with glaucoma affecting 40–60% of these patients.6,7 It is therefore important
for the patient to be referred to a paediatric ophthalmologist for monitoring and regular
follow-up.

Benign lesions summary

Benign lesions that should be referred include:

• hordeolum with signs of orbital cellulitis – emergency department

Only for Study purpose / Only Notes / AMC MCQ


Only Study Notes / Only for Study purposes / Personal notes/ AMC MCQ

• enlarging pigmented eyelid lesion with suspicious features – urgent oculoplastics


ophthalmology outpatient referral
• eyelid infantile haemangioma or port wine stain – urgent paediatric ophthalmology
outpatient referral
• chronic chalazion that is persistent despite 2–3 months of conservative treatment – non-
urgent ophthalmology outpatient referral
• benign lesion that is cosmetically concerning – non-urgent ophthalmology or dermatology
outpatient referral.

Sinister signs: When to suspect a malignant lesion

Most eyelid lesions are not malignant, but careful history-taking and examination will reveal
important clues when suspecting a more sinister lesion. Skin cancer is Australia’s most common
cancer, with two-thirds of Australians being diagnosed with a skin cancer before the age of 70
years.8 The eyelids are involved in 10% of cases of skin cancer,9 and the vast majority are basal
cell carcinomas (BCC). Risk factors for developing skin cancers, as with systemic cutaneous skin
cancer, include: fair skin, history of previous skin cancer, excessive sun exposure, previous
radiation, immunosuppression and smoking.10

Clinical signs that should arouse suspicion are listed in Table 1. Anatomically, lesions of the
medial canthus are known to be more aggressive because of proximity to the lacrimal drainage
system, increasing the risk of tumour spread.11 Eversion of the upper eyelid during examination
can reveal lesions otherwise missed with an external-only exam or a more extensive lesion than
initially expected (Figure 3). Loss of lashes and/or destruction of eyelid architecture is often a
telltale sign of malignancy. Central ulceration, especially if associated with ‘flaky’ skin, can
represent the more aggressive squamous cell carcinomas (SCCs). When suspecting a malignant
lesion, it is essential to examine the regional lymph nodes (preauricular in particular), as well as
check sensation around the periocular area, particularly in the distribution of the infraorbital nerve
for lesions of the lower eyelid or lateral canthus. Any loss of sensation may indicate perineural
invasion.

Table 1. When to suspect malignancy

History Exam

• Risk factors: prior skin cancer, fair skin, previous • Ulceration


radiation, immunosuppression • Induration
• Gradual enlargement • Irregular or ‘pearly’
• Painless borders
• Destruction of eyelid
margin
• Loss of lashes
(madarosis)
• Telangiectasia
• Reduced sensation

Only for Study purpose / Only Notes / AMC MCQ


Only Study Notes / Only for Study purposes / Personal notes/ AMC MCQ

Rarely, eyelid lesions can indicate highly malignant disease with potential for distant spread.
Melanoma of the eyelid is extremely rare, accounting for <1% of eyelid lesions. More common but
less well known is sebaceous carcinoma, a clinical and histological ‘masquerader’ with up to a
30% five-year mortality.12 Sebaceous carcinoma often presents as chronic eyelid inflammation or
a non-resolving chalazion that gradually increases in size. Figure 3 describes a case of sebaceous
carcinoma initially misdiagnosed as a chronic chalazion.

Figure 3.
Importance of
eyelid eversion:
An extensive
upper eyelid
sebaceous
carcinoma, the
full extent of
which can be
better
appreciated with
eversion. This
patient presented with chronic upper eyelid changes initially misdiagnosed as chalazion.

If a malignant lesion is suspected, the patient should be referred to either an oculoplastic


ophthalmologist or to a tertiary care hospital for further evaluation. Where possible, a biopsy of the
lesion will be performed for preoperative planning, and intraoperative margin control is preferred
for any malignant lesion. Although the majority of lid lesions encountered have low metastatic
potential, larger lesions are associated with more difficult reconstruction and associated ocular
morbidity. Early diagnosis is key. Malignant lesions encountered are described in the following
section, and Figure 4 shows commonly encountered malignant and premalignant lesions.

Figure 4. Malignant eyelid tumours: A basal cell


carcinoma presenting as an isolated nodule with
associated telangiectasia and loss of lashes
(upper left), keratoacanthoma (upper right),
actinic keratosis (lower left) and invasive
squamous cell carcinoma with central ulceration
(lower right)

Only for Study purpose / Only Notes / AMC MCQ


Only Study Notes / Only for Study purposes / Personal notes/ AMC MCQ

Types of malignant lesions


Basal cell carcinomas

BCCs are the most common eyelid malignancy, accounting for 80–90% of cases, and are typically
found on the lower eyelid or medial canthus.13 These are typically slow-growing, locally-
destructive skin cancers with a low propensity for metastatic spread. A classical presentation of a
BCC is the nodular type – a smooth, pearly-edged nodule with telangiectasia. As the tumour
enlarges and outgrows its nutrient supply, central tumour necrosis may occur, leaving a central
ulcer. Multiple other histological subtypes of BCC have been described including morphoeic,
superficial, infiltrative, micronodular and linear.14 Most cannot be distinguished clinically, but if
initial biopsy reveals an aggressive subtype, surgical planning can be adapted accordingly.14
Complete surgical excision with intraoperative margin control is the gold-standard treatment for
eyelid BCC, regardless of histological subtype, and is associated with low rates of recurrence.

Actinic keratosis

Actinic keratosis is a common precancerous skin lesion that has potential to develop into an SCC
and is found on sun-damaged skin. The risk of malignant transformation is only 0.24% per year,
but over an extended follow-up period, the incidence of SCC in an individual with multiple actinic
keratoses is as high as 12–16%.2 Actinic keratoses present as scaly, hyperkeratotic plaques with
a sandpaper-like texture. Although they are usually treated with cryotherapy, when located on the
eyelid margin, surgical excision is generally recommended. Topical imiquimod is an alternative
option that requires dermatology referral.

Keratoacanthoma

Keratoacanthoma has previously been considered a benign lesion but more recently is regarded
as a low-grade SCC for which complete excision is recommended. It has a characteristic dome-
shape with a central crater filled with keratin and may be associated with surrounding inflammatory
changes.

Squamous cell carcinomas

SCCs account for approximately 5% of epithelial tumours of the eyelid but are much more
aggressive than the more commonly encountered BCC.15 SCCs develop either spontaneously or
from actinic keratosis, Bowen’s disease (SCC in situ), keratoacanthomas or radiation dermatosis.
They are more common in immunocompromised patients, particularly following solid organ
transplant. SCCs can be difficult to distinguish from BCCs; however, they usually have more
scaling and may have an adherent crust or associated ulceration. Prompt referral is required given
the higher rates of regional nodal (24%), perineural (8%) and distant metastatic (6%) spread. 16 For
this reason, margin-controlled excision is the gold-standard treatment, although newer targeted
therapy involving the hedgehog pathway and epidermal growth factor receptor have been
promising in patients with advanced metastatic disease.17

Only for Study purpose / Only Notes / AMC MCQ


Only Study Notes / Only for Study purposes / Personal notes/ AMC MCQ

Sebaceous carcinoma

Sebaceous carcinoma is a rare but frequently misdiagnosed tumour (Figure 3). It accounts for
1.0–5.5% of all eyelid malignancies and was previously thought to be more common in people of
Asian ethnicity, although recent evidence suggests more equal incidence across all ethnicities.18
Sebaceous carcinoma arises from the Meibomian glands within the tarsus and presents either as
a solitary nodule or diffuse eyelid thickening, often with associated inflammatory changes.
Anatomically, given the similarities with chalazion, these lesions are often misdiagnosed initially.
Although sebaceous carcinomas are rare, any chronic chalazion associated with a gradual
increase in size or persistent, diffuse eyelid inflammation and associated thickening (sometimes
only evident with eyelid eversion) should raise suspicion. These tumours are aggressive, with
metastatic and mortality rates up to 30%, as well as an associated high risk of recurrence.12
Management generally involves excision with intraoperative margin control and conjunctival
mapping biopsies, or Mohs micrographic surgery.

Melanoma

Melanomas account for <1% of all eyelid malignancies.19 The lower eyelid is more commonly
involved. Any pigmented lesion that increases in size, has irregular borders or multiple colours or
is associated with ulcerating and bleeding warrants further investigation.

Malignant lesions summary


• An enlarging lesion, destruction of eyelid architecture, loss of lashes or induration/ulceration
(Table 1) may indicate malignant pathology.
• Urgent referral is required to outpatient pathway to oculoplastic ophthalmologist or tertiary
care centre.
• Malignant lesions are generally treated by clinical biopsy followed by surgical excision.

Conclusion

Patients with a variety of eyelid lesions present to the GP (Table 2). The majority of eyelid lesions
represent cystic, inflammatory or benign proliferative lesions. Skin cancers involve the eyelid
region in up to 10% of cases, with the vast majority being BCCs, which generally have an
excellent prognosis when excised completely. An understanding of the basic anatomy of eyelid
lesions, targeted history-taking and an eye for key red flag clinical features will ensure that more
sinister lesions will not be missed. Any lesion with features of malignancy should be referred to an
oculoplastic ophthalmologist for further evaluation and management.

Only for Study purpose / Only Notes / AMC MCQ


Only Study Notes / Only for Study purposes / Personal notes/ AMC MCQ

Table 2. Types of benign and malignant eyelid lesions

Benign lesions Benign tumours Malignant tumours

Cystic Papillomas Basal cell carcinoma

• Cyst of Moll • Seborrheic • Nodular


• Cyst of Zeis keratosis • Morphoeic
• Epidermoid cyst • Squamous • Superficial
• Viral wart • Others
Chalazion
Naevi Squamous cell carcinoma
Hordeolum (stye)
Vascular • Actinic keratosis
(pre-cancerous
Molluscum contagiosum • In situ (Bowen’s)
• Infantile • Keratoacanthoma
haemangioma • Invasive
Xanthelasma

Vascular malformation (port Sebaceous carcinoma


wine stain)
Melanoma

https://www1.racgp.org.au/ajgp/2019/august/eyelid-lesions-in-general-
practice?fbclid=IwAR2qHbwGdfAzngHrcjWOUo5F5ttWVXu5N3aOhtoU7TkDi0HNRztIWcWp0Yw

33- No purulent discharge, no itchiness. No impairment of vision. Which eyedrop will you give?

A. Hypromellose
B. Ketotifen
C. Sodium cromoglycate
D. Prednisolone
E. Chloramphenicol
Hypromellose eye drops are also known as 'artificial tears'. They are used to relieve eye dryness and
soreness, particularly where the dryness is caused by a reduced flow of tears. They moisten, soothe
and lubricate the surface of your eye, making it feel more comfortable.
There are three common preparations using as an eye lubricant 1 hypomellose, sodium hydrochloride,
and sodium hyaluronic
• No itchiness rule out allergic cause

Only for Study purpose / Only Notes / AMC MCQ


Only Study Notes / Only for Study purposes / Personal notes/ AMC MCQ

• No discharge rule out conjunctivitis


• No impaired vision rule out glaucoma,uveitis,scleritis (painful loss of vision)
• So A artificial tears by excluding all others

34- Patient presented with acute glaucoma what is the first thing you should give:
1. Oral acetazolamide
2. Topical timolol
3. Topical pilocarpine
4. topical acetazolamide
5. Topical atropine
IV acetazolamide // Timolol – Carteolol

Primary open-angle glaucoma


In open-angle glaucoma, optic nerve damage results in a progressive loss of retinal ganglion cell
axons, which is manifested initially as visual field loss and, ultimately, irreversible blindness if left
untreated

The aim is to reduce IOP safely, effectively and asymptomatically, with minimal inconvenience. Four types of
drugs are used: beta blockers, the miotics (para-sympathomimetics and anti-cholinesterases), adrenergic
agonists and carbonic anhydrase inhibitors.

Prostaglandins (preferred therapy) — The topical prostaglandins are increasingly chosen as


initial monotherapy in open-angle glaucoma and have been consistently shown to be effective at
lowering IOP and well tolerated [17,20,22-25]. Prostaglandins have the advantage of once-daily
dosing and do not have the risk of systemic side effects seen with topical beta blockers.
Ocular side effects of prostaglandins include hyperemia, eye irritation, increase in the number and
length of eyelashes, and changes in iris and lash pigmentation; the latter two are more easily
noticed if only one eye is treated. Different prostaglandins may differ in side effects. In one
network meta-analysis of randomized trials, latanoprost had the lowest mean proportion of patients
with hyperemia (24 percent) while bimatoprost had the highest (59 percent)
Beta blockers — Beta blockers may be appropriate as initial therapy for those patients who
cannot afford a topical prostaglandin. Like prostaglandins, topical beta blockers (eg, timolol,
betaxolol) have a long duration of action which allows for once- or twice-daily dosing. They are
associated with few ocular side effects, although some patients have hyperemia or a burning
sensation in the eyes [28-30].
In the network meta-analysis discussed above [13], the mean reductions in IOP at three months
for beta blockers ranged from 4.51 mmHg (95% CI 3.85-5.24) for levobunolol to 2.24 mmHg (95%
CI 1.59-2.88) for betaxolol, which is slightly less than the reduction (5.61 mmHg) observed with the
most effective prostaglandin.
In contrast to prostaglandins, which have an excellent profile as far as lack of systemic side effects
or contraindications, topical beta blockers are contraindicated in some patients with pulmonary or
cardiac disease. In these patients, side effects are similar to those associated with systemic beta-
blocker therapy including worsening of heart failure, bradycardia, heart block, and increased
airway resistance. Airway obstruction has been reported with topical beta-blocker therapy even in
patients who do not have a history of airway disease [31,32]. Other potential side effects of topical
beta-blocker therapy include exercise intolerance, depression, and sexual dysfunction.

Only for Study purpose / Only Notes / AMC MCQ


Only Study Notes / Only for Study purposes / Personal notes/ AMC MCQ

Other drugs

Alpha adrenergic agonists (eg, brimonidine) appear to be similarly effective to beta blockers in
lowering IOP in open-angle glaucoma, but they are associated with a number of ocular side
effects including allergic conjunctivitis, hyperemia, and ocular pruritus [33]. The nonselective
agents (epinephrine) also may be associated with arrhythmia, hypertension, and tachycardia.
These agents might be considered when there is intolerance or formulary or cost issues, or in
combination with the preferred agents if adequate control is not achieved.

Topical carbonic anhydrase inhibitors do not appear to be as effective in treating open-angle


glaucoma compared with other pharmacologic therapy [28]. In the meta-analysis described
above [13], the most effective carbonic anhydrase inhibitor (dorzolamide) produced a mean
IOP decrease at three months of 2.49 mmHg (95% CI 1.85-3.13). These agents might be
considered when there is intolerance or formulary or cost issues with the preferred agents, or
in combination if adequate control is not achieved.

For Pilocarpine: Cholinergic agonists have fewer systemic adverse effects than beta blockers,
but ocular side effects such as fixed, small pupils, myopia, and increased subjective visual
disturbance related to coexistent cataract have decreased the utilization of these drugs,
especially over the last three decades as prostaglandins, alpha adrenergic agonists, and
topical carbonic anhydrase inhibitors became available. These agents are now rarely used
except in low-resource settings where newer drugs may be too expensive or unavailable.

Combination products are available for several classes of drugs, in various combinations.
Such combinations should not be used as initial treatment, but for patients requiring treatment
with two of these drug classes they offer convenience and cost advantages. Which
combinations are available where depends on national differences in drug approval and
regulation.

Netarsudil was approved in 2017 by the US Food and Drug Administration (FDA) for topical
use for open-angle glaucoma or ocular hypertension [34]. Netarsudil is a rho kinase inhibitor; it
is believed to reduce IOP by increasing the outflow of aqueous humor through the trabecular
meshwork. Licensing studies suggested that netarsudil is noninferior to timolol. Post-market
experience is necessary before definitive recommendations for this drug can be made.

Laser therapy — Laser therapy (trabeculoplasty) increases aqueous outflow by improving


drainage of aqueous humor through the trabecular meshwork. The procedure is performed as an
outpatient in the ophthalmologist’s office or at a surgicenter or hospital outpatient clinic.
Acute primary angle-closure glaucoma
Medical therapy — There are no available trials comparing medical options for treatment of acute
angle-closure glaucoma, and treatment recommendations are based on clinical experience [22].
When an ophthalmologist is available for consultation within one hour of patient presentation,
patients with signs or symptoms suggesting possible acute angle-closure should be referred for
emergency assessment and treatment.
When there is likely to be an hour or more delay before a patient can be seen by an
ophthalmologist, and the suspicion of an acute attack is high, empiric treatment should be initiated.
If the vision is normal, but other symptoms and signs suggest an acute angle-closure attack,
empiric treatment should only be given if the intraocular pressure is significantly elevated (eg, >40
mmHg). For an acute primary angle-closure attack, initial management involves prompt
administration of pressure-lowering eye drops. A possible regimen would be one drop each, one
minute apart, of [22]:

Only for Study purpose / Only Notes / AMC MCQ


Only Study Notes / Only for Study purposes / Personal notes/ AMC MCQ

0.5% timolol maleate;


1% apraclonidine; and

2% pilocarpine

We also suggest giving the patient 500 mg of oral or intravenous (IV) acetazolamide. The eye
pressure should be checked 30 to 60 minutes after giving pressure-lowering drops and
acetazolamide. If the eye pressure is still significantly elevated, the same drops could be given
again, but the patient should also be examined immediately by an ophthalmologist. Systemic
medications other than acetazolamide (such as IV mannitol) should be administered under the
guidance of an ophthalmologist, since angle-closure should be confirmed before they are given.
If medical treatment is successful in reducing IOP, as is most often the case, corneal edema and
eye pain will typically lessen or resolve. In refractory cases, the ophthalmologist may consider
performing an anterior chamber paracentesis to remove some aqueous humor and immediately
lower the eye pressure temporarily, which may help to break the attack. Once the attack is broken,
the treatment of choice is a peripheral iridotomy. If laser peripheral iridotomy fails to remain patent,
or the cornea is too cloudy to visualize the iris, surgical peripheral iridectomy may be necessary.
Laser peripheral iridotomy — This procedure creates a tiny hole in the peripheral iris through
which aqueous humor can flow and reach the angle [23]. Once the iridotomy is patent, pupillary
block is bypassed. A peripheral iridotomy is usually created with a laser.
The IOP is rechecked 30 to 120 minutes after the iridotomy is performed, and mild steroid drops
are given for several days. Repeat gonioscopy is then performed to determine if the angle is wider.
The pupil is dilated to ensure that the IOP does not rise significantly and to better assess any
glaucoma damage to the optic nerve.
Possible complications of laser peripheral iridotomy include:

Increased IOP

Inflammation

Laser burns to the cornea, lens, or retina


A ghost image in the vision


Increased rate of cataract formation [24,25]


Need for repeat treatment if the hole were to shrink or close spontaneously

The fellow eye should be examined. If a narrow angle is found, prophylactic laser peripheral
iridotomy should be performed to prevent future attacks of angle-closure glaucoma [24]. Untreated,
approximately 50 percent of fellow eyes in acute angle-closure glaucoma patients will have
another attack within five years [24,26].
Surgical peripheral iridectomy — In this procedure, the ophthalmologist makes an incision into
the anterior chamber and manually removes a small amount of iris tissue to create a passage for
aqueous humor to reach the angle.
Other surgery — In a few cases of angle-closure glaucoma, goniosynechialysis may be
performed in the operating room at some point after a peripheral iridotomy is in place. In
goniosynechialysis, adhesions that result in scarring of the angle are mechanically lysed in an

Only for Study purpose / Only Notes / AMC MCQ


Only Study Notes / Only for Study purposes / Personal notes/ AMC MCQ

attempt to restore some drainage function. Results tend to be better if the scarring has been
present for less than one year [27].
Cataract surgery (phacoemulsification) with an intraocular lens implant may resolve the issue of
acute or chronic primary angle-closure glaucoma in some patients by removing the lens that may
be crowding the angle [24,28-30]. A randomized trial of 62 Chinese patients with cataracts who had
been medically treated for acute primary angle-closure glaucoma found that early
phacoemulsification was more effective than laser peripheral iridotomy in preventing recurrence of
IOP rise [31]. In a 2017 meta-analysis, patients with angle-closure glaucoma experienced a 6.4
mmHg (95% CI, -9.4 to -3.4 mmHg) lowering of IOP at 12 months’ or longer follow-up [32]. A
multicenter study randomizing angle-closure patients to either clear lens extraction (ie, lens
removal without significant cataract) or laser peripheral iridotomy found that clear lens extraction
was more efficacious, with IOP 1.18 mmHg lower than after iridotomy (95% CI -1.99 to -0.38), as
well as more cost-effective [7].
Chronic angle-closure glaucoma — Laser peripheral iridotomy is the first step in treatment of
patients with chronic angle-closure glaucoma, to relieve any pupillary block component. The IOP
may remain elevated, however, if scarring has already damaged the drainage angle. In this case,
the remaining glaucoma is treated medically and surgically much as in open-angle glaucoma. (See
"Open-angle glaucoma: Epidemiology, clinical presentation, and diagnosis".)
Secondary angle-closure glaucoma — Secondary angle-closure glaucoma is managed by
treating the cause if possible. Controlling the IOP medically and surgically afterwards is the next
step, much as in open-angle glaucoma.

35- 40 year old woman of retro orbital pain and reduced visual acuity for 3 days. What is the most
appropriate ix?
1. Temporal artery biopsy??
2. CT scan
3. VEP
4. Fundoscopy
5. Gonioscopy
The most appropriate dx - MRI but that not in the
options so we go for VEP
VEP is an acronym for Visual Evoked Potential.

Diagnosis — In general, optic neuritis is a clinical


diagnosis based upon the history and
examination findings. Because important findings
on funduscopic examination help differentiate
typical from atypical cases of optic neuritis, an
ophthalmologic examination should be considered
an essential feature of the clinical evaluation.
Magnetic resonance imaging (MRI) study of the
brain and orbits with gadolinium contrast provides
confirmation of the diagnosis in most cases and
also provides an assessment of the risk of
subsequent multiple sclerosis (MS).
The connection between MS and optic neuritis is the
inflammation and loss of the myelin covering of your optic
nerve and retina.

Only for Study purpose / Only Notes / AMC MCQ


Only Study Notes / Only for Study purposes / Personal notes/ AMC MCQ

May be due to demyelination (associated with MS),


infection, parainfection or autoimmune disease.
Autoimmune optic neuritis: systemic lupus erythematosus,
Sjogren syndrome, ankylosing spondylitis, diabetes
mellitus, temporal arteritis, and sarcoidosis
Symptoms of optic neuritis
Symptoms of optic neuritis usually occur in one eye and might
include:

• pain on eye movement or a dull ache behind your eye


• vision loss or temporary reduction in vision
• loss or dulling of color vision
• reduced side vision
• flashing lights or flickering lights with eye movement

Other possible causes of optic neuritis


The exact cause of optic neuritis isn’t known. Along with MS,
there are other things that have been linked to the
development of optic neuritis including:

• neuromyelitis optica, an autoimmune disorder that can


affect your optic nerve
• bacterial infections, including Lyme disease, cat-scratch
fever, and syphilis
• viruses, like mumps, herpes, and measles
• sarcoidosis and lupus, both of which cause inflammation
• certain medications like quinine and some antibiotics

Magnetic resonance imaging — An MRI study of the brain and orbits with gadolinium contrast
provides confirmation of the diagnosis of acute demyelinating optic neuritis and important
prognostic information regarding the risk of developing MS.
Lumbar puncture — Lumbar puncture is not an essential diagnostic test in optic neuritis but
should be considered in atypical cases (eg, those with bilateral presentation, <15 years in age, or
symptoms suggesting infection)
Approximately 60 to 80 percent of patients with acute optic neuritis have nonspecific abnormalities
in the cerebrospinal fluid (CSF), including lymphocytes (10 to 100) and elevated protein [43].
Other CSF findings in optic neuritis can include [47]:

●Myelin basic protein in approximately

20 percent

●Immunoglobulin G (IgG) synthesis in

20 to 36 percent

●Oligoclonal bands (OCB) in 56 to 69

percent

The presence of OCB implies a higher risk


of developing MS. However, since OCB are

Only for Study purpose / Only Notes / AMC MCQ


Only Study Notes / Only for Study purposes / Personal notes/ AMC MCQ

also associated with white matter lesions on brain MRI, their presence is not clearly of
independent prognostic importance
ther testing — When there are relevant clues to an alternative diagnosis (table 2), measurement
of the erythrocyte sedimentation rate, antinuclear antibodies, and angiotensin converting enzyme
levels and serologic and CSF tests for Lyme disease and syphilis should be obtained [45,46]. (See
"Optic neuropathies".)
Fluorescein angiography — Fluorescein angiography is not routinely performed in the evaluation
of optic neuritis and is often normal. Up to 25 percent demonstrate either dye leakage or
perivenous sheathing [12]. These findings may identify patients at somewhat higher risk for
developing MS.
Visual evoked response — A delay in the P100 of the visual evoked response (VER) is the
electrophysiologic manifestation of slowed conduction in the optic nerve as a result of axonal
demyelination [48]. This test is not usually helpful in the diagnosis of acute optic neuritis, unless
there is a suspicion that the visual loss is functional.
Abnormalities in the VER can persist after recovery of full vision. At one year, 80 to 90 percent will
be abnormal; 35 percent will return to normal at two years [28,31,32]. The VER is often employed
to find evidence of previous, asymptomatic episodes of optic neuritis, but the sensitivity and
specificity are imperfect [1].
The multifocal VER is a technical advance that appears to be more sensitive and specific for
identifying optic neuritis, but this technology is not generally available [1,49].
Optical coherence tomography — Optical coherence tomography (OCT) measures the
thickness in the retinal nerve fiber layer and detects thinning in most (85 percent) of patients with
optic neuritis [48,50-53]. These abnormalities are also common in patients with MS who do not
have a clinical history of optic neuritis [54]. While lower values correlate with impaired visual
outcome, the utility of OCT as a prognostic tool is limited in that abnormal values do not show up
until early swelling disappears. In one study, OCT was less sensitive than VER in detecting
subclinical optic neuritis [55].
A number of studies have found that a greater severity of optic nerve injury seen on OCT suggests
neuromyelitis optica (NMO) rather than optic neuritis associated with MS [56-58].
Antibody testing — Serum NMO antibody testing is suggested for individuals with recurrent optic
neuritis, particularly if the MRI brain is negative for any abnormal T2/FLAIR lesions outside of the
affected optic nerve(s) [59].
Patients with recurrent optic neuritis appear to be particularly at risk for NMO or Devic disease.
This is particularly true for patients with a normal brain MRI and those with optic neuritis events in
rapid succession or with a presentation of severe vision loss [60]. In one series of 51 patients with
either severe or recurrent optic neuritis, six patients were seropositive for the aquaporin-4-specific
serum autoantibody, a sensitive biomarker for NMO, while 10 patients were seropositive for
antibodies to myelin-oligodendrocyte glycoprotein (MOG), which has also been associated with
NMO [61-64]. In other studies, seropositivity for the aquaporin-4-specific serum autoantibody was
predictive of subsequent NMO among patients with recurrent optic neuritis [65,66]. MOG antibody
testing is recommended for patients with recurrent optic neuritis who test negative for aquaporin-4-
specific serum autoantibody.
The evaluation and treatment of neuromyelitis optica spectrum disorders are discussed in detail
separately
Progression and treatment

Visual recovery begins within a few weeks. Most achieve 20/40 vision or better at one year.
When vision is more severely affected at onset, a good prognosis is less certain.
●In adults, 30 percent of individuals will develop multiple sclerosis (MS) at five years. The risk

is higher, 56 percent, in those with two or more typical white matter lesions on magnetic

resonance imaging (MRI). Younger children may be less likely than adults to develop MS.

Only for Study purpose / Only Notes / AMC MCQ


Only Study Notes / Only for Study purposes / Personal notes/ AMC MCQ

●We suggest treatment with intravenous methylprednisolone for children and adults with either

severe vision loss or two or more white matter lesions on MRI (Grade 2A). Treatment is

associated with a more rapid recovery of vision and with delayed onset of MS, but does not

impact long-term visual function. (See 'Corticosteroids' above.)

We recommend not using oral prednisone, which does not affect short- or long-term visual
outcomes in acute optic neuritis and may be associated with an increased risk of recurrent
optic neuritis - IV methylprednisolone 1g/day single or divided dose x 3 days followed by
tapering oral steroid course.
In infectious causes, steroids should be withheld until appropriate antimicrobial agents are started.

●Individuals with acute optic neuritis and two or more white matter lesions on brain MRI are

considered to have a clinically isolated syndrome suggestive of MS. If treatment is tolerated,

potential benefits as demonstrated by randomized clinical trials include a longer attack-free

interval, reduced number of demyelinating attacks, and a delay to MS-associated disability.

https://www.ncbi.nlm.nih.gov/pmc/articles/PMC4878987/
Giant cell arteritis (GCA), also known as temporal arteritis, is a systemic autoimmune disease primarily
affecting the elderly. It is characterized by granulomatous inflammation of large and medium-sized arteries
[1]. Arteritic anterior ischemic optic neuropathy (AAION) is the most common type of ophthalmic
complication of GCA, and can cause permanent visual loss [2]. Therefore, AAION represents a true
ophthalmic emergency, because the possibility of visual loss is very high if it is not recognized and treated
promptly [3]. There have been few reports of GCA among Asians, and only one involving a patient with
silent GCA-associated AAION in Korea [2]. Herein, we report a case of GCA-associated AAION with typical
systemic GCA symptoms in an elderly Korean man, as confirmed by temporal artery biopsy.

40- Eye pic of an old man with sudden onset of pain, dilated pupil, and red eye. T/m?
1-Atropine
2-Timolol
3- pilocarpine
4- trabeculoplasty (open angle)
5- steroid

Dx: Acute Angle Closure Glaucoma


• Stamp= Supine -Timolol- Acetazolamide -
Mannitol- Pilocarpine
• For surgical treatment = Open is
trabeculectomy - Close iridotomy

Only for Study purpose / Only Notes / AMC MCQ


Only Study Notes / Only for Study purposes / Personal notes/ AMC MCQ

Emergency Department Care The


treatment of acute angle-closure glaucoma
(AACG) consists of IOP reduction,
suppression of inflammation, and the
reversal of angle closure. Once diagnosed,
the initial intervention includes
acetazolamide, a topical beta-blocker, and
a topical steroid. Acetazolamide should be
given as a stat dose of 500 mg IV followed
by 500 mg PO. A dose of a topical beta-
blocker (ie, carteolol, timolol) will also aid
in lowering IOP. (Medscape)
- Manitol and Acetazolamide used to fast
decrease the intraocular pressure.
- Topical corticosteroids to limit
inflammation
- Miotic drug: Pilocarpine to break the
pupilar lock.
- Topical Betablockers alpha-2 agonist
also indicated.
(CTO ophthalmology)

Based on the clinical findings, acute


closed-angle glaucoma is the most likely
diagnosis. Treatment of acute closed-
angle glaucoma is with immediate
application of topical agents that inhibit
aqueousproduction.
• The following topical agents might
be used:
o Topical beta blockers (first-line): timolol, carteolol
o Alpha adrenergic agonists: e.g. apraclonidine

Only for Study purpose / Only Notes / AMC MCQ


Only Study Notes / Only for Study purposes / Personal notes/ AMC MCQ

o Topical prostaglandins
• Other management options depend on the setting:
o If the patient can be seen within 1 hour of presentation,
▪ urgent referral to an ophthalmologist will be the next best step.

▪ If the referral is delayed, the patient should be given acetazolamide PO (250mg x2).
▪ After one hour of treatment topical pilocarpine (option can be started as well (2
doses 15
minutes apart).
• NOTE – The most frequently drug group in the emergency department is topical beta blockers
(timolol, carteolol) and intravenous acetazolamide

Acute AC: Immediate medical therapy in acute AC consists of commencing IOP-lowering eye medications
such as topical β-blocker, α2-agonist and even prostaglandin analogues as soon as possible. Once the IOP
is sufficiently reduced to allow iris reperfusion, pilocarpine is instilled to induce miosis in an attempt to
widen the anterior chamber angles and reestablish aqueous outflow.

41- (repeated questions) Picture of red eye with complain of severe pain, no discharge. Treatment asked -
- (Looks like scleritis)
a) Topical hydrocortisone
b) Topical methylprednisolone
c) Chloramphenicol
• Anterior scleritis NSAIDs plus systemic steroids n posterior
systemic steroid
• Scleritis is a serious condition that can lead to loss of vision.
So scleritis needs to be treated as soon as you notice
symptoms. Treatment varies depending on the type of
scleritis.
• Treatment can include corticosteroid pills nonsteroidal anti-
inflammatory drugs (like aspirin or ibuprofen) for pain and inflammation eye solutions or antibiotics
immunosuppressive drugs (drugs that weaken the immune system) when the condition is severe

Only for Study purpose / Only Notes / AMC MCQ


Only Study Notes / Only for Study purposes / Personal notes/ AMC MCQ

Only for Study purpose / Only Notes / AMC MCQ


Only Study Notes / Only for Study purposes / Personal notes/ AMC MCQ

Only for Study purpose / Only Notes / AMC MCQ


Only Study Notes / Only for Study purposes / Personal notes/ AMC MCQ

42- Young female complaining of eye pain, no watery eyes, no


purulent discharge, no ulcer on cornea on examination. Picture
given of left eye, woman looking down and medial, sclera
filled with blood vessels. Asking for topical treatment
a) Acyclovir
b) Prednisolone Self resolving if not Topical Steroids!
c) Timolol

Episcleritis

Only for Study purpose / Only Notes / AMC MCQ


Only Study Notes / Only for Study purposes / Personal notes/ AMC MCQ

Monocular attack of diffuse episcleritis in a patient with relapsing polychondritis. There is diffuse injection of
the episcleral vessels, but the underlying sclera is normal

Bilateral anterior scleritis in a patient with Cogan's


syndrome, manifested by ocular pain and erythema. (The
patient also had vertigo and bilateral sensorineural
hearing loss as a complication of her Cogan's syndrome.)

Nodular anterior scleritis in a patient with rheumatoid


arthritis. Nodular elevation of the sclera is surrounded by
deep scleral vascular engorgement

Only for Study purpose / Only Notes / AMC MCQ


Only Study Notes / Only for Study purposes / Personal notes/ AMC MCQ

In Scleritis emergency? → Refer to ophthalmologist JM

Dr Simra’s Bullet point: !


Episcelrits
Painful But no discharge
Normal vision
Self resovlong
Blanches on phenylpherine drops too
And in scleritis Refr to ophthalmologist -
Screen for secondary disease and treat
accordingly

Only for Study purpose / Only Notes / AMC MCQ


Only Study Notes / Only for Study purposes / Personal notes/ AMC MCQ

43- a man with painful eye had cataract as well, his eye moment is not affected, vision is good, asking
what to do in investigation
A slit lamp examination or Gonioscopy-for glaucoma
Indications for the use of a slit lamp in the nonophthalmology setting include any acute condition that
requires magnification to inspect the anterior segment of the eye (ie, lids, lashes, conjunctiva, cornea,
anterior chamber, iris, and lens) or to facilitate ocular foreign body removal. Thus, it is well suited for
diagnosing conditions such as corneal epithelial defect, keratoconjunctivitis, hyphema, hypopyon, lens
dislocation, herpetic infections, iritis, or evaluation of the red eye.
The slit lamp is less useful in diagnosing conditions of the posterior segment (ie, vitreous, fundus, optic
disk) in the primary care office or emergency department. Such conditions include papilledema, vitreous
hemorrhage, and retinal detachment. Diagnosis of such conditions with a slit lamp is feasible only if special
attachments are available and the examiner has training and experience in their use

Diagnostic tests Angle-closure glaucoma


Gonioscopy — Gonioscopy is the gold-standard method of diagnosing angle-closure glaucoma. This
technique involves using a special lens for the slit lamp, which allows the ophthalmologist to visualize the
angle

Left: Patient undergoing


gonioscopy. Right: Anterior
chamber angle as viewed on
gonioscopy

Slit lamp grading of anterior chamber depth — In this technique, the width of the angle is estimated by
shining a light beam
from the slit lamp on the
peripheral anterior
chamber. It is not as
reliable as gonioscopy
for diagnosing
angle- closure
glaucoma.

Initial for

glaucoma— tonometry Confirmatory— gonioscopy

Only for Study purpose / Only Notes / AMC MCQ


Only Study Notes / Only for Study purposes / Personal notes/ AMC MCQ

46- Trachoma outbreak .....


a) Chloramphenicol to patients
b) Chloramphenicol as a prophylaxis
c)Surgery for trachoma
S.A.F.E. 3,4: Surgery to correct the advanced, blinding stage
of the disease (trichiasis), Antibiotics to treat active infection,
Facial cleanliness and, Environmental improvements in the
areas of water and sanitation to reduce disease transmission
Acute chlamydial conjunctivitis
Chlamydia trachomatis conjunctivitis (adult inclusion
conjunctivitis) usually presents as an acute or subacute unilateral
conjunctivitis with mucopurulent discharge, or as a chronic
conjunctivitis. It should be considered as a differential diagnosis of
bacterial or viral conjunctivitis.

Chlamydial conjunctivitis usually occurs in neonates, and in


adults with exposure to sexually transmitted infections.
Infection in a neonate, infant or child may reflect mother-to-child
transmission, accidental transmission or sexual abuse

Conjunctival swabs for nucleic acid amplification testing (NAAT)


(eg polymerase chain reaction [PCR]) are recommended in all
neonates, and in patients with persistent conjunctivitis when C.
trachomatis is suspected.

Only for Study purpose / Only Notes / AMC MCQ


Only Study Notes / Only for Study purposes / Personal notes/ AMC MCQ

Systemic treatment is necessary. There is no evidence that concomitant topical therapy improves outcomes. For
adults and children older than 1 month, use:

azithromycin 1 g (child: 20 mg/kg up to 1 g) orally, as a single


dose.

For neonates, use:

azithromycin 20 mg/kg orally, daily for 3 days.

Approximately 50% of neonates with chlamydial conjunctivitis have associated chlamydial pneumonia
(see Pneumonia caused by Mycoplasma pneumoniae or Chlamydophila (Chlamydia) species).

Treat the mother of the infected neonate for C. trachomatis infection—for treatment regimens, management of
sexual contacts and other information, see Approach to Chlamydia trachomatis infection.

Refer patients with chlamydial conjunctivitis who have pain, photophobia or reduced vision to an ophthalmologist
because corneal complications can occur.

Trachoma
Trachoma is a form of chronic C. trachomatis conjunctivitis caused by repeated infections with C.
trachomatis serotypes A, B, Ba or C. It is the leading cause of preventable infectious blindness in the world,
especially in developing countries, and is still common in remote Aboriginal and Torres Strait Islander
communities in Australia. Without treatment, recurrent infection can lead to scarring of the eyelids, corneal
ulceration, corneal scarring and loss of vision. Suspect trachoma in all cases of conjunctivitis in endemic areas.
Treatment is with azithromycin (see Acute chlamydial conjunctivitis above for dosage).

In areas where trachoma is prevalent, regular face washing and treatment of household contacts is
recommended. Community-wide treatment may be required in areas where prevalence is high. For further
information on public health management of trachoma, see the Communicable Diseases Network
Australia National guidelines for the public health management of trachoma
https://www1.health.gov.au/internet/main/publishing.nsf/Content/cdna-song-trachoma.htm

https://tgldcdp.tg.org.au/viewTopic?topicfile=conjunctivitis&guidelineName=Antibiotic#toc_d1e448
Contact management
a. Contacts of cases identified through community screening:

Active trachoma community prevalence in 5-9 year old Aboriginal and Torres Strait Islander children:
i. ≥20%: Treat all people >3kg living in households with children <15 years of age.
ii. ≥5 to < 20% and there is no obvious clustering of cases: Treat all people >3kg living in households with children <15
years of age.
iii. ≥5 to < 20% and cases are obviously clustered within several households and health staff can easily identify all
household contacts of cases: Single-dose azithromycin to all people >3kg living in households with an active trachoma
case.
iv. <5%: Treat all people >3kg living in households with an active trachoma case.

b. Contacts of cases identified outside of screening programs


Treat people >3kg who living in the same household (s) as the case

Only for Study purpose / Only Notes / AMC MCQ


Only Study Notes / Only for Study purposes / Personal notes/ AMC MCQ

Incubation period
The incubation period of C. trachomatis is 5 to 10 days.4 However most episodes of infection are reinfections and
usually occur in children with already established clinical disease.

Infectious period
The infectious period is 2 to 3 months. It may be shorter with repeated infections and decrease with age.

Clinical presentation and outcome


Cases of active trachoma are often asymptomatic or may present with discharging or red eyes.

Repeated infections lead to long-term inflammation, scarring of the tarsal conjunctivae and distortion of the upper
eyelid with inturning of eyelashes (trichiasis) that abrade the cornea. This constant abrasion, in turn, can cause
irreversible corneal opacity and blindness. This can be further complicated by secondary bacterial or rarely fungal
infection.

Persons at increased risk of disease


In Australia today, trachoma is only found in rural and remote Aboriginal and Torres Strait Islander populations and is
endemic in some parts of the NT, SA and WA.

Active trachoma is usually seen in young children and adolescents. The highest ocular chlamydial loads are found in
children younger than 5 years, with the greatest risk of infection in younger children. In contrast, trachomatous
trichiasis most commonly presents in older adults, usually over the age of 40 years.

Unlike developing countries where active trachoma lasts longer in girls than in boys and where trichiasis is more
common among adult women than men, Australian surveys have not identified any sex differences in active trachoma
or trichiasis prevalence.

Disease occurrence and public health significance


Trachoma is the leading cause of preventable infectious blindness in the world. Endemic in 53 countries, trachoma is
responsible for visual impairment in about 2.2 million people worldwide, of whom 1.2 million people are irreversibly
blind.6 The prevalence of blindness in Aboriginal and Torres Strait Islander people is 6 times higher than non-
Aboriginal people, and trachoma accounts for 9% of this blindness. 6

Australia is the only developed nation in the world to still have endemic trachoma. Although trachoma was eliminated
from most parts of Australia by the 1930s, it continues to be a significant public health problem in Aboriginal and
Torres Strait Islander communities in many rural and remote areas of the NT, SA and WA. The National Indigenous
Eye Health Survey8 in 2008 detected cases in other jurisdictions and mapping is ongoing. In Australia, the prevalence
of trichiasis is approximately 1.4% in Aboriginal and Torres Strait Islanders. 5

In line with its Vision 2020 initiative, the World Health Organization (WHO) has adopted a resolution to eliminate
blinding trachoma by 2020. Australia is a signatory to this resolution, the Global Elimination of Trachoma (GET 2020).

Only for Study purpose / Only Notes / AMC MCQ


Only Study Notes / Only for Study purposes / Personal notes/ AMC MCQ

Figure 1. Trachoma prevalence in 5-9 year old Aboriginal and Torres Strait Islander children9, 2011

Routine prevention activities


The WHO GET 2020 initiative is built on the
implementation of the SAFE strategy for the
effective prevention and control of trachoma.

• S – Surgery for trichiasis: Surgical


procedures to reduce impact of trichiasis
• A – Antibiotics: Antibiotic (azithromycin)
treatment of individual active trachoma cases and
to reduce the community reservoir of infection
• F – Facial cleanliness: Promote clean faces
to reduce spread of infection
• E – Environmental health – improve water
access, good sanitation, waste and fly control,
and reduce overcrowding
top of page

The acronym SAFE covers four public health


components, and in order of public health priority
are:

The ‘E’ component ‘environmental health’ covers a very broad category of potential activities.

Safe access to clean and functioning water supplies, adequate sanitation including clean linen and aired mattresses,
improved housing, reducing overcrowding and attempts to minimise fly density are all potentially important factors for
trachoma control.

The “F” component ‘facial cleanliness’ is seen as the key preventive measure that can be taken to prevent infection.
Facial cleanliness is the absence of nasal and ocular discharge. 10 It requires the proper maintenance of housing,
especially washing facilities and bathrooms, and the development of household and personal hygiene skills and
behaviours. Facial cleanliness in children should be promoted by including regular face-washing as part of a holistic
personal hygiene program, which may also include tooth-brushing, hand washing and general hygiene

The “A” component ‘antibiotic distribution’ of the strategy has an important role in prevention by reducing the
duration of infection (both symptomatic and asymptomatic), thereby reducing disease transmission (see section 9)

The “S” component ‘surgery’ of the strategy involves the detection, referral and surgical management of entropion
(in-turned eye lid margin) and trichiasis (in-turned eye lashes) to prevent further corneal abrasion and the
development of corneal scarring and blindness

Clinical diagnosis and laboratory testing

Active Trachoma
The diagnosis of active trachoma is a clinical diagnosis based on the WHO simplified grading system as outlined
below (Table 1). Trachomatous inflammation - Follicular (TF) and Trachomatous inflammation Intense (TI) are
indications of active trachoma and are usually found in children and teenagers, but may occasionally occur in older
persons. Each sign is individually graded as being absent or present. One or more signs can, and often do, occur
together (see Appendix 2).

Only for Study purpose / Only Notes / AMC MCQ


Only Study Notes / Only for Study purposes / Personal notes/ AMC MCQ

Chronic sequelae of trachoma


Trachomatous conjunctival Scarring (TS) is sequelae of trachoma and a sign of severe damage which signals the risk
of developing trichiasis. Trachomatous Trichiasis (TT) is a result of severe distortion and scarring in the upper eyelid
and the rubbing lashes lead to corneal scarring and corneal opacity (CO) which represents late stages; both are
usually found in older adults see Table 1).

top of page

Table 1 WHO simplified grading system for trachoma

Grade Signs

Infectious TF Trachomatous inflammation- Presence of 5 or more follicles of >0.5mm in


Follicular diameter on the upper tarsal conjunctiva

TI Trachomatous inflammation - Presence of pronounced inflammatory thickening


Intense of the upper tarsal conjunctiva obscuring more than
half of the normal deep tarsal vessels

Non- TS Trachomatous conjunctival Scarring Presence of easily visible scars on the upper tarsal
infectious conjunctiva

TT Trachomatous Trichiasis Presence of at least one in-grown eyelash touching


the eyeball, or evidence of recent removal of in-
turned lashes

CO Corneal Opacity Presence of corneal opacity blurring part of the


pupil margin

See the ‘WHO simplified grading card’ (Appendix 2) and the on-line training package for clinicians at
(http://www.iehu1.unimelb.edu.au/trachoma/)

For a list of resources required and directions on how to screen for trachoma see Appendix 3.

Laboratory tests including nucleic acid amplification tests (e.g. polymerase chain reaction or PCR) are available but
the results do not correlate well with clinical signs and are not recommended for routine use.

Case Management
This addresses the antibiotic (A) and surgery (S) elements of the WHO strategy.

Active trachoma
Trachoma control programs should be conducted on a regional or state level. Screening and treatment at a population
level is the best method to decrease the prevalence of trachoma. Treatment at a population level reduces the pool of
infection circulating in the community. Health services should not conduct opportunistic trachoma screening
indiscriminately but should work with the identified at risk communities. However, on the occasion that an individual

Only for Study purpose / Only Notes / AMC MCQ


Only Study Notes / Only for Study purposes / Personal notes/ AMC MCQ

presents to a clinic with symptoms indicating trachoma, the individual should be examined, and if trachoma is
diagnosed, the case and their contacts should be treated at the same time.

Response times
Cases of active trachoma and their contacts should be treated at the same time, regardless of whether the case was
found on screening or presented with symptoms (See Contact Management – Section 11). All members of the
relevant household/s should be treated within one week of commencement of treatment.

Where community-wide treatment is being undertaken (see Section 11), contacts, including children >3kg, should be
treated together at one time and all treatment in the community should be completed within a two week period. As the
population in remote communities is highly mobile, completing treatment within this timeframe will minimise the
likelihood of re-infection and achieve higher population coverage.

Response procedure

Case investigation
The diagnosis of active trachoma is based on clinical examination. Contacts of case need to be identified.

Case treatment
If a diagnosis of trachoma is made clinically the case and all contacts should be treated.

The target is for 100% of active cases to receive treatment, and for 85% of contacts to receive treatment.

It is important to remember that treatment of cases of active trachoma found during screening programs should not be
managed separately from the treatment of households and communities. Reducing the prevalence of trachoma in a
community is dependent on completing all treatment in as short a timeframe as possible (See Section 11. Contact
Management for details).

• Azithromycin in a single dose is recommended for the treatment of both cases and their contacts >3kg.
• The recommended dose is Azithromycin 20mg/kg (maximum dose of 1000mg) orally as a single dose. Dosing
guidelines for trachoma management can be found in Appendix 5
• Single dose azithromycin is contraindicated only in the case of a known allergy i, and weight less than 3 kg. There are
no other contraindications for administration of single dose azithromyci

Treatment Schedules
Regardless of prevalence in the community, people >3kg living in households with an active trachoma case should be
treated with single-dose azithromycin at the same time as the case(s) according to the schedule in Appendix 4.
Treatment of the all household members including cases and their contacts should be completed within 1 week of
commencement of treatment. Treatment coverage of contacts should be at least 85%.

Treatment coverage of a community is defined as the proportion of Aboriginal and Torres Strait Islander people who
weigh >3kg and live in a house with 1 or more children aged <15 years old, who were treated for trachoma at the time
of community wide treatment. The denominator used to calculate treatment coverage should be based on the number
of Aboriginal and Torres Strait Islander people in such households in the community at the time that treatment is being
administered. For most communities this will be people in the community over the several days it takes to complete
the distribution of treatment; in the case of large communities treatment may take longer. People from places that do
not have a school should be included in the treatment coverage calculation of the community of the school that their
children attend.

Only for Study purpose / Only Notes / AMC MCQ


Only Study Notes / Only for Study purposes / Personal notes/ AMC MCQ

Consent for each occasion of treatment must be obtained, and both the consent and administration of treatment must
be appropriately documented in accordance with the medical records policy of the health service administering the
treatment.

Screening and treatment activities within the region, and ideally the, state or territory, should be completed in as short
a timeframe as possible to achieve high population coverage and minimise the likelihood of reinfection.

Table 2. Screening# and treatment schedule of contacts according to prevalence*.

Trachoma prevalence in Treatment Treatment frequency Screening frequency


screened children aged
5-9 years

≥20% Single-dose azithromycin 0, 6, 12, 18 & 24 months Screen at 36 months after


to people >3kg living in the initial screen (12
houses with children <15 months after the 5th
years of age treatment)*

≥5 to < 20% and there is Single-dose azithromycin 0, 12 & 24 months Screen at 36 months after
no obvious clustering of to people >3kg living in the initial screen (12
cases houses with children <15 months after the 3rd
years of age treatment)*

≥5 to < 20% and cases Single-dose azithromycin Once at 0 months. Screen at 1 year to
are obviously clustered to people >3kg living in Further treatment determine prevalence
within several households houses with an active determined by prevalence
and health staff can easily trachoma case at next screen
identify all household
contacts of cases

<5% Single-dose azithromycin Once at 0 months and Screen at 1, 3 and 5


to people >3kg living in retreat if trachoma is years, then cease if
houses with an active found on further prevalence <5% at each
trachoma case screening screen.

top of page

#Community treatment decisions are based on screening at least 85% of the target population. Every effort should be
made to reach this coverage level. If 85% coverage is not achieved the public health practitioner will need to make a
judgement whether a sufficiently high and representative proportion of the target population has been screened to
provide a basis for a valid estimate of prevalence on which decisions regarding treatment can be based.
* Jurisdictions where treatment coverage of at least 85% is clearly being achieved and with sufficient resources to
undertake annual screening should consider doing so. Treatment frequency should be based on prevalence from the
most recent screen.

Note: The target is for 100% of active cases to receive treatment, and for 85% of contacts to receive treatment.

Only for Study purpose / Only Notes / AMC MCQ


Only Study Notes / Only for Study purposes / Personal notes/ AMC MCQ

Antimicrobial resistance
Expert opinion is that concern regarding macrolide resistance in Indigenous communities should not discourage use of
programmatic azithromycin MDA for the elimination of trachoma. Resistance to macrolide antibiotics occurs very
rarely among Chlamydiae and cases of resistance have not been found in the context of MDA.

Special situations

Active trachoma in non-endemic situations


If an individual presents with symptoms consistent with follicular conjunctivitis in an area non-endemic with trachoma
the clinician should consider alternative diagnoses, see Appendix 6.

Appropriate history, examination and tests should be carried out and advice sought from an ophthalmologist if
required.

Clinical and public health management of cases and contacts of trachoma in non-endemic areas should be in
accordance with the trachoma SoNG.

If active trachoma is diagnosed in a non-endemic area in a person who has no epidemiological links with trachoma
endemic areas, the person making the diagnosis should inform the regional public health unit.

Remote or rural Aboriginal and Torres Strait Islander communities with no surveillance data
Some remote and rural Aboriginal communities may have environmental and health related features that resemble
those of Aboriginal communities where trachoma is, or has been endemic, but where there is no evidence for decision
making as to whether trachoma is present. In these situations a prevalence survey in 5-9 year olds should be
considered with the goal of determining whether the community is at risk. Such surveys require clinical assessment by
health staff trained in trachoma clinical diagnosis. Consideration may be given to obtaining ocular swabs for nucleic
acid amplification testing. In these circumstances caution needs to be exercised in interpreting results because of the
potential poor specificity of trachoma signs and symptoms in low endemic situations and the relatively weak sensitivity
of nucleic acid amplification testing

https://mspgh.unimelb.edu.au/__data/assets/pdf_file/0007/1977991/trachoma_resource_booklet.pdf (read
pdf at the end of this document) Important

47- (We need to check these questions) Rural indigenous community. 5 kids have trachoma. Contact tracing resulted
in 20 people. Community population is 250. What to do apart from treating index patient?

a. Nothing more

b. Treat contact

c. Treat community

d. Prophylactic drops to contact

e. Prophylactic drops to community

5 is t he 2% of 250 – treat contacts <5%

Following this: Treat contact

Only for Study purpose / Only Notes / AMC MCQ


Only Study Notes / Only for Study purposes / Personal notes/ AMC MCQ

To determine the prevalence of active trachoma in the community: (We didn’t have info to
do this so we improvised)

The key to treatment is to treat all members of a household (or family) in which one or more child has active
trachoma. When the prevalence is sufficiently high, it is logistically much easier to treat all members of the
community.
Based on evidence from other communities, WHO has recommended that when the prevalence of active
trachoma in children exceeds 10%, it is easier to treat the whole community. For Australia, at these
prevalences CDNA recommends treating all affected households if household clustering is obvious and
otherwise treating the whole community. Below this prevalence, treatment should be given at the
household level
https://emedicine.medscape.com/article/1202088-treatment

48- you are working in an area inhabited by aboriginals mostly, there is an epidemic of trachoma, advice
to community
1.face washing
2.tetracycline to all
3.Chloramphenicol ointment for pt only

SAFE protocol
The World Health Organization has targeted trachoma for elimination by 2020 through an innovative,
multi-faceted public health strategy known as S.A.F.E. 3,4: Surgery to correct the advanced, blinding
stage of the disease (trichiasis), Antibiotics to treat active infection, Facial cleanliness and,
Environmental improvements in the areas of water and sanitation to reduce disease transmission

49- Trachoma recall. Abt 7 cases with 20 something contacts in 240 community. How will you mgt
1) antibiotics drops to contacts
2) antibiotics drops to community
3) treatment of cases
4) treatment of contacts
5) treat whole community
240 – 100% / 7 is x = 3 % <5% Treat contacts

Only for Study purpose / Only Notes / AMC MCQ


Only Study Notes / Only for Study purposes / Personal notes/ AMC MCQ

50- A man came because he can’t see the temporal side in his right eye and nasal side of his left eye
field, where is the lesion?
a. Optic chiasma
b. Right optic
c. Left optic tract
D- Right optic
L
nerve

tract
R

Only for Study purpose / Only Notes / AMC MCQ


Only Study Notes / Only for Study purposes / Personal notes/ AMC MCQ

Right temporal and left nasal→ Means left side Means left optic tract Cus left optic tract means Left
nasal And right temporal

51- Patient presented with acute glaucoma what is the first thing you should give :
1. Oral acetazolamide
2. Topical timolol
3. Topical pilocarpine
4. topical atropine

52-Patient clinical features: Chemosis, blurring of vision, painful, photophobia, pupils semi dilated fixed and
irregular when compared to the other side. He has been having similar attacks in the past. What is the long
management of his condition? (angle closure glaucoma cus sudden onset)
A. Acetazolamide
B. Pilocarpine
C. Trabeculectomy
D Peripheral iridotomy

53- Given right eye picture (the whole eye red, no discharge) patient come with severe pain in eye,
injected and loss of vision and pupil is not reacting to light
a.prenisolone
b.timolol
c.chloramphenicol
d.surgery

54- 70-year-old man (kaung,


bo recall) woman (nyi recall)
complains of sudden
painless blurring of vision
in left eye. On examination,
total loss of left eye vision,
relative afferent pupillary
defect are detected. BP and
HR are normal.
1. MRI
2. Carotid Doppler
3. ESR >50
4. Forget two other options.
Dx: giant cell arteritis (GCA)
so dx can be CRAO GCA or
ON, but we need to do ESR
first in elderly sudden loss of
vision

Only for Study purpose / Only Notes / AMC MCQ


Only Study Notes / Only for Study purposes / Personal notes/ AMC MCQ

MRI of the brain plays a role in the evaluation of optic neuropathy or neuritis as MRI can demonstrate other
evidence of demyelinating disease. While this patient has a family history of multiple sclerosis, optic neuritis
usually presents with pain on eye movement

Patients with transient monocular visual loss and vascular risk factors should be evaluated for carotid artery
disease with ultrasound of the carotid arteries.
----Checking serum inflammatory markers such as erythrocyte sedimentation rate (ESR) is an appropriate
initial screening test for patients greater than 50 years of age to exclude the diagnosis of giant cell arteritis
(GCA). This patient is under age 50 and has vascular risk factors that make an ischemic etiology more
likely.---

Optic Neuritis: INTRODUCTION


Optic neuritis is an inflammatory, demyelinating condition
that causes acute, usually monocular, visual loss. It is
highly associated with multiple sclerosis (MS). Optic
neuritis is the presenting feature of MS in 15 to 20 percent
of patients and occurs in 50 percent at some time during
the course of their illness.
Most cases of acute demyelinating optic neuritis occur in
women (two-thirds) and typically develop in patients
between the ages of 20 and 40
Vision loss typically develops over a period of hours to
days, peaking within one to two weeks. Continued
deterioration after that time suggests an alternative
diagnosis [1,2]. Greater than 90 percent of patients in the
ONTT had a significant decrease in central visual acuity. In
most, the visual acuities ranged from 20/25 to 20/190
(median visual acuity 20/60). However, some patients had
20/20 acuity (11 percent), and, at the other extreme, a few
had no light perception (3 percent).
●Eye pain occurred in 92 percent of patients in the ONTT
and often worsened with eye movement [7]. The onset of
pain generally coincided with the visual acuity loss and
improved along with it.
Central retinal artery occlusion (CRAO) and branch retinal artery occlusion (BRAO) present
with acute, painless loss of monocular vision. These disorders are considered a form of stroke, with a
similar clinical approach and management; the clinician attempts to treat the acute event, find the source of
the vascular occlusion, and prevent further vascular events from occurring.

Only for Study purpose / Only Notes / AMC MCQ


Only Study Notes / Only for Study purposes / Personal notes/ AMC MCQ

Demographic characteristics of patients with


CRAO and BRAO are consistent with those
seen for other vascular disorders. The mean
age of patients is between 60 and 65 years.

The distribution of etiologies is also


influenced by the patient's age. Carotid
artery atherosclerosis is the most common
etiology for retinal artery occlusion overall
but is relatively unusual in patients under
40, in whom cardiogenic embolism is the
most likely etiology [12,13]. In patients over
70 years of age, giant cell arteritis (GCA) is
more likely than in younger patients.
Ethnicity may also be relevant; as an
example, carotid artery occlusive disease is
most prevalent among Caucasians [14].

Determining the underlying etiology is


important to preventing recurrent CRAO
and BRAO and other vascular
complications
In the acute phase, fluorescein angiography
shows slowed or absent filling of the central
retinal artery with normal filling of the choroid. If
the choroid is also affected, giant cell arteritis
(GCA) should be considered, especially in an
older adult patient
The erythrocyte sedimentation rate (ESR) and serum C-reactive protein (CRP) should be measured in all
individuals over age 50 with CRAO and no visible retinal emboli to exclude GCA.
If the ESR or serum CRP is elevated or if the clinical setting is very suggestive, the patient should be
evaluated for GCA and possible treatment with high-dose corticosteroids.
On funduscopic examination, ischemic retinal whitening is seen immediately after an occlusion of the
central retinal artery. A "cherry red spot" appears in the macula, where the retina is thinner and the retinal
pigment epithelium and choroidal vasculature can be seen more easily as the ischemic retina becomes less
translucent

The best way to confirm a diagnosis of giant cell arteritis is by taking a small sample (biopsy) of
the temporal artery. This artery is situated close to the skin just in front of your ears and continues up to
your scalp

Only for Study purpose / Only Notes / AMC MCQ


Only Study Notes / Only for Study purposes / Personal notes/ AMC MCQ

55- A little girl with severe pain in left eye and around
it, with discharge and fever. Hx of sinusitis. Dx
A. Blood culture
B. CT scan

Orbital cellulitis is an infection involving the contents of


the orbit (fat and ocular muscles). It must be
distinguished from preseptal cellulitis (sometimes
called periorbital cellulitis), which is an infection of the
anterior portion of the eyelid. Neither infection involves
the globe itself.
common signs and symptoms of orbital cellulitis
include pain with eye movement, sudden vision loss,
chemosis, bulging of the infected eye, and limited eye
movement. Along with these symptoms, patients
typically have redness and swelling of the eyelid, pain,
discharge, inability to open the eye, occasional fever
and lethargy. It is usually caused by a previous
sinusitis. Other causes include infection of nearby
structures, trauma and previous surgery.
The diagnosis of orbital cellulitis is suspected clinically
and can be confirmed by computed tomography (CT)
scanning.

Microbiologic studies — Despite the low yield, we recommend obtaining blood cultures in all patients with
suspected orbital cellulitis before the administration of antibiotics. In patients with concomitant acute

Only for Study purpose / Only Notes / AMC MCQ


Only Study Notes / Only for Study purposes / Personal notes/ AMC MCQ

bacterial sinusitis who are examined by an otolaryngologist, any purulent sinus drainage seen on
endoscopic examination should be cultured.

Antibiotic therapy — Most patients with uncomplicated orbital cellulitis can be treated with antibiotics
alone [4,5,66]. Even in patients who are initially treated with antibiotics alone, an ophthalmologist and an
otolaryngologist should be consulted because the physical examination requires ophthalmologic and/or
otolaryngologic expertise and surgery is sometimes required.
(See 'Surgery' below.

Initial therapy — Appropriate antibiotic regimens for initial


empiric treatment in patients with normal renal function include a
combination of:

●Vancomycin (in children: 40 to 60 mg/kg IV per day in three


or four divided doses, maximum daily dose 4 g; in adults: 15
to 20 mg/kg IV per dose every 8 to 12 hours, with a
maximum 2 g for each dose; the appropriate dose in adults
requires measurement of a trough concentration of vancomycin)
PLUS
●One of the following:
•Ceftriaxone (in children: 50 mg/kg per dose IV once or twice per day [the higher dose
should be used if intracranial extension is suspected], maximum daily dose 4 g/day; in
adults: 2 g IV every 24 hours [2 g IV every 12 hours if intracranial extension is
suspected]) or
•Cefotaxime (in children: 150 to 200 mg/kg IV per day in three doses, maximum daily
dose 12 g; in adults: 2 g IV every four hours)

Anaerobic coverage is warranted in certain situations:

●Until the possibility of intracranial involvement has been assessed and excluded, we
add metronidazole (in children: 30 mg/kg IV or orally per day in divided doses every eight
hours; in adults: 500 mg IV or orally every eight hours) for anaerobic coverage.
(See "Treatment and prognosis of bacterial brain abscess".)
●For patients who have orbital cellulitis associated with chronic sinusitis or an odontogenic
source, we also add metronidazole (at the doses above) if they are being treated with a
ceftriaxone- or cefotaxime-containing regimen given the involvement of anaerobes with such
processes. Metronidazole does not need to be added if the regimen contains

Diagnosis The diagnosis of orbital cellulitis is suspected clinically and can be confirmed

by computed tomography (CT) scanning. During the initial evaluation, it is critical to distinguish
preseptal cellulitis from the more serious orbital cellulitis. It is also important to look for
complications of orbital cellulitis, such as subperiosteal abscess, orbital abscess, visual loss, and
intracranial extension. Although both preseptal cellulitis and orbital cellulitis typically cause eyelid
swelling with or without erythema, the presence of ophthalmoplegia, pain with eye movement,
decreased vision, relative afferent pupillary defect, and/or proptosis occur only with orbital
cellulitis. Another finding that is more common with orbital cellulitis, and rare or absent with
preseptal cellulitis, is chemosis. Fever is more common with orbital cellulitis than with preseptal
cellulitis [2,4].

It is important for an ophthalmologist to evaluate patients with suspected orbital cellulitis in order to
evaluate the extraocular movements and visual acuity, and to assess for proptosis. In patients in
whom rhinosinusitis is also present, an otolaryngologist should generally be consulted as well.
(See 'Surgery' below.)

Only for Study purpose / Only Notes / AMC MCQ


Only Study Notes / Only for Study purposes / Personal notes/ AMC MCQ

Imaging studies — The goals of imaging studies are to support the diagnosis of orbital cellulitis and
to search for an abscess or other complications requiring surgical drainage. (See 'Surgery' below.)

Imaging modalities — CT scanning and magnetic resonance imaging (MRI) are useful for the
diagnosis of orbital cellulitis and its complications. There are no controlled trials comparing these
modalities and the choice is usually based on the availability of the test and on the clinical
experience of the physicians involved. CT scanning of the orbits and sinuses is used most
commonly to evaluate for possible orbital cellulitis and its complications. MRI is superior to CT
scan in following the progression of soft tissue disease [63], but may not be readily available and
may require sedation in young children.

When cavernous sinus thrombosis is suspected, an imaging study that includes venography
should be performed, such as magnetic resonance (MR) venography or CT venography. These
modalities are discussed in detail separately. MR venography is used most commonly, and would
show nonfilling of the cavernous sinus in patients with cavernous sinus thrombosis. (See "Cerebral
venous thrombosis: Etiology, clinical features, and diagnosis", section on 'Diagnosis'.)

Imaging findings — In preseptal cellulitis, inflammation is confined to the eyelids. In orbital


cellulitis, inflammation of extraocular muscles, fat stranding, and anterior displacement of the
globe are common findings. It is important to note that some cases of orbital cellulitis will have
subtle abnormalities by CT scan. There is almost always evidence of rhinosinusitis, most
commonly ethmoid sinusitis, and the most intense inflammatory response in the orbit is often seen
adjacent to the ethmoid sinus.

Subperiosteal abscesses and orbital abscesses appear as low-density collections on CT scan.


Subperiosteal abscesses lie between the periosteum and the wall of the bony sinus (image 1).
Both types of abscesses, but especially subperiosteal abscesses, are often found adjacent to the
ethmoid sinus (figure 2).

Indications for imaging — Whether every patient with suspected orbital cellulitis should have a
CT scan is controversial [19,31,64,65]. Clinicians may be reluctant to obtain a CT scan due to
concern about exposure of the pediatric patient to radiation. Guidelines for the management of
orbital cellulitis from the United Kingdom offer the following indications for CT scanning: inability to
assess vision; proptosis, ophthalmoplegia, bilateral edema, or deteriorating visual acuity; lack of
improvement after 24 hours of administration of intravenous antibiotics; "swinging" fevers not
resolving within 36 hours; or signs or symptoms of central nervous system (CNS) involvement
[64].

In the study of CT scanning in children with suspected acute preseptal or orbital cellulitis described
above, the presence of edema beyond the eyelid margin or an absolute neutrophil count (ANC)
>10,000 cell/microL were independent risk factors for an orbital abscess; the authors concluded
that the presence of either of these findings should necessitate an expedited evaluation, including
emergent CT scanning [39]. (See 'Orbital abscess' above.)

In addition, CT scanning should be done in all patients in whom surgical drainage is being
considered [5].

Based on the studies and guidelines described above, we recommend that patients with
suspected orbital cellulitis with any of the following features undergo a contrast-enhanced CT scan
of the orbits and sinuses:

●Proptosis
●Limitationof eye movements
●Pain with eye movements
●Double vision
●Decreased vision
●Relative afferent pupillary defect

Only for Study purpose / Only Notes / AMC MCQ


Only Study Notes / Only for Study purposes / Personal notes/ AMC MCQ

●Edema extending beyond the eyelid margin


●ANC >10,000 cell/microL
●Signs or symptoms of CNS involvement
●Inability to examine the patient fully (usually patients less than one year of age)
●Patients who do not begin to show improvement within 24 to 48 hours of initiating
appropriate therapy

Microbiologic studies — Despite the low yield, we recommend obtaining blood cultures in all
patients with suspected orbital cellulitis before the administration of antibiotics. In patients with
concomitant acute bacterial sinusitis who are examined by an otolaryngologist, any purulent sinus
drainage seen on endoscopic examination should be cultured. If surgery is performed, the material
obtained should be examined by Gram stain and, in patients with risk factors for a fungal and/or
mycobacterial etiology, by special stains for fungi and mycobacteria. Cultures should be done for
ordinary bacterial pathogens and, depending on the circumstances, fungi and mycobacteria.
If orbital cellulitis is suspected, collect blood samples for culture. A CT scan of the orbits and sinuses should be
performed in all cases of suspected orbital cellulitis, because a delay in diagnosis can result in visual
compromise or spread to the cranial fossa. If there is intracranial extension, consider brain abscess, which
requires empirical antibiotic therapy with activity against anaerobes

— Urgent surgical drainage of the sinuses or of an orbital, subperiosteal or intracranial abscess may be required
to prevent loss of vision.

Admit the patient to hospital and undertake 4-hourly eye observations (visual acuities and pupil reactions).
Patients with orbital cellulitis are rarely suitable for community-based parenteral antimicrobial therapy.

Use intravenous therapy initially, then switch to oral therapy when the infection is improving (see Guidance for
antimicrobial intravenous to oral switch). Modify therapy based on the results of culture and susceptibility testing,
if possible.

The total duration of therapy (intravenous + oral) is 10 to 14 days.

Only for Study purpose / Only Notes / AMC MCQ


Only Study Notes / Only for Study purposes / Personal notes/ AMC MCQ

56- 25 y old pt came pain in the eye become red during pain time also painful urination for 2wks
1. Trachoma – this is recurrent presentation
2. Chlamydia
3. Gonorrhoea – 2 week no as this is hyperacute presentation
4. Herpes – 2 weeks + urethritis in this case goes against
Eye Infection: Chlamydial Conjunctivitis. Conjunctivitis is redness and swelling of the clear membrane that
lines the inside of your eyelids and covers the white of your eye. This membrane is called the conjunctiva.
Chlamydia is a type of bacteria that can cause infections

Adult inclusion conjunctivitis — Adult inclusion conjunctivitis is not strictly an acute


conjunctivitis but rather a chronic, indolent conjunctivitis. It is a sexually transmitted infection
caused by certain serotypes of Chlamydia trachomatis. Concurrent asymptomatic urogenital
infection is typically present.

Only for Study purpose / Only Notes / AMC MCQ


Only Study Notes / Only for Study purposes / Personal notes/ AMC MCQ

The eye infection presents as a unilateral, or


sometimes bilateral, follicular conjunctivitis of weeks
to months duration that has not responded to topical
antibiotic therapy. There can be an associated
keratitis.

Conjunctivitis — Gonococcal conjunctivitis mainly


affects infants born to untreated mothers [19]
(see "Gonococcal infection in the newborn"). In adults
and adolescents, sporadic cases can occur as a result of
autoinoculation from an anogenital source (picture 4).
Outbreaks of gonococcal conjunctivitis have also been
described [47,48]. In such outbreak settings,
transmission via person-to-person nonsexual contact,
fomites, or vectors (flies) has been reported.
Gonococcal conjunctivitis ranges from mild pauci-
symptomatic infections to aggressive infections
characterized by conjunctival injection, purulent
discharge, and periorbital edema, which, if untreated,
can progress to corneal ulceration, perforation, and
blindness [49]. Infectivity ceases within hours of
appropriate antibiotic therapy; as such, isolation of
infected persons may be discontinued 24 hours after
treatment
The eye infection is characterized by a profuse purulent discharge present within 12 hours of inoculation
[9]; the amount of discharge is striking. Other symptoms are rapidly progressive and include redness,
irritation, and tenderness to palpation. There is typically marked chemosis, lid swelling, and tender
preauricular adenopathy. Gram-negative diplococci can be
identified on Gram stain of the discharge.

Hyperacute bacterial conjunctivitis — Neisseria


species, particularly N. gonorrhoeae, can cause a
hyperacute bacterial conjunctivitis that is severe and
sight-threatening, requiring immediate ophthalmologic
referral [8]. The organism is usually transmitted from
the genitalia to the hands and then to the eyes.
Concurrent urethritis is typically present.

The eye infection is characterized by a profuse


purulent discharge present within 12 hours of
inoculation [9]; the amount of discharge is striking.
Other symptoms are rapidly progressive and include
redness, irritation, and tenderness to palpation. There
is typically marked chemosis, lid swelling, and tender
preauricular adenopathy. Gram-negative diplococci
can be identified on Gram stain of the discharge.

Only for Study purpose / Only Notes / AMC MCQ


Only Study Notes / Only for Study purposes / Personal notes/ AMC MCQ

These patients require hospitalization for systemic


and topical therapy and for monitoring of the ocular
component. Keratitis and perforation can occur.

57- CRVO-stormy sunset

Mild CRVO may show no symptoms.


However: Many patients with CRVO have
symptoms such as blurry or distorted vision
due to swelling of the center part of the
retina, known as the macula. Some patients
have mild symptoms that wax and wane,
called transient visual obscurations. Patients
with severe CRVO and secondary
complications such as glaucoma (a disease
characterized by increased pressure in the eye) often have pain, redness, irritation and other problems.

Risk factors — The following are risk factors for both BRVO and CRVO [2,5-8]:

●Age
●Hypertension
●Diabetes
●Smoking
●Obesity
●Hypercoagulable state, particularly factor V Leiden and activated protein C resistance

Only for Study purpose / Only Notes / AMC MCQ


Only Study Notes / Only for Study purposes / Personal notes/ AMC MCQ

●Glaucoma, which prevents retinal vein outflow and leads to stasis


●Retinal arteriolar abnormalities

Among patients with hypertension or diabetes, risk of BRVO is increased in the presence of end-
organ damage

The evaluation of retinal vein occlusion (RVO), like other retinal diseases, begins with the history
and a complete ophthalmologic examination. Other testing such as fluorescein angiography and
coherence tomography may be performed to confirm the diagnosis, assess retinal capillary
nonperfusion, and follow disease progression or response to treatment.

Eye examination — The eye examination includes evaluation of extraocular motility, intraocular
pressure, pupillary function, and the external and internal structures of the eye. This also includes
confrontational visual fields, slit-lamp examination, testing for visual acuity, and a dilated
fundoscopic examination.

Findings in patients with BRVO are largely normal, except for the fundoscopic examination and
testing of visual acuity.

In patients with CRVO, pupillary examination can be normal but typically demonstrates ipsilateral
relative afferent pupillary defect in cases of nonperfused CRVO. Confrontational visual field
deficits are variable. Anterior slit lamp examination is usually normal in the acute phase. The
fundoscopic examination shows typical findings. Intraocular pressure is unaffected in the acute
phase, prior to development of neovascularization of the angle (NVA).

Visual acuity — Visual acuity should be documented at each visit to assess RVO progression
and/or response to treatment. Assessment of visual acuity is discussed in detail elsewhere.

Newly diagnosed patients with ischemic CRVO can be categorized by their degree of visual loss
and findings on examination:

●Severe visual loss – In patients with severe visual loss (eg, 20/400 visual acuity or less),
relative afferent pupillary defect (eg, greater than 1 log unit as tested with neutral density
filters), diffuse retinal hemorrhage (often representing indeterminate perfusion status), or
retinal capillary nonperfusion (eg, greater than 10 disk areas by fluorescein angiography),
monthly examinations for six to eight months after initial diagnosis is advised in order to detect
development of retinal or anterior segment neovascularization.
Non-severe visual loss – In patients without severe visual loss (eg, better than 20/400 visual
acuity) or any of the other above characteristics, a follow-up examination in one to three
months with subsequent lengthening of intervals is advised until stability can be established at
least 6 to 12 months.

In patients with BRVO or CRVO without macular edema or neovascularization, we suggest


observation rather than initiation of treatment.

Treatment is indicated in patients with RVO for macular edema, retinal neovascularization, and
anterior segment neovascularization.

Only for Study purpose / Only Notes / AMC MCQ


Only Study Notes / Only for Study purposes / Personal notes/ AMC MCQ

58- Glaucoma

Clinical findings of POAG include an open


iridocorneal angle, characteristic changes of the optic
disc, and – if sufficient optic nerve damage has
occurred - corresponding visual field loss.

Only for Study purpose / Only Notes / AMC MCQ


Only Study Notes / Only for Study purposes / Personal notes/ AMC MCQ

The causes for optic atrophy include: Compressive – secondary to papilledema, tumor, bony growth
(fibrous dysplasia, osteopetrosis), thyroid eye disease, chiasmal (pituitary etc), optic nerve sheath
meningioma, disc drusen, increased intraocular pressure (glaucoma)

Anything that can compromise ganglion cell function can cause (over time) optic atrophy (and more broadly
optic neuropathy). Optic atrophy can occur due to damage within the eye (glaucoma, optic neuritis,
papilledema, etc.), along the path of the optic nerve to the brain (tumor, neurodegenerative disorder,
trauma, etc.), or it can be congenital (Leber’s hereditary optic atrophy, autosomal dominant optic atrophy

59 – Macular Degeneration

Only for Study purpose / Only Notes / AMC MCQ


Only Study Notes / Only for Study purposes / Personal notes/ AMC MCQ

60- Macular Edema

61- Diabetic macular edema

Only for Study purpose / Only Notes / AMC MCQ


Only Study Notes / Only for Study purposes / Personal notes/ AMC MCQ

Macular edema occurs when there is abnormal leakage and accumulation of fluid in the macula from
damaged blood vessels in the nearby retina. ... Macular edema can also occur after eye surgery, in
association with age-related macular degeneration, or as a consequence of inflammatory diseases that
affect the eye.so Macular edema is sign of different disease but macular degeneration is one disease by it
self

62- Iritis: bound down pupil

Only for Study purpose / Only Notes / AMC MCQ


Only Study Notes / Only for Study purposes / Personal notes/ AMC MCQ

63- Retinal detachment

RETINAL DETACHMENT: 􏰄 Sudden loss of


vision 􏰄 Floaters 􏰄 Flashes 􏰄 Field lost 􏰄
Fall in acuity 􏰄 Dx: Ophthalmoscopy 􏰄 Mx: Transfer to hosp with tilted head 􏰄 Fundoscopy􏰄 Hazy
appearance

64- CRAO – Cheery Spot

CRAO 􏰄 >55 Years of age Associated with:


Vasculopathy DM Hyperlipidaemia HTN 􏰄
Sudden loss of vision 􏰄 Loss of light perception
􏰄 Rx: to reduce intraocular pressure •
Fundoscopy: 􏰄 Waxy pale retina 􏰄 Cherry red
spot 􏰄 Thin arteries• Unilateral • Sudden,
painless loss of vision • Irreversible

Only for Study purpose / Only Notes / AMC MCQ


Only Study Notes / Only for Study purposes / Personal notes/ AMC MCQ

65- SCHAEMIC OPTIC NEUROPATHY


SCHAEMIC OPTIC NEUROPATHY 􏰄 Associated with
temporal arteritis 􏰄 On & off blindness 􏰄 Irreversible
blindness 􏰄 Diplopia + 􏰄 Inv: elevated ESR, CRP. Biopsy 􏰄
Rx : steroids for 18-24 months FUNDOSCOPY: 􏰄 Pale, 􏰄
Swollen disc 􏰄 Flame shaped haemorrhages

66- Vitreous Hemorrhage

Only for Study purpose / Only Notes / AMC MCQ


Only Study Notes / Only for Study purposes / Personal notes/ AMC MCQ

67- DIABETIC RETINOPATHY


DIABETIC RETINOPATHY 􏰄 Occurs
as a complication of DM 2. 􏰄 As a
consequence of microvascular
disease of retina. 􏰄 Most common
cause of blindness in young people.
Types: 􏰄 proliferative non
proliferative Non-proliferative:
Related to ischaemia of blood
vessels and include: 1.
Haemorrhages: Dot and blot. 2.
Microaneurysms. 3. Exudates: Soft
and hard.

68-HYPERTENSIVE RETINOPATHY

Only for Study purpose / Only Notes / AMC MCQ


Only Study Notes / Only for Study purposes / Personal notes/ AMC MCQ

69- MACULAR DEGENERATION


MACULAR DEGENERATION 􏰄 Age
related macular degeneration􏰄 􏰄 wet =
neovacular / exudative 􏰄 Dry = non
exudative / drusen 􏰄 Central visual loss
􏰄 Distorted vision (line of words bend
while reading) 􏰄 Commonly affects
elderly 􏰄 Leads to blindness-
progressive 􏰄 Screening: Amslar grid
(can’t see central dot) 􏰄 Best Inv:
Fluorescein angiography Fundoscopy: 􏰄
Haemorrhage 􏰄 Fluid leak in optic disc
TREATMENT: – Laser
photocoagulation – Vascular endothelial
growth factor injection

70- 40 y/o corneal injection, hazy,


oedema, eye pressure raised what is
the long-term management
1. Iridotomy
2. Acetazolamide
3. Trabeculectomy

• For angle closure can do both laser iridotomy -pain


• For open angle-laser trabeculoplasty and trabeculectomy-painless

71- Pic of pupil asking mix cataract


1- Laser phaco
2- Lens change

• Treatment is IOL ... if method asked then phacoemulsification


• Intraocular lenses (IOLs) are medical devices that are implanted inside the eye to replace the eye's
natural lens when it is removed during cataract surgery.
• Phacoemulsification cataract surgery is a procedure in which an ultrasonic device is used to break up
and then remove a cloudy lens, or cataract, from the eye to improve vision. The insertion of an
intraocular lens (IOL) usually immediately follows phacoemulsification.

Only for Study purpose / Only Notes / AMC MCQ


Only Study Notes / Only for Study purposes / Personal notes/ AMC MCQ

https://www.surgeryencyclopedia.com/Pa-
St/Phacoemulsification-for-

Cataracts.html?fbclid=IwAR2nc5_UHgphm17ck-fwWMj_UYXS6vN3dcU19GsHRMM-
bwJGxVAu3Xd1l_s#ixzz6J1WOYZJK

Only for Study purpose / Only Notes / AMC MCQ


Only Study Notes / Only for Study purposes / Personal notes/ AMC MCQ

72- 78 yrs vision loss over 5 months,


corrected by pinhole, cause
1- Age related macular degeneration
2- cataract
73- old patient with decrease in vision from
past few months. vision 6/18 on right side
and 6/12 on left and on correction with
pinhole is 6/9 on both sides .What is the
cause for this condition?
A.Retinal degeneration
B.Glaucoma
C.Cataract
D. Refractive error
E.presbyopia: Stars earlier around 40

Only for Study purpose / Only Notes / AMC MCQ


Only Study Notes / Only for Study purposes / Personal notes/ AMC MCQ

Only for Study purpose / Only Notes / AMC MCQ


Only Study Notes / Only for Study purposes / Personal notes/ AMC MCQ

74-

Only for Study purpose / Only Notes / AMC MCQ


Only Study Notes / Only for Study purposes / Personal notes/ AMC MCQ

75-

Only for Study purpose / Only Notes / AMC MCQ


Only Study Notes / Only for Study purposes / Personal notes/ AMC MCQ

76- 50 yr Old patient with decrease in vision from past few months, vision 6/18 on right side and 6/12
on left and on correction with pinhole is 6/9 on both sides. red reflex present. What is the most likely
cause from this condition?
A- Retinal degeneration
B- Glaucoma
C- Cataract
D- Hypermetropia

• Cataract, Retinoblastoma... red reflex absent


• pinhole worsen vision: Macular diseases,[4] central lens opacities[5]
• Vision static with pinhole: Amblyopia
• In presbyopia, pinhole glasses improve visual acuity and accommodation-related parameters by pseudo-
accommodation and the pinhole effect.

A pinhole occluder is an opaque disk with one or more small holes through it, used by ophthalmologists,
orthopedists and optometrists to test visual acuity.
The occluder is a simple way to focus light, as in a pinhole camera, temporarily removing the effects
of refractive errors such as myopia. Because light passes only through the center of the eye's lens, defects
in the shape of the lens (errors of refraction) have no effect while the occluder is used. In this way, the
ophthalmologist, orthoptist or optometrist can estimate the maximum improvement in a patient's vision that
can be attained by lenses to correct errors of refraction.
This can be used to distinguish visual defects caused by refractive error, which improve when the occluder
is used, from other problems, which do not.[2] The pinhole occluder can also be used in testing visual
acuity in mydriatic patients. In this case the pinhole occluder compensates for the inability to contract the
iris assisting the eye in obtaining a retinal projection similar to that of a non-cycloplegic eye.
• In sudden loss of vision caused by retinal vessel occlusion, there may be profound loss of sight, which is
unusual in most other causes. This will not improve through a pinhole occluder. Often poor vision caused by
cataract will improve when repeated through a pinhole, in contrast to AMD, where pinhole acuity is usually
worse.
• If you can get a retinal view through an undilated pupil, you can exclude cataract as a cause of poor vision.
In symptomatic glaucoma, the optic disc will be obviously cupped. In wet AMD, there may be a macular
haemorrhage; in vein occlusion, multiple peripheral haemorrhages with a swollen disc
• In unilateral cortical infarct, confrontation fields done with both eyes open will show a vertical cut-off as the
target is moved from the unaffected to affected side. Field defects in retinal detachment and glaucoma are
less reliably demonstrated by this technique.

Only for Study purpose / Only Notes / AMC MCQ


Only Study Notes / Only for Study purposes / Personal notes/ AMC MCQ

Only for Study purpose / Only Notes / AMC MCQ


Only Study Notes / Only for Study purposes / Personal notes/ AMC MCQ

Diplopia Nice chart to have a look

Only for Study purpose / Only Notes / AMC MCQ


Only Study Notes / Only for Study purposes / Personal notes/ AMC MCQ

77- orbital cellulitis scenario, inv? → CT

Only for Study purpose / Only Notes / AMC MCQ


Only Study Notes / Only for Study purposes / Personal notes/ AMC MCQ

Check this in case of PERIORBITAL


CELLULITIS

78- young man HIT by a squash ball while playing, apart from headache which resolved by analgesic he
complained of decreased in vision. On examination,
his visual acuity was 6/12 in the left eye; right eye
was 6/6 asking the most likely cause?
A-vitreous haemorrhage
B-retinal detachment - in retinal detachment there is
a painless partial or total loss of vision and needs
immediate referral the person is asked to keep his
head up and true detachments in surgery
C-zygomatic fracture
D-orbital floor fractur - But here Decreased visual
acuity.

Only for Study purpose / Only Notes / AMC MCQ


Only Study Notes / Only for Study purposes / Personal notes/ AMC MCQ

Vitreous hemorrhage — Bleeding into the vitreous humor can occur in the setting of trauma, spontaneous
retinal tear, spontaneous vitreous detachment, or any condition with retinal neovascularization such as
poorly controlled diabetes mellitus. The reduction in vision is directly proportional to the amount of blood in
the vitreous. If the hemorrhage is dense enough, there may be a decreased red reflex (the reddish orange
reflection off the subretinal layers when examining the eye with an ophthalmoscope), or the retina may not
be visible with ophthalmoscopy.
Retinal detachment — Detachment of the neurosensory retina may occur spontaneously or in the setting
of trauma. The most common form is related to a tear or break in the retina in patients with a preexisting
area of retinal thinning (usually highly myopic, or nearsighted, individuals). Patients may describe sudden
onset of new floaters or black dots in their vision, often accompanied by flashes of light (photopsias). In its
early stages,

Orbital floor fracture — Fractures of the floor of the orbit, sometimes known as "blowout
fractures," typically occur when a small, round object (eg, a baseball) strikes the eye. In one study,
half of all orbital fractures involved the inferior wall or floor of the orbit [4]. Evidence suggests that
orbital floor fractures may be caused by one or both of the following mechanisms [12]:

●Increased intraocular pressure as the result of posterior displacement of the globe (hydraulic
theory)
●A direct blow to the infraorbital rim

In children, the floor of the orbit is more flexible. Consequently, it may fracture in a linear
pattern, creating a bone fragment that snaps back to create a "trap-door" fracture. In adults,
the floor of the orbit is thinner and more likely to shatter when exposed to force

Orbital fractures — Common signs and symptoms


of orbital fracture include periocular ecchymosis, pain
on lateral or vertical gaze, diplopia due to ocular
muscle entrapment (picture 24) disconjugate gaze on
looking vertically or horizontally, decreased
sensation in the distribution of the infraorbital nerve,
fracture site tenderness or bony stepoff, and/or
crepitus indicating orbital emphysema caused by
extension of an orbital fracture into the sinus. Thin
cut coronal CT of the orbit is the imaging modality of
choice to diagnose orbital fractures.
Orbital zygomatic fracture — The orbital zygomatic
region is the most common location of a fracture of
the orbital rim. This injury is typically the result of a
high-impact blow to the lateral orbit. There is often an
associated fracture of the orbital floor.

SUMMARY Approach to the adult with acute


persistent visual loss

●Acute persistent visual loss, defined as a sudden deficit in visual function in one or both eyes lasting more
than 24 hours, is distinguished from acute transient visual loss (amaurosis fugax). Whereas amaurosis
fugax is caused by only a few conditions (temporary vascular occlusion or temporary neuronal depression
related to seizure or migraine), the differential diagnosis of acute persistent visual loss is large. (See
'Definition of terms' above.)

●Conditions that cause acute persistent visual loss can be divided into three categories: media problems,
retinal problems, or neural visual pathway problems (table 1). Media problems include keratitis, corneal

Only for Study purpose / Only Notes / AMC MCQ


Only Study Notes / Only for Study purposes / Personal notes/ AMC MCQ

edema, hyphema, lens changes, vitreous hemorrhage, and uveitis. Retinal problems include vascular
occlusion, retinal detachment, and acute maculopathy. Neural visual pathway malfunction may be
subdivided into optic nerve disease, chiasmal, and retrochiasmal visual pathway pathology. (See 'Etiology'
above.)

●Health care practitioners in the urgent setting most often do not have the equipment or expertise to
perform a comprehensive ophthalmic examination. However, a thorough history and eye examination can
help determine the general category, if not the specific cause, of vision loss (table 2 and algorithm 1). (See
'History' above and 'Physical examination' above.)

●Appropriate management of acute persistent visual loss depends on understanding the urgency for
referral to an ophthalmologist and initiation of treatment. (See 'Patient
triage' above.)

●The treatment of acute persistent visual loss depends on the specific


diagnosis. The following causes require immediate treatment: central
retinal artery occlusion (CRAO), giant cell arteritis (GCA), and acutely
elevated intraocular pressure (IOP). (See 'Immediate treatment' above.)

Pseudoptosis: enophthalmos secondary to an orbital blow-out fracture (right eye).

Only for Study purpose / Only Notes / AMC MCQ


Only Study Notes / Only for Study purposes / Personal notes/ AMC MCQ

Only for Study purpose / Only Notes / AMC MCQ


Only Study Notes / Only for Study purposes / Personal notes/ AMC MCQ

Only for Study purpose / Only Notes / AMC MCQ


Only Study Notes / Only for Study purposes / Personal notes/ AMC MCQ

79- young adult hit his eye with squash ball apart
from headache which resolved with paracetamol. in
second day developed slightly decreased in visual
acuity in the affected eye. anterior chamber is clear
all other examination is normal what is dx.
A. vitreous haemorrhage (can be seen in RD)
B. Retinal detachment
C. retinal artery thrombosis – cherry spot in
fundoscopy
D. hyphema – ant chamber with blood
E. cerebral haemorrhage

Only for Study purpose / Only Notes / AMC MCQ


Only Study Notes / Only for Study purposes / Personal notes/ AMC MCQ

80- Marcus Gunn Pupil

Relative afferent pupillary defect (RAPD) is a


medical sign observed during the swinging-
flashlight test whereupon the patient's pupils
dilate when a bright light is swung from the
unaffected eye to the affected eye
The most common cause of Marcus Gunn pupil
is a lesion of the optic nerve (between the retina
and the optic chiasm) due to glaucoma, or
severe retinal disease, or due to multiple
sclerosis

Normal swinging light test Left relative afferent


pupillary defect* (RAPD)
1. Begin with dark room, bright pen light and
patient •fixated at distant object (to avoid a near
pupil response).
2. Shine light into right (R) eye. Both pupils
should constrict.
3. Swing light to left (L) eye. Both pupils remain
constricted.
4. Swing light back to right eye. Both pupils remain constricted
https://www.youtube.com/watch?v=WlGu6CuFJrw&feature=youtu.be&fbclid=IwAR08SOH9j37DIqIFjXbmytk
S4rdRUlxjdZn9cU0BlneCX3Ybn_Qi6PnqfkI

Only for Study purpose / Only Notes / AMC MCQ


Only Study Notes / Only for Study purposes / Personal notes/ AMC MCQ

What are possible causes of Marcus Gunn Pupil?


• Optic Nerve Disorders: optic neuritis, optic nerve infections or inflammations, glaucoma, optic nerve
tumor, optic neuropathy, orbital disease
• Retinal Causes: ischemic retinal disease, retinal detachment, sever macular degeneration,
intraocular tumor, retinal infection
• Cerebral vascular disease
• Amblyopia

Only for Study purpose / Only Notes / AMC MCQ


Only Study Notes / Only for Study purposes / Personal notes/ AMC MCQ

81 - Melanoma eye picture. Mx?


1- Excision
2- Plastic surgeon reference – we need to refer to someone

Conjunctival melanoma
●For patients with conjunctival melanoma, initial management focuses upon wide local
surgical excision, supplemented by cryotherapy and possibly alcohol application.

Overview
Malignant melanoma of the conjunctiva presents as a raised
pigmented or nonpigmented lesion. [1]
This lesion is uncommon but potentially lethal. It can arise
in previously unblemished and unpigmented regions
(approximately 10% of cases), from a preexisting nevus
(approximately 20% of cases), or from the flat, spreading
pigmentation of primary acquired melanosis with atypia
(60-70% of cases). Examples of the disease are seen
below. Clinical presentation — Conjunctival melanomas arise from melanocytes located in the basal
layer of the conjunctival epithelium, and they represent approximately 5 percent of all ocular melanomas

Conjunctival melanomas may be associated with primary acquired melanosis (75%) or


may arise from a preexisting nevus or de novo.

Only for Study purpose / Only Notes / AMC MCQ


Only Study Notes / Only for Study purposes / Personal notes/ AMC MCQ

In addition to spreading by lymphatics and the bloodstream, conjunctival melanoma can


undergo direct extension to the eyeball and orbit

Uveal melanoma
●For asymptomatic patients with small uveal melanocytic
tumors (<12 mm in diameter and <2 to 3 mm in thickness),
an initial period of observation for evidence of growth is
generally recommended until evidence of growth is
documented. (See 'Natural history of uveal melanocytic
tumors' above.)
●For patients with larger tumors and for those with
symptoms, initial treatment is usually indicated:
•For symptomatic patients and those with medium or large
tumors, treatment with radiation therapy (RT) is generally
recommended. RT can be administered using plaque
brachytherapy, the most widely available form of RT, or with charged-particle RT.
(See 'Radiation therapy' above and 'Enucleation' above.)
•Enucleation is generally reserved for patients in whom RT is unlikely to achieve
adequate local tumor control or is likely to result in unacceptable ocular radiation
complications due to large tumor size, extrascleral extension, or risk of neovascular
glaucoma.

Only for Study purpose / Only Notes / AMC MCQ


Only Study Notes / Only for Study purposes / Personal notes/ AMC MCQ

Conjunctival Melanoma:
Surgical Care In an adult, all elevated or enlarged
pigmented lesions with a history of change should be
excised as suspected malignant melanoma.
Note that metastatic melanomas from anywhere in
the body and extensions from ciliary body
melanomas may first become apparent in the
conjunctiva.
The treatment of conjunctival melanoma is surgical,
with complete removal of the tumor, if possible.
Cryotherapy to the normal-appearing conjunctiva
surrounding the lesion is usually performed. Damato
and Coupland reported that high rates of local tumor
control, with little ocular morbidity, resulted from
excision of invasive melanoma with adjunctive
brachytherapy and topical chemotherapy. [25] This
audit also noted that when no caruncular
involvement had occurred, disease-specific mortality
was rare, except in patients who were referred after
a surgical procedure. The investigators’ results
suggested that risks of local recurrence and
metastatic death are increased by inadequate
surgical intervention. The suggestions of Shields in
the surgical management of circumscribed
conjunctival melanomas are advocated. The "no
touch" technique is essential throughout the
procedure. No surgical instrument is used more than
once in any area (addressing the concern of
microscopically seeding tumor cells). Treatment of
primary conjunctival melanomas in the limbal region
of the bulbar conjunctiva can usually be
accomplished with initial, localized absolute alcohol
epitheliectomy. This treatment is followed by wide (2-
to 3-mm clear zone), local excision by a partial
lamellar scleroconjunctivectomy. The bed of
excision, as well as the adjacent conjunctiva or
cornea away from the nodule, is treated with
supplemental double freeze-thaw cryotherapy, using
a specific technique (lifting the conjunctiva). Treat the
entire area of the lesion, because untreated areas
may lead to spread through local lymphatic channels.
Laser therapy after excision has also been used. Nodal involvement indicates extensive metastatic
disease, but occasionally, cases in which lesions were limited to regional nodes and cured by node
resection have occurred. If the tumors are located in the fornical or palpebral conjunctiva, wide surgical
resection with alcohol treatment to the scleral base and cryotherapy to the surrounding conjunctiva is
performed. Exenteration of the orbit Exenteration of the orbit sometimes is necessary for large melanomas
that have invaded the orbit, but this procedure does not improve the prognosis. It may be performed for
patients in whom the objective is to do local debulking of a tumor, because this procedure is not linked to
increased patient survival. The use of radical neck dissection at the period of exenteration is not without
controversy. [26] The poor survival rate, despite orbital exenteration, suggests that metastasis has already

Only for Study purpose / Only Notes / AMC MCQ


Only Study Notes / Only for Study purposes / Personal notes/ AMC MCQ

occurred at the time of treatment and confirms that the extent of the disease at diagnosis is the most
important factor in determining the outcome. Considerable orbital invasion indicates the necessity for
exenteration; however, subtotal exenteration can be carried out if no evidence of radial extension of the
lesion to the skin of the anterior lid exists.

82 - Blowout fracture of orbit what would you


see
A. Hyphaema
B. Horizontntal diplopia – vertical diplopia
C. Enopthalmos
D. Depressed Zygomatic arch
E. Pain in mastication-mandible

Orbital floor fracture — Fractures of the floor


of the orbit, sometimes known as "blowout
fractures," typically occur when a small, round
object (eg, a baseball) strikes the eye. In one study, half of all orbital fractures involved the inferior wall or
floor of the orbit [4]. Evidence suggests that orbital floor fractures may be caused by one or both of the
following mechanisms [12]:
●Increased intraocular pressure as the result of posterior displacement of the globe (hydraulic theory)
●A direct blow to the infraorbital rim
In children, the floor of the orbit is more flexible. Consequently, it may fracture in a linear pattern, creating a
bone fragment that snaps back to create a "trap-door" fracture. In adults, the floor of the orbit is thinner and
more likely to shatter when exposed to force [13].
A significant consequence of fractures of the orbital floor is entrapment of the inferior rectus muscle and/or
orbital fat (image 1). Ischemia and subsequent loss of muscle function may occur either because of
entrapment of muscle within the fracture fragment (more likely in children) or as the result of edema and
hemorrhage of muscle and extraocular fat that have prolapsed through the fracture into the maxillary sinus
(more likely in adults) [3,13-15].
Enophthalmos (the eye is receded into the orbit (picture 1)) may develop when the globe is displaced
posteriorly in association with an orbital floor fracture and prolapse of tissue into the maxillary sinus. Orbital
dystopia (the eye on the affected side is lower in the horizontal plane than the other) may occur because
entrapped muscle and orbital fat pull the eye downward.
Injury to the infraorbital nerve (resulting in decreased sensation along the cheek, upper lip, or upper
gingiva) may occur as the result of an orbital floor fracture.

Only for Study purpose / Only Notes / AMC MCQ


Only Study Notes / Only for Study purposes / Personal notes/ AMC MCQ

Signs & Symptoms


Some clinically observed signs and
symptoms include:[10][11]
Orbital pain
Eyes displaced posteriorly into
sockets (enophthalmos)
Limitation of eye movement
Loss of sensation (hypoesthesia)
along the trigeminal (V2) nerve
distribution
Seeing-double when looking up or
down (vertical diplopia)
Orbital and lid subcutaneous
emphysema, especially when
blowing the nose or sneezing
Nausea and bradycardia due to oculocardiac reflex
Causes
Common medical causes of blowout fracture
may include:
Direct orbital blunt injury
Sports injury (squash ball,[12] tennis ball
etc.)
Motor vehicle accidents
Young man after a quarrel had a fracture of
floor of eye what is the more
consistent symptoms with that
Conjunctival hg
loss of visual activity
anaesthesia around the cheek
Cant open the mouth completely
• Epistaxis
#eye
better vertical diplopia, if absent anesthesia
cheek
Floor: vertical diplopia
Lat wall: horizontal diplopia
Orbital zygomatic fracture — The orbital zygomatic region is the most common location of a fracture of
the orbital rim. This injury is typically the result of a high-impact blow to the lateral orbit. There is often an
associated fracture of the orbital floor

Only for Study purpose / Only Notes / AMC MCQ


Only Study Notes / Only for Study purposes / Personal notes/ AMC MCQ

83- 30-year-old lady comes to you complaining of blurring of vision for last 3 days. On
examination of the eyes you found left eye is 6/12 and right eye is 6/6. There is
also pain behind the left eye on moving the eye. Which of the following is the
diagnosis?
a. Intracranial haemorrhage
b. Tumor of eyeball
c. optic neuritis - pain during movement

Most cases of acute demyelinating optic neuritis occur in women (two-thirds) and typically develop in
patients between the ages of 20 and 40
PATHOPHYSIOLOGY
The most common pathologic basis for optic neuritis is inflammatory demyelination of the optic nerve. The
pathology is similar to that of acute multiple sclerosis (MS) plaques in the brain, with perivascular cuffing,
edema in the myelinated nerve sheaths, and myelin breakdown. Inflammation of the retinal vascular
endothelium can precede demyelination and is sometimes visibly manifest as retinal vein sheathing [12].
Myelin loss exceeds axonal loss.

Only for Study purpose / Only Notes / AMC MCQ


Only Study Notes / Only for Study purposes / Personal notes/ AMC MCQ

Vision loss typically develops over a period of hours to days, peaking within one to two weeks. Continued
deterioration after that time suggests an alternative diagnosis [1,2]. Greater than 90 percent of patients in
the ONTT had a significant decrease in central visual acuity. In most, the visual acuities ranged from 20/25
to 20/190 (median visual acuity 20/60). However, some patients had 20/20 acuity (11 percent), and, at the
other extreme, a few had no light perception (3 percent).
●Eye pain occurred in 92 percent of patients in the ONTT and often worsened with eye movement [7]. The
onset of pain generally coincided with the visual acuity loss and improved along with it.

Two types of primary


tumors arise within the
eye itself and are known
as retinoblastoma in
children and melanoma in
adults.

84- A man aged 68 develops gradual vision loss over days bilaterally, what is the most likely
diagnosis?
a) macular degeneration
b) CRAO
c) CRVO
d) HTN
e) diabetic retinopathy
Age-related macular degeneration (AMD) is often
asymptomatic. Patients with dry AMD may
complain of gradual loss of vision in one or both
eyes.

Only for Study purpose / Only Notes / AMC MCQ


Only Study Notes / Only for Study purposes / Personal notes/ AMC MCQ

Distortion of straight lines (metamorphopsia) is one of the earliest changes with wet AMD. Patients may
perceive straight edges (such as doors or window blinds) as curved or distorted. Line distortion can be
evaluated by use of the Amsler grid, a tool for
detecting the occurrence of disease progression
in patients with dry AMD
The vast majority of patients who develop DR
have no symptoms until the very late stages (by
which time it may be too late for effective
treatment). Because the rate of progression may
be rapid and therapy can be beneficial for both
symptom amelioration and reduction in the rate
of

disease progression, it is important to screen patients with


diabetes regularly for the development of retinal disease

Only for Study purpose / Only Notes / AMC MCQ


Only Study Notes / Only for Study purposes / Personal notes/ AMC MCQ

85 - Scleritis Pic…Asking treatment. Same pic as below. Options-


1. Topical steroid (EPISCLERITIS)
2. Chloramphenicol
3. Topical Cipro
4. Topical Timolol
5. Acetazolamide
And: steroid or refer: If referral is in option, we need to choose referral - Systemic Steroids or NSAIDs

Episcleritis — topical steroids


therapy Scleritis— systemic steroids
Scleritis treatment
Nonsteroidal antiinflammatory drugs — NSAID therapy is primarily recommended for patients with the
diffuse and nodular subtypes of anterior scleritis (picture 1 and picture 2). There are few data on which to
base the choice of a specific NSAID for the treatment of scleritis. Investigators in Europe have
used flurbiprofen for this purpose [3]. The clinical impression of some investigators is
that indomethacin may be more effective than other available NSAIDs [1]. We suggest indomethacin (25 to
75 mg taken orally three times daily).
NSAID treatment should be continued for as long as there is evidence of scleral inflammation. Side effects
of NSAIDs and treatment failures may limit the duration of indomethacin use. (See "Nonselective NSAIDs:
Overview of adverse effects".)
Glucocorticoids — Patients with the diffuse and nodular subtypes of anterior scleritis who fail initial NSAID
therapy and those with necrotizing anterior scleritis or posterior scleritis (picture 3 and picture 4) should be
treated with glucocorticoids.
Based upon clinical experience, we suggest initial therapy with prednisone (1 mg/kg per day, up to a
maximum daily dose of 80 mg). This regimen is continued for the first four to six weeks of therapy with
ongoing assessment of the clinical response. The initial prednisone dose should not be continued

Only for Study purpose / Only Notes / AMC MCQ


Only Study Notes / Only for Study purposes / Personal notes/ AMC MCQ

beyond six weeks (and usually only four weeks) because of the potential for glucocorticoid-related
morbidity. (See "Major side effects of systemic glucocorticoids".)
If patients have not demonstrated signs of clinical improvement within four to six weeks, an additional
immunosuppressive agent must be considered.
https://www.uptodate.com/contents/treatment-of-
scleritis?search=scleritis&source=search_result&selectedTitle=2~74&usage_type=default&display_rank=2
#H3

Only for Study purpose / Only Notes / AMC MCQ


Only Study Notes / Only for Study purposes / Personal notes/ AMC MCQ

86- Baby with red eyes and swelling with yellow crust what is investigation- it is periorbital
cellulitis
A. Blood culture - If periorbital cellulitis IV ATB
B. LP

• If yellow crust maybe is blepharitis - as treatment is to


remove the crust and steroid drops. If Staph aureus –
Tetracycline and chloramphenicol ointment

Only for Study purpose / Only Notes / AMC MCQ


Only Study Notes / Only for Study purposes / Personal notes/ AMC MCQ

Blepharitis Clinical features


• Persistent sore eyes or eyelids
• Irritation, grittiness, burning, dryness and
‘something in the eye’ sensation
• Lid or conjunctival swelling and redness
• Crusts or scales around the base of the eyelids
• Discharge or stickiness, especially in morning
• Inflammation and crusting of the lid margins

Only for Study purpose / Only Notes / AMC MCQ


Only Study Notes / Only for Study purposes / Personal notes/ AMC MCQ

87-

Ans: E IV- ATB broad

Only for Study purpose / Only Notes / AMC MCQ


Only Study Notes / Only for Study purposes / Personal notes/ AMC MCQ

Only for Study purpose / Only Notes / AMC MCQ


Only Study Notes / Only for Study purposes / Personal notes/ AMC MCQ

Periorbital and orbital cellulitis

Key Points
1. Orbital cellulitis is an emergency with serious complications including intracranial infection,
cavernous sinus thrombosis and vision loss.
2. Urgent imaging and surgical consultation (ENT and ophthalmology) should be considered for
any child with suspected orbital cellulitis.
3. Periorbital cellulitis in a well child can often be treated with oral antibiotics if follow-up is
assured.

Background
• Periorbital and orbital cellulitis are distinct clinical diseases, though have overlapping clinical
features and therefore can be difficult to differentiate

Only for Study purpose / Only Notes / AMC MCQ


Only Study Notes / Only for Study purposes / Personal notes/ AMC MCQ

• Orbital cellulitis
• infection within the orbit, (i.e. postseptal, the structures posterior to the orbital
septum)
• surgical emergency with major complications including loss of vision, abscess
formation, venous sinus thrombosis and extension to intracranial infection with
subdural empyema, and meningitis.
• the majority (>80%) of cases relate to local sinus disease
• Periorbital cellulitis
• infection of the eye lids and surrounding skin not involving the orbit (i.e. preseptal,
the structures anterior to the orbital septum)
• The globe is not involved in either infection
Assessment
Red flag features in red
Typical presentation of periorbital/orbital cellulitis
• unilateral eyelid swelling and erythema

• unilateral eye pain or tenderness


Consider gonorrhoea and chlamydia infections in neonatal presentation (send PCR swabs)
Red flags concerning for orbital cellulitis
• painful or restricted eye movements

• visual impairment
• reduced acuity
• relative afferent pupil defect
• diplopia
• proptosis
• severe headache or other features of intracranial involvement

Differential diagnosis
Bilateral findings and/or painless (or non-tender) swelling in a well looking child is more likely to be an
allergic reaction.

Only for Study purpose / Only Notes / AMC MCQ


Only Study Notes / Only for Study purposes / Personal notes/ AMC MCQ

Management

Only for Study purpose / Only Notes / AMC MCQ


Only Study Notes / Only for Study purposes / Personal notes/ AMC MCQ

Antibiotic guidelines may vary depending on local resistance patterns


• Check local guidelines; these may include advice regarding community acquired MRSA

• If inadequate Haemophilus influenzae type B (Hib) vaccination, treat as severe periorbital


cellulitis
Orbital Cellulitis
• Admission

• Keep fasted until need for surgery clarified


• Seek ENT & Ophthalmology advice urgently
• Consider urgent contrast enhanced CT scan of orbits, sinuses +/- brain
• Investigations
• FBE and blood culture
• Lumbar Puncture (LP) is contraindicated due to risk of raised intracranial pressure
(ICP) secondary to possible intracranial extension
• Antibiotics (see below)
• Treat underlying sinus disease e.g. nasal decongestants, steroids (often guided by ENT)
Periorbital Cellulitis
Severe
Inpatient investigations and management as per orbital cellulitis
Moderate
Inpatient management or consider Hospital in the home (HITH) admission if available locally
• Consider blood culture if febrile and unwell

• Antibiotics (see below)


• Once improving change to oral antibiotics
• If not improving or deteriorating within 24-48 hours, consider managing as Severe Periorbital
Cellulitis
Mild
• Antibiotics (see below)
• Review

Only for Study purpose / Only Notes / AMC MCQ


Only Study Notes / Only for Study purposes / Personal notes/ AMC MCQ

Summary of antibiotic treatment*


Intravenous Therapy Oral Therapy Total
Duration

3rd generation cephalosporin 10-14 days


Orbital • IV Cefotaxime • Amoxicillin with
cellulitis 50mg/kg (max 2g) clavulanic acid
every 6 hours OR (doses based
on amoxicillin
• IV Ceftriaxone
component)
50mg/kg (max 2g)
Severe 22.5mg/kg
daily
Periorbital (max 875g),
cellulitis PLUS PO BD
• IV Flucloxacillin
50mg/kg (max 2g)
every 6 hours OR
• If suspected MRSA:
IV Vancomycin (see
link for dosing)

Duration based on clinical severity


and improvement. Usually at least
3-4 days, then switch to oral.

• IV Flucloxacillin When improving, switch to 7-10 days


Moderate 50mg/kg (max 2g), oral antibiotics as per mild
Periorbital every 6 hours periorbital cellulitis
cellulitis OR
• IV Ceftriaxone
50mg/kg (max 2g)
daily (consider
HITH)
OR
If suspected MRSA:
• IV/PO Clindamycin
15mg/kg (max
600mg) every 8
hours OR
• PO Trimethoprim
with
sulfamethoxazole
(8/40 mg/mL), 4/20
mg/kg BD (max
320/1600mg)

Duration based on clinical severity


and improvement. Usually 1-2
days, then switch to oral.

Only for Study purpose / Only Notes / AMC MCQ


Only Study Notes / Only for Study purposes / Personal notes/ AMC MCQ

Not applicable • Cefalexin 7-10 days


Mild Periorbital 25mg/kg (max
cellulitis 1g) PO TDS
OR
• Cefuroxime
3 months – 2
years: 10mg/kg
(max 125g) PO
BD
2 – 12
years: 15mg/kg
(max 250mg)
PO BD

*dosing sourced from AMH, for children aged over 1 month


Consider consultation with local paediatric team when
• suspected orbital cellulitis

• moderate-severe periorbital cellulitis


• no improvement or deterioration after 24-48hrs of therapy
Consider transfer when
• severe periorbital cellulitis or orbital cellulitis

• suspicion of intracranial involvement with altered conscious state, seizures or focal


neurological signs
• child requires care above the level of comfort of local hospital
For emergency advice and paediatric or neonatal ICU transfers, see Retrieval Services
Consider discharge when
Mild periorbital cellulitis
• oral antibiotics

• follow up assured
Moderate periorbital cellulitis
• after 24-48hrs of IV antibiotics and with improvement

• able to tolerate PO antibiotics


• follow up assured

https://www.rch.org.au/clinicalguide/guideline_index/Periorbital_and_orbital_cellulitis/?fbclid=IwAR0g9LqMJ
pJUOqMy1Y2pPcul1WVimfLeHRtK2u0OkXpIRDEEuEFYjlpc7zo

88- cellulitis, eyelid swelling, orbital redness, fever, he can’t move eyeball, there is marked
tenderness and erythema around the eye, asking investigation?
A. Ct scan - Red flag Orbital cellulitis !
B. swab test

Only for Study purpose / Only Notes / AMC MCQ


Only Study Notes / Only for Study purposes / Personal notes/ AMC MCQ

89- eyeball injury …two days later with decreased vision ..but no blood in anterior chamber ..cause?
A vitreous haemorrhage
B hyphema
C detached retina
D artery damage

https://www.ncbi.nlm.nih.gov/pmc/articles/PMC1705859/

90- yrs female, visual problem from last 1 year, left eye 6/12 & right eye ??pale
optic disc, no cupping, afferent pupillary reflex absent, DX?
a.DM
b.glucoma
If optic neuritis/ MS in option, we choose that on

Pale optic disc

• Whiteness of optic disc neuroretinal rim on ophthalmoscopy


• Sign of death of optic nerve axons
• Appears weeks to months after axons have died

Only for Study purpose / Only Notes / AMC MCQ


Only Study Notes / Only for Study purposes / Personal notes/ AMC MCQ

• Many diseases can cause this

Glaucoma is an important cause of optic atrophy. In glaucoma there is characteristic pathological cupping
of the optic disc which together with the typical visual field loss distinguishes glaucoma from other causes
of optic atrophy

Patients with optic nerve disease (e.g., optic neuritis, ischemic optic neuropathy, glaucomatous optic nerve
damage) have the most prominent relative afferent pupillary defects.
Severe retinal disease may cause a relative afferent pupillary defect, although the retinal disease must be
markedly asymmetric to produce the finding and, once the finding appears, it is subtle compared with that
seen in optic nerve disease.14

Only for Study purpose / Only Notes / AMC MCQ


Only Study Notes / Only for Study purposes / Personal notes/ AMC MCQ

Cataracts Do Not Cause the Relative Afferent Pupillary Defect15


Although this seems surprising, it is because the retina, if healthy, compensates over minutes for any
diminished brightness, just as it does after a person walks into a dark movie theater. In fact, during the time
of Galen, the classical Roman physician, clinicians tested the pupillary light reaction of patients with
cataracts to determine whether vision could be restored after couching (couching was an ancient treatment
for cataracts that used a needle to displace the cataract posteriorly; a preserved light reaction indicated that
the retina and optic nerve behind the cataract were intact).
https://www.sciencedirect.com/topics/medicine-and-dentistry/afferent-pupillary-defect

91-

Only for Study purpose / Only Notes / AMC MCQ


Only Study Notes / Only for Study purposes / Personal notes/ AMC MCQ

Only for Study purpose / Only Notes / AMC MCQ


Only Study Notes / Only for Study purposes / Personal notes/ AMC MCQ

Diabetic Retinopathy

Non-proliferative: Related to ischemia of blood vessels


and include:
1. Haemorrhages: Dot and blot.
2. Microaneurysms.
3. Exudates: Soft and hard.

Proliferative: Changes in blood vessels in respond to


ischemia to the retina.
1. New vessel formation.
2. Vitreous haemorrhage.
3. Retinal detachment.

Relative Afferent Pupillary Defect (RAPD)

Background
Relative Afferent Pupillary Defect (RAPD) is a condition in
which pupils respond differently to light stimuli shone in one
eye at a time due to unilateral or asymmetrical disease of
the retina or optic nerve. Swinging flashlight test or Marcus
Gunn test is one of the most basic eye exams that
neurologists, ophthalmologists, optometrists and other
physicians perform when visiting most of their patients. The
doctor will ask the patient to look ahead then shines a penlight first toward one eye, then swing to
the other, alternating quickly to observe patient’s pupils’ response to the light. In case if both pupils
do not show a similar response to the light stimuli, shone in one eye at a time, the patient will be
diagnosed with RAPD or Marcus Gunn pupils. off note if the condition is bilateral and symmetrical,
there will not be a RAPD but bilateral APD. The results of this simple yet very important test

Only for Study purpose / Only Notes / AMC MCQ


Only Study Notes / Only for Study purposes / Personal notes/ AMC MCQ

help doctors in early diagnosis of many important eye related diseases such as optic neuropathy and
multiple sclerosis .
[1][2]

Conditions Leading to a RAPD


• Optic nerve disorders: Unilateral optic neuropathies are common causes of RAPD.
• Demyelination Optic neuritis: Even very mild optic neuritis with a minimal loss of vision can lead to a
very strong RAPD.
• Ischemic optic neuropathies: These include arteritic (Giant Cell Arteritis) and non-arteritic causes.
Usually there will be a loss of vision or a horizontal cut in the visual field.
• Glaucoma: While glaucoma normally is a bilateral disease, if one optic nerve has particularly severe
damage, an RAPD can be seen.
• Traumatic optic neuropathy: This includes direct ocular trauma, orbital trauma, and even more remote
head injuries which can damage the optic nerve as it passes through the optic canal into the cranial
vault.
• Optic nerve tumor: This is a rare cause and includes primary tumors of the optic nerve (glioma,
meningioma) or tumors compressing the optic nerve (sphenoid wing meningioma, pituitary lesions,
etc.).
• Compressive optic neuropathy with or without orbital disease: This could include compressive damage to
the optic nerve from thyroid related orbitopathy (compression from enlarged extraocular muscles in the
orbit), orbital tumors, or vascular malformations.
• Radiation optic nerve damage
• Hereditary optic neuropathies, such as Leber's optic neuropathy (usually eventually bilateral) and other
inheritable optic neuropathies.
• Other optic nerve infections or inflammations: Cryptococcus can cause a severe optic nerve infection in
the immunocompromised. Sarcoidosis can cause inflammation of the optic nerve. Lyme disease can
affect the optic nerve.
• Optic atrophy status: post papilledema - This is usually bilateral.
• Post Surgical damage to the optic nerve: This could include damage following retrobulbar anesthesia;
damage following orbital hemorrhage related to eye, orbital, sinus, or plastic surgery; damage following
neurosurgical procedures such as pituitary tumor resection; and damage related to migration of an
orbital plate after surgery to correct a blow-out fracture.

Retinal Causes of a Relative Afferent Pupillary Defect


Again, symmetrically bilateral retinal disease will not show a RAPD. Usually, retinal disease has to be
quite severe for an RAPD to be clinically evident.

• Ischemic retinal disease: Causes include ischemic central retinal vein occlusion, central retinal artery
occlusion, severe ischemic branch retinal or arterial occlusions, severe ischemic diabetic or sickle-cell
retinopathy.
• Ischemic ocular disease (Ocular ischemic syndrome): This usually arises from obstruction of the
ophthalmic or carotid artery on one side.
• Retinal detachment: A RAPD can often be seen if the macula is detached, or if at least two quadrants
of retina are detached.

Only for Study purpose / Only Notes / AMC MCQ


Only Study Notes / Only for Study purposes / Personal notes/ AMC MCQ

• Severe macular degeneration: If unilateral and severe, a RAPD can be seen. Usually the visual acuity
would be less than 20/400.
• Intraocular tumor: Retinal and choroidal tumors including melanoma, retinoblastoma, and metastatic
lesion could lead to a RAPD if severe.
• Retinal infection: Cytomegalovirus, herpes simplex, and other causes of retinitis can lead to a RAPD if
there is extensive disease.

Other Causes of a Relative Afferent Pupillary Defect


• Amblyopia: If severe, can lead to a relative afferent pupillary defect. Usually the visual acuity would be
20/400, or worse.
• Cerebral vascular disease: Usually, it is an optic nerve disorder that leads to a RAPD, rather than an
optic tract or visual cortex disorder. However, there tends to be a higher percentage of crossed vs.
uncrossed nerve fibers at the optic chiasm. Thus, in a patient with a homonymous hemianopia from an
optic tract disorder, an RAPD could be seen in the eye with the temporal visual field defect. The nasal
retina serves the temporal visual field, and these are the fibers that would cross at the chiasm .
[3]

RAPD Diagnosis and Challenges with the Swinging Flashlight Test


Pupils are inspected for size, equality, and regularity during an eye exam. Each pupil should constrict
quickly and equally during exposure to direct light and to light directed at the other pupil (the
consensual light reflex). Using the swinging light test, physicians test and observe the pupillary response
to consensual light in order to determine if there is a defect present. Normally, the pupil constriction
does not change as the light is swung from eye to eye. When the light is moved quickly from eye to
eye, both pupils should hold their degree of constriction. But even under the best condition, it is hard
to perform this manual test accurately. Human factors, including examiner bias, light position
variability, and endpoint determination, may all influence the identification and appropriate
quantification of RAPD in patients . Moreover, Other factors unique to any given individual such as
[4]

dark irises, anisocoria or small pupils, and efferent defects may even make it much more difficult to
detect small amounts of asymmetry in pupillary reactions . Although known to be an important
[5]

physical sign, many well-practiced doctors report little incidence – not because the sign is not present,
but because it is difficult or impossible to detect subtle abnormalities and it is rarely quantified [6]
.
Misdiagnosis of optic nerve disease in primary stages, may lead to irreversible visual loss. In a recent
study , the inter-examiner disagreement in the manual evaluation of pupillary reaction was as high
[7][8]

as 39% [9] . Such high amount of disagreement justifies the need for having more appropriate test
methods.

Quantification of RAPD
Various techniques have been described to quantify or measure APDs. These include the use of neutral
density filters , cross-polarized filters
[10] , and subjective grading based on the amount of initial
[11]

contraction and subsequent re-dilation of each pupil as the light is swung . Although these techniques
[12]

have been shown to be effective and accurate, a number of factors influence the validity, variability,
and reliability of such measurements. These techniques, although objective in their quantification, are

Only for Study purpose / Only Notes / AMC MCQ


Only Study Notes / Only for Study purposes / Personal notes/ AMC MCQ

unfortunately subjective in their endpoint. clinical grading in plus scale and quantification with various
filters techniques are comparable.

Digital Marcus Gunn Test as an Alternative Solution


The development of personal computer-based infrared video instruments has allowed pupillography to
enter the clinical arena. Measuring pupil diameter for refractive surgery, distinguishing Horner syndrome
from physiologic anisocoria, quantifying the relative afferent pupillary defect, and plotting visual fields
by means of graded pupil constriction to focal light stimuli are recent applications in ophthalmology.
The primary objective of advanced and customized pupilometers is to eliminate the limitations in
conventional testing procedures for measuring afferent eye defects. A special type of digital
pupilometers perform the swinging flashlight test automatically based on the known guidelines and the
standard procedure . Several issues are being considered in designing a device for perfuming Marcus
[13]

Gunn test accurately and consistently. Usually, a mechanical frame is designed such that each eye is
kept in full isolation from the other eye and external lighting sources. It means that no light can be
sensed by the contralateral eye when a light source (which can be a full-color LED) is emitting light to
the right or the left eye. Image processing algorithms will be fed by real-time images of the anterior
segment of patients’ both eyes that taken by high-resolution camera while one of the eyes is being
stimulated with controllable lights automatically or manually. These computer algorithms separate pupils
from other eye components and measure pupils size (Area, diameter, etc.) accurately. Usually, these
devices provide eye care professional with more information such as surface area of both pupils and
dilation velocity which are not possible to measure with naked eye. This extra information is very
useful not only for research purposes but also for improving diagnosing abilities of the doctor. Overall
the quality of the test is improved by having better diagnostic accuracy, as well as documenting the
test results for future follow-ups.

Diabetic Papillopathy
Diabetic papillopathy is characterized by optic disc edema in the absence of optic nerve dysfunction,
elevated intracranial pressure, nerve infiltration or infection,[236–241] afferent pupillary defect and
dyschromatopsia.[241] The condition is self-limiting and vision generally recovers to better than 20/30 in
most patients.[240] Occurrence is rare, with an incidence of 0.5% reported by Regillo et al..[241] It is
important for clinicians to be aware that this condition can mimic papilledema[242] and optic nerve
neovascularization.[243] Please refer to Figures 5 & 6.
Diabetic papillopathy, patient's left eye. (A) Late-phase fluorescein angiogram shows leakage from the optic
nerve head with surrounding peripapillary hemorrhages. (B) Late-phase fluorescein angiogram 2 weeks
later shows almost complete resolution of leakage from the optic nerve head with decrease in the
hemorrhages. Vision improved from counting fingers to 20/50.
Diabetic papillopathy, patient's right eye. (A) Late-phase fluorescein angiogram shows leakage from the
optic nerve head with counting fingers vision. (B) 2 weeks later, there is significant decrease in the leakage
on the late-phase angiogram and improvement in vision to 20/40.
Reproduced with permission from [309].

Anterior Ischemic Optic Neuropathy


Anterior ischemic optic neuropathy is clinically classified as an acute, pallid optic disc swelling (followed by
optic nerve pallor) with afferent pupillary defects associated with visual field defects.[236] Telangectasia of

Only for Study purpose / Only Notes / AMC MCQ


Only Study Notes / Only for Study purposes / Personal notes/ AMC MCQ

disc vessels may occur, which may be mistaken for disc neovascularization.[244] Anterior ischemic optic
neuropathy is thought to be precipitated by circulatory insufficiency in nonarteritic ischemic optic
neuropathy (NAION).[245] Diabetic patients are at increased risk of developing NAION.[246–248] A study
by the Ischemic Optic Neuropathy Decompression Trial Study Group, designed to determine the baseline
clinical characteristics of patients with NAION, found that 24% of patients had a history of diabetes.[246]
Patients with NAION in one eye were reported to have an increased incidence of optic disc crowding in
their contralateral eye.[249–253]

Optic nerve decompression surgery not only failed to improve the visual outcome in these patients,[254] but
was also associated with a worse prognosis.[255] Aspirin treatment appeared to reduce the occurrence of
NAION in the contralateral eye of patients in a retrospective study.[256] While neuroprotective agents have
been beneficial for the treatment of secondary neuronal degeneration in animal models, further studies will
be needed in order to determine if they are effective in the treatment of NAION.[257]

Glaucoma
Glaucoma affects an estimated 70 million people worldwide, of whom 6.7 million are blind secondary to the
disease process.[258] It is an optic neuropathy defined by changes in the optic nerve and associated visual
field defects. While patients may or may not present with elevated IOP, interestingly, significantly elevated
IOP has more often been reported in diabetic than nondiabetic individuals.[259–263] The Beijing Eye Study
reported an association between ocular hypertension and diabetes.[262] Interestingly, diabetes was
reported to reduce the risk of advancement from ocular hypertension to open-angle glaucoma (OAG).[264]
This was supported by the Singapore Malay Eye Study, which found that while IOP was significantly higher
in patients with diabetes, these patients did not go on to develop more OAG than nondiabetic subjects;[260]
these results were supported by others.[261,263] Several studies did, however, report a direct association
between OAG and diabetes. The Beaver Dam Eye Study,[265] the Blue Mountains Eye Study[266] and the
Nurses' Health Study[267] all found a significant association between diabetes and glaucoma. The Los
Angeles Latino Eye Study reported that OAG was 40% more prevalent in Type 2 diabetic Latino subjects,
especially those with diseases of long duration.[268]

A 2004 meta-analysis concluded that diabetes increases the risk of primary OAG (POAG) by 1.5-fold.[269]
An increased risk of progression of POAG was reported in diabetic patients in the Advanced Glaucoma
Intervention Study (AGIS)[270] and Collaborative Initial Glaucoma Treatment Study (CIGTS).[271] By
contrast, the Barbados Incidence Study of Eye Diseases, the Melbourne Visual Impairment Project and the
Rotterdam Eye Study[272–275] failed to conclude that diabetes was a risk factor for the development of
POAG. These findings were in agreement with those of the Singapore Malay Eye Study and the Baltimore
Eye Study.[260,261] The Early Manifest Glaucoma Trial,[276,277] the Collaborative Normal Tension
Glaucoma Study[278] and a study by Kooner et al.[279] found no associated increased risk of POAG
progression and diabetes. An increased incidence of narrow-angle glaucoma was reported in Type 2
diabetic patients and individuals with impaired glucose tolerance.[280] One important contributing factor
was speculated to be the effect of elevated serum glucose on lens swelling, which can directly lead to angle
closure.[6]

Neovascular glaucoma (NVG) is a complication of PDR that is thought to develop as a result of VEGF-
induced neovascularization of the iris and angle;[281] the newly formed blood vessels and fibrous tissue
block aqueous outflow from the anterior chamber. It is estimated that up to 43% of NVG is caused by
PDR.[282] Current therapy involves immediate treatment to reduce IOP and subsequent pan-retinal
photocoagulation, the latter of which was shown to induce vascular regression if applied early.[283,284]
Visual prognosis in NVG is poor.[285] A recent study by Vasudev et al. reported somewhat improved, but
still poor, visual outcome in patients treated with bevacizumab and PRP.[286]

Wolfram Syndrome

Only for Study purpose / Only Notes / AMC MCQ


Only Study Notes / Only for Study purposes / Personal notes/ AMC MCQ

Wolfram syndrome is a rare condition characterized by Type 1 diabetes and optic atrophy; progressive
impairment of hearing has also been associated with this condition.[287,288] The estimated prevalence of
Wolfram syndrome is approximately 1 in 770,000 in the general population and 1 in 150 cases of Type 1
diabetes.[287] The key ophthalmologic finding in these patients is optic atrophy,[288] although low VA,
color vision defects and visual field defects have also been described

93- A 67-year-old woman presents to the emergency department complaining of right eye blindness. She
states she first noticed the vision loss when she woke up this morning. She denies any pain or other
associated symptoms. Review of systems is notable for passive suicidal ideation as the patient's husband
died in the ICU last night. Her temperature is 98.3°F
(36.8°C), blood pressure is 174/104 mmHg, pulse is
80/min, respirations are 12/min, and oxygen saturation is
98% on room air. Physical exam is notable for the patient
being unable to see out of her right eye with normal vision
in her left eye. Her cranial nerves are otherwise intact and
she has normal strength and sensation in her upper and
lower extremities. A fundoscopic examination is performed
and the results are demonstrated in Figure A. Which of the
following is the most likely etiology of this patient's
symptoms?
1Arterial embolization
2Detachment of the retina
3Emotional stress
4increased intraocular pressure
5Ischemia of the brain
The patient presents with central retinal artery occlusion, characterized by sudden, painless blindness and
a pale retina with a cherry-red macula on funduscopic evaluation.

Central retinal artery occlusion (CRAO) is an eye condition where the flow of blood through the central
retinal artery is occluded. It is often associated with carotid artery atherosclerosis and a carotid bruit will
often be found on physical exam. CRAO often presents with acute, painless monocular vision loss. Retinal
whitening with a cherry-red spot is observed on funduscopic exam. Treatment consists of ocular massage,
increasing CO2 levels (for vasodilation), and an ophthalmology consultation.

Figure/Illustration A demonstrates retinal whitening with


a cherry-red spot (red arrow) as is commonly seen in
CRAO.

Incorrect Answers:
Answer 2: Detachment of the retina presents with loss
of vision like a "curtain is coming down over the eye"
and presents with a free flap on ultrasound. Treatment
may involve positioning of the head, injection of gas,
and cryoretinopexy.

Answer 3: Emotional stress can cause conversion


disorder where the patient has sudden-onset neurologic
symptoms after a life stressor. Typically, the patient is indifferent to these symptoms or not overly
concerned (La belle Indifference) and the patient's neurologic deficits will not cause the patient harm
(meaning even if claiming they are blind, they will not trip over objects). This patient's pallor on fundoscopy
suggests against this diagnosis.

Only for Study purpose / Only Notes / AMC MCQ


Only Study Notes / Only for Study purposes / Personal notes/ AMC MCQ

Answer 4: Increased intraocular pressure describes glaucoma which can present with unilateral vision loss
as well as a rock hard eye and a steamy pupil with an elevated intraocular pressure.

Answer 5: Ischemia of the brain describes an ischemic stroke which would present with focal neurologic
deficits in the distribution of a vascular territory. Isolated vision loss with pallor on fundoscopy suggests
against this diagnosis.

Bullet Summary:
Central retinal artery occlusion presents with sudden, monocular vision loss and pallor on fundoscopy.

94- Macular oedema in fundoscopy in Htn and DM n o options


Manifestations of diabetic retinopathy include microaneurysms, intraretinal hemorrhage,
exudates, macular edema, macular ischemia, neovascularization, vitreous hemorrhage, and traction retinal
detachment. Symptoms may not develop until late in the disease. Diagnosis is by funduscopy; further
details are elucidated by color fundus photography, fluorescein angiography, and optical coherence
tomography. Treatment includes control of blood glucose and BP. Ocular treatments included retinal laser
photocoagulation, intravitreal injection of antivascular endothelial growth factor drugs (eg, aflibercept,
ranibizumab, bevacizumab), intraocular corticosteroids, vitrectomy, or a combination.
Diabetic retinopathy is a major cause of blindness, particularly among working-age adults. The degree of
retinopathy is highly correlated with

Only for Study purpose / Only Notes / AMC MCQ


Only Study Notes / Only for Study purposes / Personal notes/ AMC MCQ

• Duration of diabetes
• Blood glucose levels
• BP levels
Pregnancy can impair blood glucose control and thus worsen retinopathy.
Nonproliferative retinopathy
Nonproliferative retinopathy (also called background retinopathy) develops first and causes increased
capillary permeability, microaneurysms, hemorrhages, exudates, macular ischemia, and macular edema
(thickening of the retina caused by fluid leakage from capillaries).
Proliferative retinopathy
Proliferative retinopathy develops after nonproliferative retinopathy and is more severe; it may lead to
vitreous hemorrhage and traction retinal detachment. Proliferative retinopathy is characterized by abnormal
new vessel formation (neovascularization), which occurs on the inner (vitreous) surface of the retina and
may extend into the vitreous cavity and cause vitreous hemorrhage. Neovascularization is often
accompanied by preretinal fibrous tissue, which, along with the vitreous, can contract, resulting in traction
retinal detachment. Neovascularization may also occur in the anterior segment of the eye on the iris;
neovascular membrane growth in the anterior chamber angle of the eye at the peripheral margin of the iris
can occur, and this growth leads to neovascular glaucoma. Vision loss with proliferative retinopathy may be
severe.
Clinically significant macular edema can occur with nonproliferative or proliferative retinopathy and is the
most common cause of vision loss due to diabetic retinopathy.
Nonproliferative retinopathy
Vision symptoms are caused by macular edema or macular ischemia. However, patients may not have
vision loss even with advanced retinopathy. The first signs of nonproliferative retinopathy are
• Capillary microaneurysms

• Dot and blot retinal hemorrhages


• Hard exudates
• Cotton-wool spots (soft exudates)
Diabetic Retinopathy (Nonproliferative)

PAUL WHITTEN/SCIENCE PHOTO LIBRARY


Hard exudates are discrete, yellow particles within the
retina. When present, they suggest chronic edema.
Cotton-wool spots are areas of microinfarction of the
retinal nerve fiber layer that lead to retinal
opacification; they are fuzzy-edged and white and
obscure underlying vessels.
Signs in later stages are
• Macular edema (seen on slit-lamp
biomicroscopy as elevation and blurring of
retinal layers)
• Venous dilation and intraretinal microvascular
abnormalities
Proliferative retinopathy
Symptoms may include blurred vision, floaters (black spots) or flashing lights in the field of vision, and
sudden, severe, painless vision loss. These symptoms are typically caused by vitreous hemorrhage or
traction retinal detachment.

Only for Study purpose / Only Notes / AMC MCQ


Only Study Notes / Only for Study purposes / Personal notes/ AMC MCQ

Proliferative retinopathy, unlike nonproliferative retinopathy, causes formation of fine preretinal vessel
neovascularization visible on the optic nerve or retinal surface. Macular edema or retinal hemorrhage may
be visible on funduscopy.
Macular edema is the build-up of fluid in the
macula, an area in the center of the retina. The
retina is the light-sensitive tissue at the back of the
eye and the macula is the part of the retina
responsible for sharp, straight-ahead vision. Fluid
buildup causes the macula to swell and thicken,
which distorts vision.
Macular edema occurs when there is abnormal
leakage and accumulation of fluid in the macula
from damaged blood vessels in the nearby retina. A
common cause of macular edema is diabetic
retinopathy, a disease that can happen to people
with diabetes.
https://www.msdmanuals.com/professional/SearchResults?query=hypertensive+retinopathy

Hypertensive retinopathy is retinal vascular damage caused by hypertension. Signs usually


develop late in the disease. Funduscopic examination shows arteriolar constriction, arteriovenous nicking,
vascular wall changes, flame-shaped hemorrhages, cotton-wool spots, yellow hard exudates, and optic disk
edema. Treatment is directed at controlling blood pressure and, when vision loss occurs, treating the retina.

Acute blood pressure elevation typically causes reversible vasoconstriction in retinal blood vessels, and
hypertensive crisis may cause optic disk edema. More prolonged or severe hypertension leads to exudative
vascular changes, a consequence of endothelial damage and necrosis. Other changes (eg, arteriole wall
thickening, arteriovenous nicking) typically require years of elevated blood pressure to develop. Smoking
compounds the adverse effects of hypertensive retinopathy.
Hypertension is a major risk factor for other retinal disorders (eg, retinal artery or vein occlusion, diabetic
retinopathy). Also, hypertension combined with diabetes greatly increases risk of vision loss. Patients with
hypertensive retinopathy are at high risk of hypertensive damage to other end organs.
Symptoms usually do not develop until late in the disease and include blurred vision or visual field defects.
In the early stages, funduscopy identifies arteriolar constriction, with a decrease in the ratio of the width of
the retinal arterioles to the retinal venules.
Chronic, poorly controlled hypertension causes the following:
• Permanent arterial narrowing

• Arteriovenous crossing abnormalities (arteriovenous nicking)


• Arteriosclerosis with moderate vascular wall changes (copper wiring) to more severe vascular wall
hyperplasia and thickening (silver wiring)
Sometimes total vascular occlusion occurs. Arteriovenous nicking is a major predisposing factor to the
development of a branch retinal vein occlusion.
If acute disease is severe, the following can develop:
• Superficial flame-shaped hemorrhages

• Small, white, superficial foci of retinal ischemia (cotton-wool spots)


• Yellow hard exudates
• Optic disk edema
Yellow hard exudates represent intraretinal lipid deposition from leaking retinal vessels. These exudates
can develop a star shape within the macula, particularly when hypertension is severe. In severe

Only for Study purpose / Only Notes / AMC MCQ


Only Study Notes / Only for Study purposes / Personal notes/ AMC MCQ

hypertension, the optic disk becomes congested and edematous (papilledema indicating hypertensive
crisis).

Only for Study purpose / Only Notes / AMC MCQ


Only Study Notes / Only for Study purposes / Personal notes/ AMC MCQ

95-. A boy presented with a punch in the face. He is asked to look upward and presented this-wats cause-
a.zygoma fracture
b.blowout fractur

Only for Study purpose / Only Notes / AMC MCQ


Only Study Notes / Only for Study purposes / Personal notes/ AMC MCQ

96- No purulent discharge, no itching. No impairment of vision. Which eyedrop will you give?
A. Hypromellose
B. Ketotifen
C. Sodium cromoglycate
D. Prednisolone
E. Chloramphenicol

Papilledema Review:
Papilledema
Papilledema is a condition in which increased pressure in or around the brain causes the part of the
optic nerve inside the eye to swell.

Only for Study purpose / Only Notes / AMC MCQ


Only Study Notes / Only for Study purposes / Personal notes/ AMC MCQ

• Symptoms may be fleeting disturbances in vision, headache, vomiting, or a combination.


• Doctors make the diagnosis by looking in the person’s eye with an ophthalmoscope.
• The disorder causing increased brain pressure is treated as soon as possible.
Causes
Papilledema is usually caused by the following:
• Brain tumor or abscess

• Head injury
• Bleeding in the brain
• Inflammation of the brain (encephalitis) or its tissue coverings (meningitis)
• Idiopathic intracranial hypertension
• Uncontrolled, life-threatening hypertension
These conditions typically result in papilledema in both eyes.
At first, papilledema may be present without affecting vision. Fleeting vision changes—blurred vision,
double vision, flickering, or complete loss of vision—typically lasting seconds are characteristic of
papilledema. Other symptoms may be caused by the elevated pressure in the brain. Headache, nausea,
vomiting, or a combination may occur. This disorder does not cause pain.
• A doctor's evaluation

• Imaging tests
• Lumbar puncture (spinal tap)
To diagnose papilledema, a doctor uses an ophthalmoscope (a light with magnifying lenses that is used to
look into the back of the eye). Often an ophthalmologist (a medical doctor who specializes in the evaluation
and treatment of eye disorders) needs to confirm the diagnosis and help determine the cause.
Magnetic resonance imaging (MRI) or computed tomography (CT) may be used to help determine the
cause and monitor the effect of treatment.
A lumbar puncture (spinal tap) is done to measure the pressure of the cerebrospinal fluid unless something
is seen on the MRI or CT scan indicating a spinal tap is not safe to do. A sample of the cerebrospinal fluid
may be examined for evidence of a brain tumor or infection.
Sometimes ultrasonography of the eye is done to distinguish between papilledema and other disorders that
cause apparent swelling of the optic nerve.
• Treatment of cause

The disorder causing increased brain pressure is treated as soon as possible. For example, if the high
pressure of the cerebrospinal fluid is caused by a brain tumor, corticosteroids may be given, but surgery to
remove the tumor or radiation therapy to decrease its size may be needed.
Papilledema that occurs as a result of idiopathic intracranial hypertension can be treated with weight loss
and a diuretic. If unsuccessful, surgical procedures can be done.
An infection, if bacterial, can be treated with antibiotics.
A brain abscess is drained, and antibiotics are given.

Only for Study purpose / Only Notes / AMC MCQ


Only Study Notes / Only for Study purposes / Personal notes/ AMC MCQ

97- Man with excessive tearing for two weeks, no vision impairment, no purulent discharge and not
itchy. What eye drops should give to him? Pic is given but there is no sign of bacterial
conjunctivitis, I think. Only reddening of lower eye lids.
A. Ketotifen
B. chloramphenicol
C. artificial tears
D. prednisolone
E. sodium cromoglycate

No drops or ointments can treat viral


conjunctivitis. Antibiotics will not
cure a viral infection. Like a common
cold, the virus has to run its course,
which may take up to two or three
weeks. Symptoms can often be
relieved with cool compresses and
artificial tear solutions

Viral — There is no specific


antiviral agent for the treatment of
viral conjunctivitis. Some patients
derive symptomatic relief from
topical
antihistamine/decongestants.
These are available over-the-
counter (Naphcon-A, Ocuhist,
generics). It is worthwhile to
explain that these agents treat the
symptoms but not the disease, just
as "cold remedies" treat the symptoms rather than the cause of a cold. Warm or cool compresses
may provide additional symptomatic relief. Systemic agents play no role in viral conjunctivitis.

Some providers prescribe antibiotic ointments for viral conjunctivitis to provide lubrication. A more
sensible alternative is to use nonantibiotic lubricating agents such as those used for noninfectious
conjunctivitis (table 1). (See 'Noninfectious, nonallergic conjunctivitis' above.)

Patients must be told that the eye irritation and discharge may get worse for three to five days
before getting better, that symptoms can persist for two to three weeks, and that use of any topical
agent for that duration might result in irritation and toxicity, which can itself cause redness and
discharge. Clinicians must be wary of trying one agent after another in patients with viral
conjunctivitis who are expecting drugs to "cure" their symptoms. Patient education is often more
effective than prolonged or additional therapies for patients who experience improvement but
incomplete resolution of symptoms after a few days.

Only for Study purpose / Only Notes / AMC MCQ


Only Study Notes / Only for Study purposes / Personal notes/ AMC MCQ

Only for Study purpose / Only Notes / AMC MCQ


Only Study Notes / Only for Study purposes / Personal notes/ AMC MCQ

Only for Study purpose / Only Notes / AMC MCQ


Only Study Notes / Only for Study purposes / Personal notes/ AMC MCQ

98- Child with periorbital swelling and fever, when open eye lid showing yellow discharge
What to test
1. Eye swab
2. Blood culture
3. Lumbar puncture
4. Ct head
It is extremely important to differentiate
periorbital cellulitis and orbital cellulitis
which is a medical emergency. Both will
have periorbital edema and redness and
the patient might complain of recent
rhinorrhea, difficulty opening eye and
excessive tearing.
Patients with orbital cellulitis might also
present with proptosis, fever and may
look systemically unwell.
Other red flags when examining a child
with suggestive orbital cellulitis includes
reduced visual acuity, limited, painful or
reduced eye movements, impaired
colour vision and relative pupillary
defects.
The diagnosis of periorbital cellulitis can
often be made clinically without any
further testing. However, it can be
difficult to examine children with swollen
eyes and thus difficult to make a
definitive diagnosis.
Radiologic imaging is often required and
is necessary if orbital cellulitis is
suspected. CT scans of the orbits and
sinuses need to be done. Sinusitis is the
most common predisposing factor to
orbital cellulitis and abscess formation.
CT imaging allows rapid and accurate
clinical staging of the condition and guides treatment. MRI can also be used, but it is practically difficult for
younger children and is not always available in smaller centres. Imaging can show where the infection is
and its severity.
Blood tests are often performed but they have limited diagnostic value. However, tests can be used to
monitor patient progress. High white blood cell count, C-reactive protein or positive ESR may suggest
orbital cellulitis. Blood cultures and eye swabs can also be done, but do not always yield microbiologically
diagnostic results.

Only for Study purpose / Only Notes / AMC MCQ


Only Study Notes / Only for Study purposes / Personal notes/ AMC MCQ

99- A 65-year-old man is brought to his


primary care provider by his concerned wife.
She reports he has had this "thing" on his eye
for years and refuses to seek care. He denies
any pain or discharge from the affected eye.
A picture of his eye is shown below. Given the
diagnosis, what are you most likely to
discover when taking this patient's history?

1He experienced shingles three years ago,


with a positive Hutchinson's sign

2He suffered from recurrent conjunctivitis in


his youth

3He grew up in Ecuador, where he worked


outdoors as a farmer for 30 years

4He was involved in a bar fight and


experienced a ruptured globe 10 years ago

5 He suffered a burn to his eye while cleaning


his bathroom with bleach 5 years earlier

The patient is most likely suffering from a pterygium. Living near the equator is associated with pterygium
development.

Pterygium are benign growths of the conjunctiva associated with excessive exposure to wind, sunlight, or
sand. Other risk factors include working outdoors with high exposure to UV rays. Symptoms of pterygium
include persistent irritation, redness, foreign body sensation, and tearing. If the pterygium advances over
the cornea it can impair vision.

Peate discusses work related eye injuries including corneal abrasions, burns, and globe injury. The review
states that eye patching does not reduce discomfort following corneal abrasions. Working with chisels,
hammers, grinding wheels, or saws places patients at risk for globe injuries. The treatment of chemical eye
burns is immediate irrigation with copious amounts of fluid.

Only for Study purpose / Only Notes / AMC MCQ


Only Study Notes / Only for Study purposes / Personal notes/ AMC MCQ

Janson et al. review the surgical management of pterygium. Pterygium are noncancerous proliferations of
conjunctiva. Treatment is most often surgical and may be required because of discomfort, cosmesis, or
obstruction of vision. Following surgical excision, pterygia can often recur.

Figure A displays a ptergium which is characterized by elastotic degeneration of collagen and fibrovascular
proliferation

Hutchinson's sign. Hutchinson's


sign is a clinical sign which may refer
to: Hutchinson's pupil, an
unresponsive and enlarged pupil on the
side of an intracranial mass. Vesicles
on the tip of the nose, or vesicles on
the side of the nose, precedes the
development of ophthalmic herpes
zoster.

99- 40 year old woman of retro orbital


pain and reduced visual acuity for 3
days. What is the most appropriate
investigation?
A- Temporal artery biopsy
B- CT scan
C- Visual evoked potential
D- fundoscopy
E- gonioscopy
Retro ocular pain, 40yrS loss
vision over a few days—optic
neuritis.
Optic neuritis Best MRI if not
VEP

100- Patient came with history of reduced vision she is asking of about MS as she searched in the web.
What is percentage of development of neurological deficit from episode of optic neuritis
1. 10%.
2. 30 %.
3. 50%.
4. 70%
5. 90%
https://www.uptodate.com/contents/optic-neuritis-pathophysiology-clinical-features-and-
diagnosis?search=optic%20neuritis&source=search_result&selectedTitle=1~150&usage_type=default&disp
lay_rank=1
JM says around 60% but Mayo, RACGP, Uptodate say 50%

Only for Study purpose / Only Notes / AMC MCQ


Only Study Notes / Only for Study purposes / Personal notes/ AMC MCQ

Optic neuritis is the presenting feature of MS in 15 to 20 percent of patients and occurs in 50 percent at
some time during the course of their illness. Uptodate
What percentage of optic neuritis patients have MS? Optic neuritis is the first sign of MS in about 15 to 20
percent of people who have MS. The lifetime risk of developing MS after an episode of optic neuritis is
about 50 percent according to the Mayo Clinic.
Studies show that in about 15–20% of MS cases, ON was the presenting symptom and more than half of
people with MS experience at least one episode of ON during their disease RACGP

Optic Neuritis:
INTRODUCTION
Optic neuritis is an inflammatory, demyelinating
condition that causes acute, usually monocular,
visual loss. It is highly associated with multiple
sclerosis (MS). Optic neuritis is the presenting
feature of MS in 15 to 20 percent of patients and
occurs in 50 percent at some time during the course
of their illness [1-4].
The term "optic neuritis" is sometimes applied to
other inflammatory and infectious conditions affecting
the optic nerve. These and other causes of optic
neuropathy are discussed separately. (See "Optic
neuropathies".)
The epidemiology, pathophysiology, clinical features,
and diagnosis of demyelinating optic neuritis will be
covered here. Prognosis and treatment of optic
neuritis and other clinical manifestations of MS are
discussed separately.
EPIDEMIOLOGY
Most cases of acute demyelinating optic neuritis
occur in women (two-thirds) and typically develop in
patients between the ages of 20 and 40 [5-7].
The incidence of optic neuritis is highest in
populations located at higher latitudes, in the
northern United States and western Europe, and is
lowest in regions closer to the equator. In the United States, studies have estimated the annual incidence of
optic neuritis to be as high as 6.4 per 100,000 [8,9]. In the United States, optic neuritis occurs more
frequently in whites than blacks [10]. In Asia, optic neuritis is proportionately more common relative to the
incidence of multiple sclerosis (MS) than in the United States or Western Europe
PATHOPHYSIOLOGY
The most common pathologic basis for optic neuritis is inflammatory demyelination of the optic nerve. The
pathology is similar to that of acute multiple sclerosis (MS) plaques in the brain, with perivascular cuffing,
edema in the myelinated nerve sheaths, and myelin breakdown. Inflammation of the retinal vascular
endothelium can precede demyelination and is sometimes visibly manifest as retinal vein sheathing [12].
Myelin loss exceeds axonal loss.
It is believed that the demyelination in optic neuritis is immune mediated, but the specific mechanism and
target antigen(s) are unknown. Systemic T cell activation is identified at symptom onset and precedes
changes in the cerebrospinal fluid (CSF) [13]. Systemic changes also normalize earlier (within two to four
weeks) than central changes. T cell activation leads to the release of cytokines and other inflammatory
agents. B cell activation against myelin basic protein is not seen in peripheral blood but can be
demonstrated in the CSF of patients with optic neuritis [14].
As with MS, a genetic susceptibility for optic neuritis is suspected. This is supported by an over-
representation of certain human leukocyte antigen (HLA) types among patients with optic neuritis

Only for Study purpose / Only Notes / AMC MCQ


Only Study Notes / Only for Study purposes / Personal notes/ AMC MCQ

CLINICAL FEATURES
Acute features — Optic neuritis is usually monocular in its clinical presentation. In approximately 10 percent
of cases, symptoms occur in both eyes, either simultaneously or in rapid succession [17]. Bilateral optic
neuritis is more common in children younger than 12 to 15 years old and also in Asian and black South
African patients [17-22]. Because bilateral symptoms are relatively uncommon, they should suggest an
alternative cause of optic neuropathy. However, subclinical visual deficits in acuity, contrast sensitivity,
color vision, and visual field in the contralateral eye can often be elicited by detailed visual testing in
patients with clinically monocular disease [9,23]. Because these deficits usually resolve along with the
clinical deficits in the symptomatic eye, it is unlikely that these findings represent prior episodes of optic
neuritis.
Other clinical features of optic neuritis were systematically characterized in the Optic Neuritis Treatment
Trial (ONTT), which enrolled 457 patients, aged 18 to 46 years, with acute unilateral optic neuritis [7,24].
The two most common symptoms of optic neuritis are vision loss and eye pain:
●Vision loss typically develops over a period of hours to days, peaking within one to two weeks. Continued
deterioration after that time suggests an alternative diagnosis [1,2]. Greater than 90 percent of patients in
the ONTT had a significant decrease in central visual acuity. In most, the visual acuities ranged from 20/25
to 20/190 (median visual acuity 20/60). However, some patients had 20/20 acuity (11 percent), and, at the
other extreme, a few had no light perception (3 percent).
●Eye pain occurred in 92 percent of patients in the ONTT and often worsened with eye movement [7]. The
onset of pain generally coincided with the visual acuity loss and improved along with it.

Other common visual symptoms and signs include:


●An afferent pupillary defect always occurs in optic neuritis if the other eye is uninvolved and otherwise
healthy. This is demonstrated by shining a light alternately in one eye and then the other and finding that
the direct response to light is more sluggish in the affected eye. The room should be dark, and the patient
should fixate on a distant target to prevent miosis due to accommodation. (See "The detailed neurologic
examination in adults", section on 'Afferent pupillary defect'.)
●The visual field defect in optic neuritis is typically characterized as a central scotoma [1,25]. However, in
the ONTT, almost all types of visual field defects were seen, including diffuse vision loss and altitudinal,
arcuate, hemianopic, and cecocentral defects. Nonetheless, a defect that extends to the periphery should
suggest a compressive lesion, while an altitudinal defect, particularly an inferior altitudinal defect, is more
common in anterior ischemic optic neuropathy [1,25]. Visual field defects usually resolve; in the ONTT, 56
percent had normalized at one year and 73 percent had normalized at 10 years [26,27].
●Papillitis with hyperemia and swelling of the disk, blurring of disk margins, and distended veins is seen in
one-third of patients with optic neuritis (picture 1) [7]. Two-thirds of these patients have retrobulbar neuritis
with a normal funduscopic examination (picture 2). Papillitis is more common in children less than 14 years
old and in certain ethnic populations, including black South Africans and Southeast Asians [11,18-20].
Peripapillary hemorrhages are rare in optic neuritis but are a common accompaniment to papillitis due to
anterior ischemic optic neuropathy [1].
●Photopsias (flickering or flashes of light) are often precipitated with eye movement and were reported by
30 percent of patients in the ONTT [7].

●Loss of color of vision out of proportion to the loss of visual acuity is specific to optic nerve pathology.
Abnormal color vision by Ishihara plates was found in 88 percent of involved eyes in the ONTT; this
increased to 94 percent with the more sensitive Farnsworth-Munsell 100 hue test [7].
●Other signs of ocular inflammation may be observed by the ophthalmologist on funduscopic or slit lamp
examination. Perivenous sheathing or periphlebitis retinae can be seen in approximately 12 percent of
patients with optic neuritis and implies a high risk for multiple sclerosis (MS) [9,12]. Uveitis, cells in the
anterior chamber, and/or pars planitis are uncommonly seen in optic neuritis and are more typical of
infections and other autoimmune diseases. (See "Optic neuropathies".)

Only for Study purpose / Only Notes / AMC MCQ


Only Study Notes / Only for Study purposes / Personal notes/ AMC MCQ

Chronic features — Even after clinical recovery, signs of optic neuritis can persist. These signs in a patient
without a history of optic neuritis may suggest a previous, subclinical attack. When a patient presents with a
possible first attack of MS elsewhere in the central nervous system, these signs are often sought because
evidence of other demyelinating episodes separated in "time and space" can affect prognosis and
treatment decisions.
Chronic signs of optic neuritis can include:
●Persistent visual loss. Most patients with optic neuritis recover functional vision within one year. However,
on testing, deficits in color vision, contrast sensitivity, stereo acuity, and light brightness are detectable in
most patients at up to two years [28]. (See "Optic neuritis: Prognosis and treatment", section on 'Recovery
of vision'.)
●A relative afferent pupillary defect remains in approximately one-fourth of patients two years after
presentation [28].
●Color desaturation refers to a qualitative intereye difference in color perception that can be tested by
comparing vision of a red object with each eye. A patient with monocular "red desaturation" may report that
the red color appears "washed out," pink, or orange when viewed with the affected eye.
●Temporary exacerbations of visual problems in patients can occur with increased body temperature
(Uhthoff phenomenon). Hot showers and exercise are classic precipitants.
●Optic atrophy to at least some degree almost always follows an attack of optic neuritis, despite the return
of visual acuity [29]. Normal, 20/20 visual acuity requires less than one-half of normal foveal axons [30].
The disc appears shrunken and pale, particularly in its temporal half (temporal pallor). The disk pallor
extends beyond the margins of the disk into the peripapillary retinal nerve fiber layer.
●The pattern-shift visual evoked response (VER) remains delayed in most patients, even with visual
recovery. Although latencies continue to shorten (improve) up to two years after presentation, abnormalities
are seen in most (80 percent) at two years [28,31,32]. (See 'Visual evoked response' below.)

DIFFERENTIAL DIAGNOSIS
In a young child, infectious and postinfectious causes of optic nerve impairment should be considered as
alternatives to optic neuritis, while in an older patient (>50 years), ischemic optic neuropathy (due, for
example, to diabetes mellitus or giant cell arteritis) is a more likely diagnosis than optic neuritis.
Alternative diagnoses should also be considered in patients with a bilateral presentation or those with other
neurologic or systemic symptoms.
In cases of recurrent optic neuritis that are not due to neuromyelitis optica (NMO) or multiple sclerosis
(MS), other causes of recurrent optic neuritis should be thoroughly investigated (such as sarcoidosis, lupus,
chronic relapsing inflammatory optic neuropathy [CRION], or paraneoplastic optic neuropathy [serum
CRMP-5/CV2 antibody]).
The differential diagnosis of optic nerve diseases is summarized in the tables and is discussed separately
(table 1 and table 2). (See "Optic neuropathies".)

EVALUATION AND DIAGNOSIS

Diagnosis — In general, optic neuritis is a clinical diagnosis based upon the history and examination
findings. Because important findings on funduscopic examination help differentiate typical from atypical
cases of optic neuritis, an ophthalmologic examination should be considered an essential feature of the
clinical evaluation. Magnetic resonance imaging (MRI) study of the brain and orbits with gadolinium
contrast provides confirmation of the diagnosis in most cases and also provides an assessment of the risk
of subsequent multiple sclerosis (MS).
Further diagnostic testing is directed toward excluding other causes of visual loss in atypical cases. (See
'Differential diagnosis' above.)
Magnetic resonance imaging — An MRI study of the brain and orbits with gadolinium contrast provides
confirmation of the diagnosis of acute demyelinating optic neuritis and important prognostic information
regarding the risk of developing MS.

Only for Study purpose / Only Notes / AMC MCQ


Only Study Notes / Only for Study purposes / Personal notes/ AMC MCQ

Innovations in MRI technology (eg, short tau inversion recovery [STIR], fast spin echo [FSE], and fluid-
attenuated inversion recovery with fat suppression techniques [FLAIR], diffusion tensor imaging [DTI]) have
improved imaging of the optic nerve [33-35]. Optic nerve inflammation can be demonstrated in
approximately 95 percent of patients with optic neuritis with gadolinium contrast-enhanced MRI of the brain
and orbits (image 1) [6,36-38]. The longitudinal extent of nerve involvement as seen on MRI correlates with
visual impairment at presentation and with visual prognosis [31,36,38]. Gadolinium enhancement persists
for a mean of 30 days since onset [31]. The signal abnormality in the nerve can still be seen after recovery
of vision, and is also present in as many as 60 percent of patients with MS who do not have a clinical
history of optic neuritis [31,39-41].
The brain MRI often shows white matter abnormalities characteristic of MS (image 2). Typical lesions are
ovoid, periventricular, and larger than 3 mm (see "Evaluation and diagnosis of multiple sclerosis in adults",
section on 'Lesion characteristics'). The reported prevalence of white matter abnormalities varies
substantially among patients with optic neuritis (23 to 75 percent) [42]. In the ONTT, almost 40 percent of
patients had MRI lesions, but this trial represents a selected patient group [7]. Small case series of
unselected patients have noted a higher coincidence of MRI brain lesions [31,43,44]. Individuals with white
matter abnormalities are at a higher risk of developing MS. (See "Optic neuritis: Prognosis and treatment".)
The yield of spinal cord imaging is low in unselected patients. Among 115 patients presenting with optic
neuritis, MRI abnormalities in the spinal cord were seen in only four patients with a normal brain MRI [42].
Lumbar puncture — Lumbar puncture is not an essential diagnostic test in optic neuritis but should be
considered in atypical cases (eg, those with bilateral presentation, <15 years in age, or symptoms
suggesting infection) [45,46]. (See "Optic neuropathies", section on 'Infections'.)
Approximately 60 to 80 percent of patients with acute optic neuritis have nonspecific abnormalities in the
cerebrospinal fluid (CSF), including lymphocytes (10 to 100) and elevated protein [43].
Other CSF findings in optic neuritis can include [47]:
●Myelin basic protein in approximately 20 percent
●Immunoglobulin G (IgG) synthesis in 20 to 36 percent
●Oligoclonal bands (OCB) in 56 to 69 percent
The presence of OCB implies a higher risk of developing MS. However, since OCB are also associated
with white matter lesions on brain MRI, their presence is not clearly of independent prognostic importance
[47]. (See "Optic neuritis: Prognosis and treatment".)
Other testing — When there are relevant clues to an alternative diagnosis (table 2), measurement of the
erythrocyte sedimentation rate, antinuclear antibodies, and angiotensin converting enzyme levels and
serologic and CSF tests for Lyme disease and syphilis should be obtained [45,46]. (See "Optic
neuropathies".)
Fluorescein angiography — Fluorescein angiography is not routinely performed in the evaluation of optic
neuritis and is often normal. Up to 25 percent demonstrate either dye leakage or perivenous sheathing [12].
These findings may identify patients at somewhat higher risk for developing MS.
Visual evoked response — A delay in the P100 of the visual evoked response (VER) is the
electrophysiologic manifestation of slowed conduction in the optic nerve as a result of axonal demyelination
[48]. This test is not usually helpful in the diagnosis of acute optic neuritis, unless there is a suspicion that
the visual loss is functional.
Abnormalities in the VER can persist after recovery of full vision. At one year, 80 to 90 percent will be
abnormal; 35 percent will return to normal at two years [28,31,32]. The VER is often employed to find
evidence of previous, asymptomatic episodes of optic neuritis, but the sensitivity and specificity are
imperfect [1].
The multifocal VER is a technical advance that appears to be more sensitive and specific for identifying
optic neuritis, but this technology is not generally available [1,49].

Optical coherence tomography — Optical coherence tomography (OCT) measures the thickness in the
retinal nerve fiber layer and detects thinning in most (85 percent) of patients with optic neuritis [48,50-53].
These abnormalities are also common in patients with MS who do not have a clinical history of optic

Only for Study purpose / Only Notes / AMC MCQ


Only Study Notes / Only for Study purposes / Personal notes/ AMC MCQ

neuritis [54]. While lower values correlate with impaired visual outcome, the utility of OCT as a prognostic
tool is limited in that abnormal values do not show up until early swelling disappears. In one study, OCT
was less sensitive than VER in detecting subclinical optic neuritis [55].
A number of studies have found that a greater severity of optic nerve injury seen on OCT suggests
neuromyelitis optica (NMO) rather than optic neuritis associated with MS [56-58].
Antibody testing — Serum NMO antibody testing is suggested for individuals with recurrent optic neuritis,
particularly if the MRI brain is negative for any abnormal T2/FLAIR lesions outside of the affected optic
nerve(s) [59].
Patients with recurrent optic neuritis appear to be particularly at risk for NMO or Devic disease. This is
particularly true for patients with a normal brain MRI and those with optic neuritis events in rapid succession
or with a presentation of severe vision loss [60]. In one series of 51 patients with either severe or recurrent
optic neuritis, six patients were seropositive for the aquaporin-4-specific serum autoantibody, a sensitive
biomarker for NMO, while 10 patients were seropositive for antibodies to myelin-oligodendrocyte
glycoprotein (MOG), which has also been associated with NMO [61-64]. In other studies, seropositivity for
the aquaporin-4-specific serum autoantibody was predictive of subsequent NMO among patients with
recurrent optic neuritis [65,66]. MOG antibody testing is recommended for patients with recurrent optic
neuritis who test negative for aquaporin-4-specific serum autoantibody.

101- 10-day old baby thriving well, healthy baby had yellowish discharge on the right only, cause
1.nasolacrimal duct obstruction: Conjunctivitis in a newborn may be caused by a blocked tear duct, healthy
baby, unilateral
2.gonorrhoea: The eye infection is characterized by a profuse purulent discharge present within 12 hours
of inoculation (iv cephalosporins and local sulfacetamide)
3.chlamydia : The incubation period for C. trachomatis conjunctivitis is 5 to 14 days after delivery.
Presentation before five days is unusual, but has been reported to occur earlier in infants born to mothers
with premature rupture of the membranes – lack of systemic symptoms.
4. chemical
• The most frequent clinical manifestation of C. trachomatis infection in the newborn is conjunctivitis.
The condition is sometimes referred to as inclusion conjunctivitis of the newborn (ICN) or inclusion
blennorrhea.

Twelve-day-old with five-day history of progressive lid swelling


and discharge typical of chlamydial conjunctivitis.
Choice of antibiotic — Oral erythromycin is our preferred
treatment for neonatal C. trachomatis infections, including both
conjunctivitis and pneumonia.
Data on the effectiveness of azithromycin in treating C.
trachomatis infections in infants are limited.
Risk of pyloric stenosis — Both erythromycin and azithromycin
are associated with increased risk of infantile hypertrophic
pyloric stenosis (IHPS), particularly in infants younger than two
weeks

Only for Study purpose / Only Notes / AMC MCQ


Only Study Notes / Only for Study purposes / Personal notes/ AMC MCQ

Congenital nasolacrimal duct (NLD) obstruction


(dacryostenosis) occurs in approximately 6 percent of
newborns and is the most common cause of persistent tearing and ocular discharge in infants and young
children.
Acute dacryocystitis − If left untreated, dacryocystoceles
may become infected [27,29]. The risk of acute
dacryocystitis is higher in infants with dacryocystoceles
compared with those with simple nasolacrimal duct (NLD)
obstruction because of the greater degree of tear pool
stasis.
Clinical features — Dacryocystoceles usually are noted at
or shortly after birth. A bluish swelling of the skin overlying
the lacrimal sac and superior displacement of the medial
canthal tendon are the typical findings (picture 1). The
diagnosis is made clinically and further work-up generally is
not necessary, although neuroimaging (image 1) can
confirm the diagnosis.

Only for Study purpose / Only Notes / AMC MCQ


Only Study Notes / Only for Study purposes / Personal notes/ AMC MCQ

102- Eye pictures with yellow crust at lower eye lid margin. Patient present with itchiness treatment
1. Chloramphenicol eye drop
2. Frequently wash lower eyelid
Blepharitis
Blepharitis is a clinical diagnosis based on characteristic findings of redness and irritation of the eyelid
margin associated with crusting or flakes on the lashes or lid margins.

Only for Study purpose / Only Notes / AMC MCQ


Only Study Notes / Only for Study purposes / Personal notes/ AMC MCQ

ANTERIOR POSTERIOR

Posterior blepharitis – Posterior blepharitis, the more common type, is characterized by inflammation of
the inner portion of the eyelid at the level of the meibomian glands
Anterior blepharitis – Anterior blepharitis is less common than posterior blepharitis and characterized by
inflammation at the base of the eyelashes

General approach — Good lid hygiene is the mainstay of treatment for all forms of blepharitis. The goal is
to alleviate symptoms and to develop a maintenance regimen to prevent or minimize future exacerbations
Patients with mild to moderate symptoms can generally be managed with symptomatic measures, including
warm compresses, lid massage, lid washing, and artificial tears. Patients with severe or refractory
symptoms may require additional therapies such as topical or oral antibiotics, topical glucocorticoids,
or topical cyclosporine.
The management of contact (allergic) blepharitis consists of eliminating use of the offending agent (eg,
cosmetics). Patients who use cosmetics should be vigilant about removing their makeup at night, cleaning
applicators
Blepharitis associated with Demodex species infestation can be treated with oral ivermectin (200 mcg/kg in
a single dose and repeated once in one week) or topical tea-tree oil eyelid scrubs (administered weekly for
six weeks) and tea-tree shampoo (applied daily for six weeks)
Mild to moderate symptoms — For patients with mild to moderate symptoms, management consists of
warm compresses, lid massage, lid washing, and artificial tears.
Omega-3 fatty acids — Clinical trials of oral omega-3 fatty acid supplementation for treatment of
meibomian gland dysfunction, posterior blepharitis, and dry eye have shown mixed results
Severe or refractory symptoms — For patients who do not respond to the symptomatic measures
described above and for those with severe symptoms, we suggest topical or oral antibiotic therapy.
Because of the potential for systemic side effects with oral drugs, topical therapy is usually tried first.
(Topical ATB and if very severe oral)
Topical glucocorticoids — Topical glucocorticoids are generally reserved for patients with blepharitis that
is unresponsive to other therapies
Topical cyclosporine — Topical cyclosporine should be reserved for patients with blepharitis who do not
respond to standard therapies and should be prescribed by an ophthalmologis

Symptoms — Patients with blepharitis generally present with chronic recurrent symptoms, which may vary
over time, involving both eyes. These include:
●Red, swollen, or itchy eyelids
●Gritty or burning sensation
●“Pink eyes”
●Excessive tearing (which can paradoxically be a sign of dry eye)
●Crusting or matting of eyelashes in the morning
●Flaking or scaling of the eyelid skin
●Light sensitivity

Only for Study purpose / Only Notes / AMC MCQ


Only Study Notes / Only for Study purposes / Personal notes/ AMC MCQ

●Blurred vision (transient in nature; usually improves with blinking)


Eyelids — The eyelid edges in patients with blepharitis often appear pink or irritated (picture 1 and picture
2). Crusting of the lashes or lid margins may also be visible.
Patients with anterior blepharitis typically have adherent material around their eyelashes. In the seborrheic
variant of anterior blepharitis, there are often greasy-appearing flakes; whereas in the staphylococcal
variant, a hard cylindrical crust develops around the eyelash (called a "collarette") [29]. In posterior
blepharitis, it is common to see enlargement of the meibomian gland openings and plugging with thickened,
waxy secretions appearing as white or yellow mounds at the gland opening.

103-

Ans: B
Three pairs of extraocular muscles move each eye in three
directions: vertically (superior and inferior), horizontally
(medial and lateral), and torsionally (intorsion when the eye
rotates toward the patient's nose and extorsion when the eye
rotates toward the patient's shoulder). The following muscles
are responsible for these movements.
• The superior rectus and inferior oblique muscles are
responsible for upward vertical movements. The
superior rectus acts in all fields of gaze and the
inferior oblique on medial gaze.
• The inferior rectus and superior oblique muscles are responsible for downward vertical movement.
The inferior rectus acts in all fields of gaze, and the superior oblique on medial gaze.
• The lateral rectus is responsible for abduction.
• The medial rectus is responsible for adduction.
• The superior oblique is responsible for intorsion.
• The inferior oblique is responsible for extorsion.
The superior oblique muscle is innervated by cranial nerve IV, the lateral rectus muscle by cranial nerve VI,
and all others by cranial nerve III.
Vertical diplopia in primary gaze suggests underactivity of the right or left inferior rectus, superior rectus,
inferior oblique, or superior oblique.
If vertical separation is worse on right gaze, the right superior or inferior rectus or the left inferior or
superior oblique may be underactive;
Horizontal diplopia — Horizontal diplopia is usually due to disease processes that affect the medial or
lateral rectus muscles or the innervation of these muscle

Only for Study purpose / Only Notes / AMC MCQ


Only Study Notes / Only for Study purposes / Personal notes/ AMC MCQ

Third cranial nerve palsy — Third cranial nerve (oculomotor) palsies may cause both vertical and
horizontal diplopia. Patients
with a complete unilateral
third nerve palsy have ptosis,
a large unreactive pupil, and
paralysis of adduction,
elevation, and depression; the
eye rests in a position of
abduction, slight depression,
and intorsion [2]. The
asymmetry of pupil size is
greater in the light than in the
dark.

Fourth cranial nerve palsy — Cranial nerve IV (trochlear) palsy is a common cause of vertical diplopia.
Congenital fourth nerve palsy may present at any age as most patients are able to compensate for the
double vision. Most acquired fourth nerve palsies are either traumatic or microvascular in etiology.

Sixth cranial nerve palsy


refers to dysfunction of the
sixth cranial nerve
(abducens nerve). This is
also known as lateral rectus
palsy and abducens nerve
palsy. It is the most
common ocular cranial

Sixth nerve palsy — Patients with sixth nerve palsies typically complain of horizontal diplopia. The diplopia
is binocular (ie, it requires both eyes to be open) and worsens with horizontal gaze toward the paretic
(weak) lateral rectus muscle. More mildly affected individuals may complain of blurred vision, difficulty

Only for Study purpose / Only Notes / AMC MCQ


Only Study Notes / Only for Study purposes / Personal notes/ AMC MCQ

focusing, and dizziness rather than overt diplopia.nerve palsy to occur in isolation.An ipsilateral horizontal

gaze palsy (failure of both eyes to deviate in one direction) rather than an isolated abduction deficit occurs
with lesions involving the abducens nucleus in the pons because of involvement of the interneurons of the
medial longitudinal fasciculus. These
interneurons control contralateral medial
rectus function (adduction) during attempted
ipsilateral horizontal gaze
An ipsilateral facial nerve palsy usually
also occurs with a pontine lesion because of
the close proximity of the facial and
abducens nuclei.
Other brainstem signs (eg, hemiparesis,
hemisensory loss, central Horner syndrome)
may accompany a sixth nerve nucleus lesion
with more extensive brainstem pathologies.
●Bilateral sixth nerve palsies suggest a
lesion in the subarachnoid space (eg,
meningitis). A sixth nerve palsy can occur in
these patients as a nonlocalizing sign of
increased intracranial pressure (ICP), due to
traction of the sixth nerve. The sixth nerve is
particularly susceptible to this phenomenon
because of its long course within the
subarachnoid space.

Only for Study purpose / Only Notes / AMC MCQ


Only Study Notes / Only for Study purposes / Personal notes/ AMC MCQ

104- Patient presented with diplopia for 1 day. On examination there was diplopia on the right eye
whenlooking towards the right side. his glucose is normal. ECG has occasional ventricular ectopic. What’s
the most appropriate management?
A. Warfarin-
B. Perindopril
c. Metformin
d. Observation
• Lat rectus of the right side so VI – 6th Nerve Palsy ONLY
We answer according to the clue and available information in the question....no history of trauma or
any other associated disease only information is provided that he has PVC...look at the causes of
PVC are carcinogenic or stroke-related ischemia or drugs
Premature ventricular complex (PVCs) detected from long-term ECG recordings have been associated with
an increased risk of ischemic stroke.
Premature ventricular complex (PVCs) are mostly asymptomatic irregular heart rhythms commonly seen on
electrocardiograms (ECGs) of the middle-aged and elderly

The causes of abducens (cranial nerve VI)


palsy include:1
• microvascular (8–36%)
• neoplastic (39–45% in series of children)
• traumatic (5–20%).
The most common cause in those aged
>50 years is microvascular ischaemia, but
it is important to consider this palsy as a
false localising sign for raised intracranial
pressure

Spontaneous recovery is common in patients with unilateral, isolated, nontraumatic sixth nerve palsy

Only for Study purpose / Only Notes / AMC MCQ


Only Study Notes / Only for Study purposes / Personal notes/ AMC MCQ

Causes of sixth nerve (abducens) palsy include a variety of processes that can affect the nerve anywhere
between the sixth nerve nucleus in the pons, or its course through the subarachnoid space along the
petrous apex, or in the cavernous sinus and orbit.
Patients with sixth nerve palsies
complain of binocular horizontal
diplopia that worsens with gaze toward
the paretic lateral rectus muscle
Examination reveals a primary position
esotropia worse in gaze toward the
paretic muscle (lateral incomitance)
and an ipsilateral abduction deficit.
Other manifestations depend upon the
location of the lesion along the course
of the nerve
An MRI of the brain has an
important role in evaluating a
patient with an abducens (cranial
nerve VI) palsy. Ophthalmology or
neuro-ophthalmology consultation
is recommended for further
assessment and management.
Preferred modality — Magnetic
resonance imaging (MRI) of the brain
and orbits with gadolinium is the
preferred modality because of superior
capability of imaging of the posterior
fossa [32]. Contrast-enhanced
computed tomography (CT) is an
alternative when MRI cannot be
performed.
Most isolated sixth nerve palsies will
recover spontaneously. Treatment
modalities for those with persistent
disability may include patching, prism
therapy, strabismus surgery, and/or
botulinum toxin. The goal is to
maximize visual function, including
alignment.
The evaluation of patients with sixth
nerve palsy depends upon whether the palsy is isolated or accompanied by other neurologic abnormalities.

For How long can you observe with this patient? Some suggest observation without neuroimaging unless
there is no spontaneous resolution by three to six months.2–8 The argument is that microvascular infarction
is a common cause for this category of patients and it typically resolves spontaneously over 10–12 weeks.

Only for Study purpose / Only Notes / AMC MCQ


Only Study Notes / Only for Study purposes / Personal notes/ AMC MCQ

105- a pt presented with c/o dry eyes. He has history of seasonal allergic conjunctivitis c/o dry mouth as
well as positive anti Ro and La antibodies what is ur next step in her management?
1. Hydroxychloroquine
2. Artificial tear drops
3. Methotrexate
4. Topical prednisolone
Sjögren's syndrome (SS) is a chronic autoimmune
inflammatory disorder characterized by diminished
lacrimal and salivary gland function with resultant
dryness of the eyes and mouth [1,2]. In addition, a
variety of other disease manifestations affecting
multiple organs and organ systems may occur, and
the clinical features of SS can be divided into the
two broad categories of exocrine glandular features
and extraglandular features [3]. Disease severity
can vary over a wide range.
Diagnostic criteria — The diagnosis of Sjögren's
syndrome (SS) can be made in individuals with an
objective finding of ocular and/or oral dryness for whom
there is substantive evidence of an underlying
autoimmune basis for the exocrine glandular dysfunction.
SS should be suspected in individuals with persistent
symptoms of dry eyes and/or mouth, parotid gland
enlargement, an unexplained increase in dental caries, or
abnormal results of specific serologic tests (eg, anti-
Ro/SSA antibodies with or without anti-La/SSB
antibodies, rheumatoid.
Sjogren eye disease may present with keratoconjunctivitis
sicca = dry conjunctiva and cornea

106- Another pic of red painful eye other examination


normal no history of any discharge no history of itching
and crusting c/o lower back pain asking
1) Glaucoma
2) Uveitis
3) Episcleritis
4) Scleritis
Low back pain and neck pain — Almost all patients
with axSpA report back pain, which frequently but not
invariably has characteristics suggesting an
inflammatory etiology. Several criteria sets are available
that define inflammatory back pain, which all perform
similarly and can be used in daily practice

Only for Study purpose / Only Notes / AMC MCQ


Only Study Notes / Only for Study purposes / Personal notes/ AMC MCQ

Only for Study purpose / Only Notes / AMC MCQ


Only Study Notes / Only for Study purposes / Personal notes/ AMC MCQ

What is the test we do to check the eye dryness?


Schirmer rest by placing a paper near ur eyelids and check
how much water or tear drops is absorbed.

107- builder 48 yr, weakness in left side of body for few


minutes. Past same episode for 5 times. Advice after giving
aspirin and discharge with follow up by local doctor
1) More antiplatelet
2) Can’t drive 6months.
3) Can’t go to work without local doctor’s
permission
4) No strenuous work

TIA - Driving
after stroke or
transient ischaemic attack
All patients are unfit to drive after a stroke or TIA.

After a stroke, a private vehicle driver cannot drive for at


least 4 weeks, and a commercial vehicle driver cannot
drive for at least 3 month. Before starting to drive again,
the patient must be assessed for residual impairments
that could affect the functions needed to drive safely.

No B: After a TIA, it is advised that private drivers do not


drive for 2 weeks and commercial drivers do not drive
for 4 weeks

• Thromboembolic events
o DVT = 2wks (private and commercial)
o PE = 6wks (private and commercial)
• Cardiovascular events
o MI = 2wks (private) 4wks (commercial)
o PCI = 2days (private) 4wks (commercial)
o Coronary artery bypass graft = 4wks (private)
3mts (commercial)
o Cardiac arrest = 6mts (private and
commercial)
o Cardiac pacemaker insertion = 2wks (private)
4wks (commercial)
o Syncope (CV origin) = 4wks (private) 3mts
(commercial)
o Aneurysm repair = 4wks (private) 3mts (commercial)
o Valvular replacement = 4wks (private) 3mts (commercial)
o Persons with Implantable Cardioverter Defibrillator are NOT ELIGIBLE TO HOLD A COMMERCIAL
VEHICLE LICENSE
No A: For acute treatment aspirin 300 mg orally or via nasogastric tube or rectally, on the first day. Reduce
dose to 100 mg daily on the second day and continue daily therapy indefinitely
https://tgldcdp.tg.org.au/viewTopic?topicfile=stroke-and-transient-ischaemic-attack&sectionId=nrg5-c43-
s3#toc_d1e460

Only for Study purpose / Only Notes / AMC MCQ


Only Study Notes / Only for Study purposes / Personal notes/ AMC MCQ

The combination of dipyridamole and aspirin is marginally more effective than aspirin alone, but has more
adverse effects. It should be considered in patients who have recurrent cerebral ischaemic events despite
aspirin therapy. The most common adverse effect of dipyridamole+aspirin is headache, and starting treatment
with smaller doses may avoid
it.
In patients without atrial
fibrillation or another source of
cardiogenic embolism, the use
of warfarin is not
recommended. It causes harm
due to major bleeding
complications

108- A 22 yr pt came with numbness, heaviness of the rt hand & leg together with left homonymous hemianopia
.Her hand & foot state resolved after 2 weeks but her visual field defect is still present 6 wklater.wht would u do?
1) Cannot drive for 2wks
2) Cant drive for 2 months
3) Cant drive for 6 months
4) Cant drive forever
5) refer to occupational therapist
After a stroke, a private vehicle driver cannot drive for at least 4 weeks

109- Patient with DM and HTN crashed into a parked car O/E peripheral vision loss next step?
1) Check IOP
2) CT head rule our head trauma
- Cerebrovascular accident (CVA)

A common cause of loss of peripheral vision (also called a peripheral field defect) is optic nerve
damage from glaucoma. Eye "strokes" (occlusions) that block normal blood flow to the eye's internal
structures, including the optic nerve, also can lead to loss of peripheral vision.
Basic causes of peripheral vision loss include:

Only for Study purpose / Only Notes / AMC MCQ


Only Study Notes / Only for Study purposes / Personal notes/ AMC MCQ

• Glaucoma
• Retinitis pigmentosa
• Eye strokes or occlusions
• Detached retina
• Brain damage from stroke, disease or injury
• Neurological damage such as from optic neuritis
• Compressed optic nerve head (papilledema)
• Concussions (head injuries)

If the blockage of
aqueous at the
beginning, it's
closed or at
anywhere in the
pathway it's open.

Only for Study purpose / Only Notes / AMC MCQ


Only Study Notes / Only for Study purposes / Personal notes/ AMC MCQ

110-

Initial primary care evaluation — In the primary care setting, a complaint of visual disturbance should
prompt a history that includes the rate of vision loss, whether one or both eyes are involved, and whether
the vision loss is for distance vision, near vision, or both. Patients with an acute distortion or loss of central
vision may represent wet age-related macular degeneration (AMD).

Ophthalmologic evaluation — AMD is a clinical diagnosis based upon the presence of characteristic
findings on dilated eye examination using a slit-lamp instrument (biomicroscopy): grid pattern amsler chart

Patients with drusen and mild pigmentary changes alone may have a visual acuity within the normal
range. Patients with late AMD usually have reduced visual acuity in the affected eye. An area of
central distortion or scotoma may be mapped out by the patient on an Amsler grid. Examination of
the fundus, preferably using a stereoscopic viewing method (slit-lamp biomicroscopy), shows the
presence of drusen, pigmentary, exudative, haemorrhagic or atrophic changes affecting the macula.
Non-stereoscopic examination (direct ophthalmoscopy or fundal photography) may miss retinal
thickening or elevation due to neovascular AMD.

Fluorescein angiography is used to confirm the presence and nature of neovascular AMD. Sodium
fluorescein dye is injected intravenously. Fundus photographs are then taken through a barrier filter
as the fluorescent dye fills the choroidal and retinal circulations. CNV can be defined either by its
anatomical location relative to the centre of the fovea or by its filling characteristics. Treatment is
often determined based on an understanding of these two angiographic findings. ‘Classic’ CNV
membranes fill with dye in the arterial phase, are well defined and leak fluorescein dye beyond the
borders in later shots. Lesions that leak in a less clearly demarcated pattern in later shots, but not in
the early shots, are termed ‘occult’ lesions. CNV may be predominately classic (greater than 50%
classic component) or minimally classic (50% or less classic component). Currently, the
determination of the type of neovascular AMD is essential in order to assess the suitability for
treatment modalities such as laser photocoagulation and photodynamic therapy (PDT), but
pharmacologic therapies (anti-VEGF agents) are effective in treating all subtypes of CNV.

Only for Study purpose / Only Notes / AMC MCQ


Only Study Notes / Only for Study purposes / Personal notes/ AMC MCQ

110- fundoscopy of MD with scenario of 72 yrs old man come with gradual vision loss over last 12 months
,but he has no other complains without it.which of the following is the most probable diagnosis??
a.MD
b.CRVO
C.CRAO
papilledema
e. Optic atrophy

MACULAR DEGENERATION
􏰄 Age related macular degeneration
􏰄wet = neovacular / exudative
􏰄 Dry = non exudative / drusen
􏰄 Central visual loss
􏰄 Distorted vision (line of words bend
while reading)
􏰄 Commonly affects elderly
􏰄 Leads to blindness- progressive
􏰄 Screening: Amslar grid (can’t see central dot)
􏰄 Best Inv: Fluorescein angiography

TREATMENT MD: https://www.uptodate.com/contents/age-related-macular-degeneration-treatment-and-


prevention?search=macular%20degeneration&source=search_result&selectedTitle=1~68&usage_type=def
ault&display_rank=1#H2
•Dry AMD (atrophic or nonexudative):
-We suggest that patients with extensive intermediate size drusen, at least one large drusen, or noncentral
geographic atrophy in one or both eyes be treated with a daily oral eye vitamin supplement

Only for Study purpose / Only Notes / AMC MCQ


Only Study Notes / Only for Study purposes / Personal notes/ AMC MCQ

(Grade 2B). These should be consistent with the Age-Related Eye Disease Study 2 (AREDS2) formulation
(containing vitamin C 500 mg, vitamin E 400 international units, lutein 10 mg, zeaxanthin 2 mg, zinc 80 mg
[as zinc oxide], and copper 2 mg [as cupric oxide]). Alternatively, patients who are not smokers or former
smokers may use the standard AREDS formulation, which contains beta-carotene rather than lutein or
zeaxanthin. (See 'Antioxidant vitamins and zinc' above.)
-Patients may also wish to consider entering clinical trials of therapy aimed at slowing progression of AMD.
In the United States, information regarding available trials is available at the government’s clinical
trials database.
-Smoking may increase the risk of progression to advanced AMD. Thus, patients with AMD should be
encouraged to quit smoking. (See 'Smoking' above and "Overview of smoking cessation management in
adults".)
•Wet AMD (neovascular or exudative):
-For most patients with AMD and neovascularization, we recommend treatment with an intravitreal vascular
endothelial growth factor (VEGF) inhibitor (eg, bevacizumab, ranibizumab, aflibercept) (Grade 1B).
Photodynamic therapy is an alternative for patients who cannot be treated with an intravitreal VEGF
inhibitor and for patients with chronic exudative lesions who have preserved vision in one eye and are
unlikely to achieve reading vision in the second eye. (See 'VEGF inhibitors and inhibitor-like drugs' above
and 'Photodynamic therapy' above.)
-We also suggest that patients with wet AMD in one or both eyes be treated with daily oral supplements
(Grade 2B). These should be consistent with the AREDS2 formulation (containing vitamin C 500
mg, vitamin E 400 international units, lutein 10 mg, zeaxanthin 2 mg, zinc 80 mg [as zinc oxide], and
copper 2 mg
[as cupric
oxide]).

Alternatively, patients who are not smokers or former smokers may use the standard AREDS formulation,
which contains beta-carotene rather than lutein or zeaxanthin. (See 'Antioxidant vitamins and zinc' above.)
●Counseling patients to quit smoking is important part of preventing AMD and also may reduce the risk of
other smoking-related disorders. (See 'Smoking' above and "Overview of smoking cessation management
in adults".)
●The available evidence does not support the use of antioxidants to prevent or delay the onset of AMD in
unaffected individuals. However, given other apparent health benefits, we encourage a diet that includes
fruits, green leafy vegetables, fish, and nuts. We also encourage physical activity. (See 'Prevention' above.)

Only for Study purpose / Only Notes / AMC MCQ


Only Study Notes / Only for Study purposes / Personal notes/ AMC MCQ

Papiledema :

Optic disc atrophy:

Only for Study purpose / Only Notes / AMC MCQ


Only Study Notes / Only for Study purposes / Personal notes/ AMC MCQ

Only for Study purpose / Only Notes / AMC MCQ


Only Study Notes / Only for Study purposes / Personal notes/ AMC MCQ

Only for Study purpose / Only Notes / AMC MCQ


Only Study Notes / Only for Study purposes / Personal notes/ AMC MCQ

111- Lady was taking hypertensive medicine for high BP and Metformin for Diabetes Mellitus. 2 weeks ago
she started to fell gradual loss of her vision. No BP & lab values given. Fundoscopy was done and the
picture is as follows: What is the cause of her problem? Almost the same picture.
1) CRVO
2)CRAO
3) Retinal detachment
4) Hypertensive retinopathy
5) Diabetes Retinopathy

Only for Study purpose / Only Notes / AMC MCQ


Only Study Notes / Only for Study purposes / Personal notes/ AMC MCQ

Only for Study purpose / Only Notes / AMC MCQ


Only Study Notes / Only for Study purposes / Personal notes/ AMC MCQ

Only for Study purpose / Only Notes / AMC MCQ


Only Study Notes / Only for Study purposes / Personal notes/ AMC MCQ

No - retinal detachment: in JM 899 retinal detachment is less


than 7 days loss of vision

No- diabetic retinopathy- Micro-aneurysm,


hard exudates, IRMA, neovasc..

Diabetic retinopathy can lead to other serious eye conditions:


• Diabetic macular edema (DME). Over time, about half of people with diabetic retinopathy will
develop DME. DME happens when blood vessels in the retina leak fluid, causing swelling in the
macula (a part of the retina). If you have DME, your vision will become blurry because of the extra
fluid in your macula.
• Neovascular glaucoma. Diabetic retinopathy can cause abnormal blood vessels to grow out of the
retina and block fluid from draining out of the eye. This causes a type of glaucom
Diabetic retinopathy (DR) is one of the most important causes of visual loss worldwide and is the principal
cause of impaired vision in patients between 25 and 74 years of age. Visual loss from DR may be
secondary to macular edema (ME; retinal thickening and edema involving the macula), hemorrhage from
new vessels, retinal detachment, or neovascular glaucoma.

Macular edema — ME can occur at any stage of DR. It is defined as retinal thickening and edema
involving the macula, and it may be visualized by specialized fundus exam with stereoscopic
viewing, fluorescein angiography, and, most directly, by optical coherence tomography (OCT; a
noninvasive, low-energy laser imaging technology) (image 1).

Clinically significant macular edema (CSME) is defined as retinal thickening within 500 microns of
the fovea, hard
exudates within 500
microns of the fovea if
associated with
adjacent retinal
thickening, or one or
more areas of retinal

Only for Study purpose / Only Notes / AMC MCQ


Only Study Notes / Only for Study purposes / Personal notes/ AMC MCQ

thickening at least 1500 microns in diameter that is within one disc diameter (1500 microns) of the
fovea
(A) Optical coherence tomography (OCT) of diabetic macular edema. There are numerous large cysts
visible within the macula (arrows), and the retinal thickness is increased.
(B) OCT of normal macula (for comparison) showing typical foveal contour.

Nonproliferative retinopathy
Vision symptoms are caused by macular edema or
macular ischemia. However, patients may not have
vision loss even with advanced retinopathy. The first
signs of nonproliferative retinopathy are
• Capillary microaneurysms

• Dot and blot retinal hemorrhages


• Hard exudates
• Cotton-wool spots (soft exudates)
Diabetic Retinopathy (Nonproliferative)
Hard exudates are discrete, yellow particles within the
retina. When present, they suggest chronic edema. Cotton-wool spots are areas of microinfarction of the
retinal nerve fiber layer that lead to retinal opacification; they are fuzzy-edged and white and obscure
underlying vessels.
Signs in later stages are
• Macular edema (seen on slit-lamp biomicroscopy as elevation and blurring of retinal layers)

• Venous dilation and intraretinal microvascular abnormalities


• Proliferative diabetic retinopathy with high-risk characteristics of vitreous hemorrhage, extensive
preretinal neovascularization, or anterior segment neovascularization/neovascular glaucoma should
be treated with panretinal laser photocoagulation. Recent studies have also supported the use of
intravitreal anti-VEGF drugs in the treatment of proliferative diabetic retinopathy (2). These
treatments significantly reduce the risk of severe vision loss.
Treatment of Hypertensive Retinopathy?

Moderate to severe hypertensive retinopathy − Moderate to severe hypertensive retinopathy,


corresponding to grades III and IV hypertensive retinopathy, is characterized by retinal hemorrhages,
exudates, and papilledema
Goal of therapy — The initial aim of treatment in patients with moderate to severe hypertensive
retinopathy and/or hypertensive encephalopathy is to rapidly lower the mean arterial pressure by
approximately 10 to 15 percent in the first hour, and by no more than 25 percent compared with
baseline by the end of the first day of treatment.
Once BP is controlled, the patient should be switched to oral therapy, with the systolic pressure being
gradually reduced to <130 mmHg and the diastolic pressure being gradually reduced to <80

Only for Study purpose / Only Notes / AMC MCQ


Only Study Notes / Only for Study purposes / Personal notes/ AMC MCQ

mmHg over two to three months. The initial reduction in BP is often associated with a modest
worsening of kidney function; this change, however, is typically transient as the vascular disease tends
to resolve and renal perfusion improves over one to three months [22], even among patients treated
with angiotensin-converting enzyme (ACE) inhibitors, renin inhibitors, or angiotensin II receptor blockers
(ARBs)
Screening in Australia: RACGP

Who to screen and how often? The general recommendation is that GPs need to ensure that their patients
with diabetes have been appropriately screened with a dilated fundus and a trained examiner every 2
years.6 However, many patients with diabetes will have an extra risk factor necessitating yearly screening
as per National Health and Medical Research Council (NHMRC) recommendations (Table 1). This can be
provided by suitably trained GPs, optometrists or where available, ophthalmologists. Some doctors remain
concerned about mydriatic drops owing to risks of acute angle closure glaucoma. However, at a rate of 1–6
per 20 000 people, this is uncommon and has not been known to occur with 0.5% or 1% tropicamide drops,

which are commonly used in Australian general practice.


For a normal patient and based on the stages non proliferative 3-6months proliferative with macular edema
please note this -4 weeks

112- your in a rural area and 4 cases of trachoma come to you then you find out extra 20 how can you treat
acute indexes?
a-hand washing

Only for Study purpose / Only Notes / AMC MCQ


Only Study Notes / Only for Study purposes / Personal notes/ AMC MCQ

b-azithromycin
c-doxycyclin
d-don’t do anything
e-penicillin

Prevention – SAFE
113- It was something like this.. 5 kids in a community found with Trachoma...20 household contacts were
related to these kids.. it was a small community of total 240 ppl.
How to manage this outbreak?
1- treat kids
2- 2- treat kids and household
3- 3- treat whole community
4- 4- prophylactic abx for household
5- 5- prophylactic abx for community
2%
For trachoma: we need to
1) Consider all the cases and contacts as we need to treat them all
2) We need to find is it is worth to treat just contacts and cases or the whole community
3) If the percentage of cases and contacts is >10% the WHO recommends treat the whole community
4) But Australia differentiates in <5, 5-20. >20
5) Only if in the questions we got a reference related to health staff able to identify contacts, and
obviously clustered of cases then if 5 -20% we just treat the households
6) For the time being we haven't seen any questions giving this reference

Only for Study purpose / Only Notes / AMC MCQ


Only Study Notes / Only for Study purposes / Personal notes/ AMC MCQ

114 – You see many trachoma cases in indigenous community. What to do to prevent trachoma
cases?
A) Give eye drops to community
Give eye drops to contact
C) Give treatment to community
D) Give treatment to contact

The WHO GET 2020 initiative is built on the implementation of the SAFE strategy for the effective prevention and
control of trachoma.

• S – Surgery for trichiasis: Surgical procedures to reduce impact of trichiasis


• A – Antibiotics: Antibiotic (azithromycin) treatment of individual active trachoma cases and to reduce the community
reservoir of infection
• F – Facial cleanliness: Promote clean faces to reduce spread of infection
• E – Environmental health – improve water access, good sanitation, waste and fly control, and reduce overcrowding
top of page

The acronym SAFE covers four public health components, and in order of public health priority are:

The ‘E’ component ‘environmental health’ covers a very broad category of potential activities.

Safe access to clean and functioning water supplies, adequate sanitation including clean linen and aired mattresses,
improved housing, reducing overcrowding and attempts to minimise fly density are all potentially important factors for
trachoma control.

The “F” component ‘facial cleanliness’ is seen as the key preventive measure that can be taken to prevent infection.
Facial cleanliness is the absence of nasal and ocular discharge.10 It requires the proper maintenance of housing,
especially washing facilities and bathrooms, and the development of household and personal hygiene skills and
behaviours. Facial cleanliness in children should be promoted by including regular face-washing as part of a holistic
personal hygiene program, which may also include tooth-brushing, hand washing and general hygiene

The “A” component ‘antibiotic distribution’ of the strategy has an important role in prevention by reducing the
duration of infection (both symptomatic and asymptomatic), thereby reducing disease transmission (see section 9)

The “S” component ‘surgery’ of the strategy involves the detection, referral and surgical management of entropion
(in-turned eye lid margin) and trichiasis (in-turned eye lashes) to prevent further corneal abrasion and the
development of corneal scarring and blindness

115- A 4-year-old boy who was recently adopted from


Ghana is brought to his pediatrician's office. His mother
notes he often scratches his right eye and also seems to
have poor vision in that eye. On exam, the right eyelid
appears similar to Figure A with eversion. Which of the
following is the best treatment for this patient?
1Topical tetracycline
2Topical vidarabine
3Topical prednisone
4Oral antihistamines

Only for Study purpose / Only Notes / AMC MCQ


Only Study Notes / Only for Study purposes / Personal notes/ AMC MCQ

Treatment involves either topical tetracycline or oral azithromycin. More severe cases with corneal scarring
and eyelid inversion may be appropriate for surgical treatment.
Mishri et al. review the diagnosis and treatment of trachoma. Occular infections with Chlamydia trachomatis
can manifest as neonatal conjunctivitis, adult conjunctivitis, or trachoma. Trachoma is caused by chronic
infection that leads to eyelid scarring and inversion, and often results in misplacement of the eyelashes
such that they consistently scratch the cornea.
Sommer et al. review challenges in giving ophthalmological care in the developing world. They note that
trachoma is the leading cause of blindness in the world, affecting roughly 40 million people and causing
blindness in nearly 8 million people. They suggest mass treatment distribution to areas with endemic
trachoma.
Figure A shows trachomatous follicles on the internal eyelid that can be seen with eyelid eversion.
Illustration A shows a Giemsa stain of Chlamydia trachomatis.
Incorrect Answers:
Answer 2: Topical vidarabine is an antiviral medication that is used for treatment of ocular herpes simplex
infection.
Answer 3: Topical steroids are used as treatment for uveitis.
Answer 4: Oral antihistamines are used for treatment of allergic conjunctivitis.
Answer 5: Surgical removal is used as the treatment for pterygium

Only for Study purpose / Only Notes / AMC MCQ


Only Study Notes / Only for Study purposes / Personal notes/ AMC MCQ

116 - A 75-year-old man with a history of hypertension and diabetes presents to his primary care physician
for blurry vision and difficulty driving at night. The patient reports a gradual blurring of vision in both eyes
over the last 10 years. He states that he is retired and was not bothered by the blurriness but is now having
difficulty driving at night due to glare from oncoming traffic. The patient denies any eye pain, flashes,
floaters, tearing, redness, or periods of complete vision loss. He lives at home alone, smokes 1 pack per
day, drinks 2 alcoholic drinks every night, and eats mostly canned food. His temperature is 98.5°F (36.9°C),
blood pressure is 184/110 mmHg, pulse is 70/min, respirations are 13/min, and oxygen saturation is 98%
on room air. Cranial nerves II-XII are grossly intact and the patient has a stable gait. The patient's vision is
20/100 in both eyes. Slit-lamp exam is performed as seen in Figure A. Which of the following is the most
likely diagnosis?
1
Diabetic retinopathy
2
Macular degeneration
3
Retinal detachment
4
Senile cataracts
5
Vitamin A deficiency

No floaters flashes no redness or loss of vision cranial


nerves intact vision is 20/100 slit lamp shows cloudiness
considering age senile cataract. excluded rest since no diabetes history or sudden loss of vision in less than 7 days
retinal detachment
This patient is presenting with gradual worsening of his vision with clouding of the lens on slit-lamp exam
suggesting a diagnosis of senile cataracts.
Senile cataracts are the leading cause
of visual impairment worldwide. Senile
cataracts are due to the denaturation
of lens proteins which cause
opacification of the lens. Although it is
a natural phenomenon that occurs as
the lens ages, the rate of formation of
cataracts can be increased by
diabetes, hypertension, UV light,
steroids, trauma, and radiation
exposure. The diagnosis can be
supported with a slit-lamp exam which
will show opacification of the lens. The
definitive treatment of cataracts is lens
extraction.
Figure A shows a magnified view of the eye as seen through a slit-lamp. Through the pupil, a clouding of
the lens can be seen
Incorrect Answers:
Answer 1: Diabetic retinopathy is a common disease that leads to blurred vision and ultimately blindness.
Though this patient has diabetes, his slit-lamp exam demonstrates opacification of the lens suggesting a
diagnosis of cataracts.
Answer 2: Macular degeneration is differentiated into wet and dry. Wet is due to neovascularization and dry
is characterized by drusens between the retina and choroid. Macular degeneration leads to gradual visual
loss, decreased color vision, and an increased risk for retinal detachment. Opacification of the lens on slit-
lamp exam is not a common finding.

Only for Study purpose / Only Notes / AMC MCQ


Only Study Notes / Only for Study purposes / Personal notes/ AMC MCQ

Answer 3: Retinal detachment typically presents unilaterally with flashes of light and floaters followed by
visual loss in a "curtain coming down" distribution.
Answer 5: Vitamin A deficiency can cause night blindness but it takes years to deplete liver stores. Vitamin
A is essential in the eye for corneal and conjunctival epithelial cell function and is involved in the
phototransduction process. However, an opacified cornea would not be seen in this condition.

117- A 38-year-old woman presents to the emergency department complaining of a “curtain” suddenly
obscuring her vision in her right eye. She reports for the past few weeks, she has experienced intermittent
episodes of seeing flashes of light as well as floaters blocking her visual field. She denies any eye pain or
irritation. She notes that since she was a child, she has relied on wearing glasses with thick lenses due to a
high degree of myopia. Which of the following images on ophthalmoscopy corresponds with this patient’s
most likely diagnosis?
A B C

D E

This is retinal detachment as per the question as the history of high degree of myopia makes
the retina thin option e characteristic curtain falling

This patient’s acute presentation of a curtain obscuring her visual field after experiencing flashes and
floaters is characteristic of retinal detachment, visualized on ophthalmoscopy as Figure E. Retinal
detachment is a vision-threatening condition in which the inner neurosensory layers of the retina are
separated from the retinal pigment epithelial layer. The classic symptoms
of retinal detachment include painless, sudden vision loss (often
described as a dark curtain or veil obscuring the visual field), flashes of
light, and floaters in the visual field. The major causes of retinal
detachment include retinal tears associated with posterior vitreous
detachment or trauma, proliferative membranes that pull on the surface
of the retina or vitreous (seen in disease processes that cause
neovascularization of the retina), and inflammatory or exudative disease

Only for Study purpose / Only Notes / AMC MCQ


Only Study Notes / Only for Study purposes / Personal notes/ AMC MCQ

processes that cause accumulation of subretinal fluid. Severe myopia (nearsightedness) is a risk factor for
retinal detachment.
Incorrect Answers:
Answer 1: Figure A shows an eye with a cataract, which would present with cloudiness and yellowing of the
lens that causes a painless, slowly progressive decrease in vision. Flashes and floaters are not
characteristic of cataracts.

Answer 2: Figure B shows an eye with central retinal artery


occlusion (CRAO), which would present with acute, painless, and
complete loss of vision in one eye. Flashes and floaters are not
characteristic of CRAO.

Answer 3: Figure C shows an eye with glaucoma, as assessment


of the optic disc shows a markedly enlarged cup-to-disc ratio
(greater than 0.3). A patient with glaucoma would present with
either insidious onset of peripheral vision loss, in the case of open-
angle glaucoma, or an acute, painful onset of blurry vision
accompanied by halos around lights, in the case of angle-closure
glaucoma.

Answer 4: Figure D shows an eye with macular degeneration, which presents with gradual vision loss and
metamorphopsia (distorted vision in which a grid of straight lines appears
wavy and parts of the grid may appear missing). Flashes and floaters are
not characteristic of CRAO.

Only for Study purpose / Only Notes / AMC MCQ


Only Study Notes / Only for Study purposes / Personal notes/ AMC MCQ

118- ld patient. Had a vehicle accident. Upon eye exam noted to have peripheral loss of vision on left only
and some cataract. Patient never noticed until now. What is the next appropriate management/invx?
a. Explain that it’s normal for old age
b. Cataract surgery
c. Urgent MRI
d. Check IOP
Check Head trauma first

Only for Study purpose / Only Notes / AMC MCQ


Only Study Notes / Only for Study purposes / Personal notes/ AMC MCQ

119- Old patient with decrease in vision


from past few months. vision s 6/18 on right
side and 6/12 on left and on correction with
pinhole is 6/9 on both sides. What is the
cause fr this condition?
1. Retinal degeneration
2. Glaucoma
3. Cataract
4. refractive error
5. presbyopia

Only for Study purpose / Only Notes / AMC MCQ


Only Study Notes / Only for Study purposes / Personal notes/ AMC MCQ

120- Most appropriate definitive


management of cataract?
a. IOL
b, Phacoemulsifiction

Only for Study purpose / Only Notes / AMC MCQ


Only Study Notes / Only for Study purposes / Personal notes/ AMC MCQ

121- A 65-year-old man, with a history of transient ischemic attack, presents to the emergency department
complaining of sudden loss of vision in his right eye. Physical exam reveals a dilated and sluggish right
pupil. On fundoscopy the right retina is pale, the fovea is dark red (Figure A). What is the proper treatment
for this patient?

1IV acetazolamide, pilocarpine, laser iridotomy


2Laser or cryotherapy to reattach the retina
3IV acetazolamide, ocular massage, carbogen therapy

Only for Study purpose / Only Notes / AMC MCQ


Only Study Notes / Only for Study purposes / Personal notes/ AMC MCQ

4Ranibizumab
5Observation, elevate head of the bed at nighttime

This is cherry red spot option 3 will be the


treatment option 1 is for closed angle glaucoma
option 2 is used for retinal detachment 4-anti vegf
macular degeneration treatment 5- used for
immediate treatment for retinal detachment

This patient has a central retinal artery occlusion (CRAO)


likely secondary to atherosclerotic disease. The treatment for
this condition can include manual massage of the globe, decompression of the anterior chamber
within first hour of onset, carbogen (95% O2, 5% CO2), and IV acetazolamide.

Retinal artery occlusions present with sudden painless vision loss in the affected eye. Fundoscopy will

demonstrate a pale retina, cherry red spot at the fovea and edema. Management for central retinal artery
occlusion is imperfect as interventions to alleviate the obstruction are too delayed. First line therapies
involve non-invasive interventions such as ocular massage (to dislodge the clot to a less impactful
location), second line therapy includes the more invasive thrombolytics.

Only for Study purpose / Only Notes / AMC MCQ


Only Study Notes / Only for Study purposes / Personal notes/ AMC MCQ

Pokhrel et al. review the management of ocular emergencies, which includes mechanical injury, acute
angle glaucoma, chemical injury, retinal detachment and CRAO. During any of these events, patients
should be brought to the emergency department or an ophthalmologist. Visual fields, acuity and ocular
movement should be investigated. Additionally, it is important to perform a pupillary and fundoscopic exam,
if possible.

Varma et al. discuss in detail the management of a CRAO. As stated above acute management can
include, massage, decompression and acetazolamide. Sublingual nitrates, IV mannitol and thrombolytics
are other options. Future management includes prevention of neovascular complications (anti-VEGF) and
treating systemic sequelae (as a CRAO indicates there is likely significant atherosclerosis and/or carotid
disease).
Figure A: Fundoscopic exam of patient's right retina.
Incorrect Answers:
Answer 1: IV acetazolamide, pilocarpine, and laser iridotomy is the treatment for acute angle glaucoma.
Acute angle closure glaucoma presents with eye pain and stiffness, loss of peripheral vision and possible
nausea and vomiting.
Answer 2: Laser or cryotherapy to reattach the retina is the treatment for retinal detachment. Retinal
detachment can present with photopsia and the "curtain coming down over eye" phenomenon.
Answer 4: Ranibizumab is one of several treatments for retinal vein occlusion. It functions through anti-
angiogenic properties. Retinal vein occlusion can present nearly the same way as an arterial occlusion.
Fundoscopy will offer definitive diagnosis.
Answer 5: Observation with elevation of the head of the bed at nighttime is conservative treatment for
vitreous hemorrhage. It can present with a painless sudden loss of vision in one eye or with "floaters".
Diabetic retinopathy is a major risk factor.

Only for Study purpose / Only Notes / AMC MCQ


Only Study Notes / Only for Study purposes / Personal notes/ AMC MCQ

122- 30’s lady decrease visual acuity for 2 years. lt eye 6/36 rt eye 6/6. RAPD (righ afferential pu[il defect)in
lt eye..pale disc. Pink optic disk in rt eye.dx?
a. Glaucoma
b.MS
c. Cataract
d. Temp arteritis
Ans: B Women rapd positive optic disc pale Answer MS

123- A 69-year-old man arrives to your clinic complaining of progressively worsening episodes of seeing
flashes of light, and small objects floating in his visual fields. He reports that he has “severe near-
sightedness,” but had not noticed any significant change in his vision clarity. Then, within the past

Only for Study purpose / Only Notes / AMC MCQ


Only Study Notes / Only for Study purposes / Personal notes/ AMC MCQ

month, the patient noticed flashes of light within the vision of his right eye, which were accompanied by
“showers of floaters.” He reports that these episodes have become more frequent. The patient's
medications include metformin, for his type II diabetes mellitus, atorvastatin, for his high cholesterol, and
daily aspirin. The patient’s temperature is 98.2°F (37.8°C), blood pressure is 130/78 mmHg, pulse is
72/min, and respirations are 14/min with an oxygen saturation of 99% O2 on room air. Physical exam is
unremarkable. Fundoscopic exam of the right eye is shown in Figure A. Which of the following would this
patient also likely experience?
1Halos around lights
2“Curtain drawn down” vision loss
3Metamorphopsia
4Scotoma
5Conjunctival injection
The patient is presenting with episodes of
flashers and floaters and a fundoscopic exam
positive for retinal “crinkling,” suggesting the
diagnosis of retinal detachment. Retinal
detachment will also be associated with “curtain
drawn down” vision loss.

Retinal detachment refers to the separation of


the layers of the retina. These patients complain
of photopsia (flashes of light) and showers of floaters (spots in the visual field), which are both associated
with posterior vitreous detachment. This may then progress to the most classic description of retinal
detachment, which is “curtain coming down” vision loss. Fundoscopic examination will reveal a crinkling
and/or grey retina and changes in vessel direction. This is a surgical emergency, treated by either laser
therapy or cryotherapy to create permanent adhesions.

Figure A shows a crinkling and grey retinal area, characteristic of retinal detachment. Illustration A is a
fundoscopy showing soft, yellow drusen in the macula region.

Incorrect Answers:
Answer 1: Halos around lights may be seen with acute narrow-angle glaucoma. Other symptoms include
pain, vision loss, rock-hard eye, red eye, and frontal headache. Fundoscopic exam shows characteristic
optic disc cupping.

Answer 3: Metamorphopsia is vision distortion, which can be seen with age-related macular degeneration
(ARMD). Dry ARMD on fundoscopic exam appears as yellow retinal drusen deposits (Illustration A). Wet
ARMD on fundoscopic exam will show neovascularization.

Answer 4: Scotoma is central vision loss, characteristic of age-related macular degeneration.

Answer 5: Conjunctival injection can be seen with many ophthalmic pathologies, such as conjunctivitis
caused by allergies, bacterial infections, or viral infections.

Macular Degeneration

Only for Study purpose / Only Notes / AMC MCQ


Only Study Notes / Only for Study purposes / Personal notes/ AMC MCQ

124- old man pt present with 2 days Hx of blurring


vision. No headache. Due to cataract we couldn't do ophthalmoscopy. Investigation?
1) LP
2) MRI
3) ESR
4) carotid artery duplex
Age?

Only for Study purpose / Only Notes / AMC MCQ


Only Study Notes / Only for Study purposes / Personal notes/ AMC MCQ

Only for Study purpose / Only Notes / AMC MCQ


Only Study Notes / Only for Study purposes / Personal notes/ AMC MCQ

125 - An 80-year-old man comes to the clinic for a follow-up visit. He has a long history of diabetes mellitus
and recently had his diabetic screening exams (including a hemoglobin A1c of 7.2%), a normal foot
examination, and normal lipid levels. Although he failed metformin and sulfonylurea therapy, the patient's
diabetes is now well-controlled with glargine and insulin lispro. The patient states that he still drives and can
see highway billboards and signs without trouble. He rarely reads, opting to spend his retirement playing
golf. During the patient's eye exam one month ago, the ophthalmologist's fundoscopic examination
revealed scattered bright yellow drusen seen in Figure A. Visual examination today is 20/40 OD (right eye)
and 20/40 OS (left eye). Two years ago, his visual acuity was 20/20 in both eyes. He states that he just has
dry eyes and needs to blink several times in order to get a perfect visual exam score. He denies any
floaters, diplopia, or blurred vision. What is the most likely diagnosis? Review Topic | Tested Concept

1. Presbyopia
2. Macular edema
3. Proliferative diabetic retinopathy
4. Nonproliferative diabetic retinopathy
5. Diabetic cataracts

This elderly man with well-controlled diabetes presents with a


worsened visual acuity on exam. In the setting of a normal
fundoscopic exam, he likely has presbyopia.
Presbyopia is an age-related change in the elasticity of the lens
zonules. The zonules connect to the lens periphery and
suspend the lens to the surrounding ciliary muscle. With near
vision, the ciliary body contracts like a sphincter causing the zonules to relax and the lens to get “rounder.”
This rounding of the lens increases its refractive ability and allows focusing of near objects. With aging and
onset of presbyopia, the lens hardens and loses its ability to round out.
Figure A shows a retina with scattered drusen.
Drusen are normal accumulations of extracellular
material that develop after age 40. If exudates were
present, abnormal vascularization and leakage
should be present, which is not obvious in this case.
Incorrect Answers:
Answer 2: Macular edema is a complication of
diabetes mellitus where multiple exudates near the
macula may be visible in the setting of complaints of
floaters or blurred vision.

Answers 3-4: Nonproliferative and proliferative


diabetic retinopathy are due to microvascular eye
disease from diabetes mellitus. Occlusions, dilations,
and increased microvascular permeability lead to
microaneurysms and leakage. Proliferative
retinopathy is the final outcome of diabetic eye
disease, when neovascularization can disrupt the
optic disc and nerve.
Answer 5: Diabetic cataracts produce lens opacity
which would limit the yield of the fundoscopic exam.

126- Patient with red eye, decrease in vision ,normal


red reflex, what is the most likely diagnosis?

Only for Study purpose / Only Notes / AMC MCQ


Only Study Notes / Only for Study purposes / Personal notes/ AMC MCQ

1. Conjunctivitis (normal vision)


2. Blepharatis (normal vision)
3. Glaucoma (alter vision and painful eye)
4. Keratitis (alter vision and painful eye)
5. Uveitis (alter vision and painful eye)

127 -Pain, redness and blurred vision , irregular pupil scenario asking DX
1. Acute glaucoma
2. Iritis
3. Acute conjunctive
4. Scleritis

Only for Study purpose / Only Notes / AMC MCQ


Only Study Notes / Only for Study purposes / Personal notes/ AMC MCQ

128- A 69-year-old male presents to your clinic with complaints of seeing “wavy lines” in the vision of his left
eye. He is not sure when it started, but does believe it has gotten progressively worse. He denies pain,
photophobia, or other ocular symptoms. His past medical history
is significant for type II diabetes mellitus and hypertension. His
current medications include metformin and lisinopril. On
examination, his visual acuity is 20/400 in the left eye and 20/70
in the right eye. His pupils are 2 mm bilaterally and equally
reactive to light with accommodation. There are no extraocular
motor deficits. A fundoscopic exam of his left eye is shown in
Figure A. What is the best next step in management?
1No treatment indicated
2Massage the globe and start hyperbaric oxygen therapy
3Start multivitamin and antioxidant supplements
4Start bevacizumab
5Start acetazolamide drops

Fundoscopy with yellow extracellular material deposits known as “drusen,” characteristic of dry age-related
macular degeneration. fundoscopy with choroidal neovascularization, which can be seen in wet age-related
macular degeneration

This patient is presenting with visual


distortion, bilateral decrease in
vision, and a fundoscopic exam
positive for “drusen,” suggesting the
diagnosis of dry age-related macular
degeneration. The best initial therapy
is a multivitamin and antioxidant
supplements.

Dry ARMD is non-neovascular/non-


exudative, and presents as a gradual central vision loss (scotomas) over decades. Wet ARMD involves
choroidal neovascularization and can cause a more rapid loss of vision caused by bleeding secondary to
the new vessel formation. Both dry and wet ARMD can present with metamorphopsia or visual distortions.
Dry ARMD on fundoscopic exam will show drusen deposition (Figure A) and areas of retinal pigmentation
and de-pigmentation. Wet ARMD will show choroidal neovascularization (Illustration A). Dry ARMD is
treated with zinc and antioxidant vitamins to prevent progression. Wet ARMD is additionally treated with
vascular endothelial growth factor (VEGF) inhibitors and laser photocoagulation therapy.

Only for Study purpose / Only Notes / AMC MCQ


Only Study Notes / Only for Study purposes / Personal notes/ AMC MCQ

Figure A shows fundoscopy with yellow extracellular material deposits known as “drusen,” characteristic of
dry age-related macular degeneration. Ollustration A shows fundoscopy with choroidal neovascularization,
which can be seen in wet age-related macular degeneration.

Incorrect Answers:
Answer 1: No definitive treatment is indicated for retinal vein occlusion. Retinal vein occlusion presents as
sudden, painless, monocular vision loss. Fundoscopic exam would show retinal hemorrhage and venous
dilation.

Answer 2: Massaging the globe and hyperbaric oxygen therapy has shown to be beneficial in central retinal
artery occlusion. Symptoms include acute, painless, monocular vision loss. Fundoscopic exam would show
a cloudy retina with attenuated vessels and a “cherry-red” macula.

Answer 4: Bevacizumab is a VEGF inhibitor, which can be used to treat wet ARMD and is also part of
treatment for proliferative diabetic retinopathy. Both wet ARMD and proliferative diabetic retinopathy will
show forms of neovascularization on fundoscopic exam.

Answer 5: Acetazolamide is a diuretic used to treat open-angle glaucoma by inhibiting carbonic anhydrase,
which then decreases aqueous humor synthesis. Open-angle glaucoma presents as painless, peripheral
vision loss, resulting in “tunnel vision.” Optic disc atrophy causes an increased cup-to-disk ratio on
examination.

Bullet Summary:
Dry-ARMD, characterized by "drusen" on fundoscopic exam, is treated with zinc and antioxidant vitamins to
prevent progression.

128 - Acute painful red eye, fixed pupil, IOP 20 mmHg.


a. Timolol – Glaucoma (no in NTG)
b. Steroid – Glaucoma
c. Chloramphenicol - conjunctivitis
d. Atropine - … OP poisoning – exploration of ant
chamber in uveitis and keratitis
e. Acetazolamide – Glaucoma

When the signs of


glaucoma are
discovered in the
absence of an
abnormally high
IOP, the working
diagnosis may be
NTG.
However, topical
beta blockers have
the potential for
significant systemic side effects, such as nocturnal
hypotension, that may be of particular concern in NTG.
Choices for medical treatment in progressive cases
include betaxolol eye drops which have a beneficial
effect on optic nerve blood flow in addition to IOP reduction.[7] Other beta blockers and adrenergic drugs

Only for Study purpose / Only Notes / AMC MCQ


Only Study Notes / Only for Study purposes / Personal notes/ AMC MCQ

(such as dipivefrine) should better be avoided because of the probability of nocturnal systemic hypotension
and optic nerve hypoperfusion. Prostaglandin derivatives tend to have greater IOP lowering effect which
may be of overriding consideration.[7] Dorzolamide-timolol fixed combination (DTFC) is a safe and effective
IOP-lowering agent in patients with NTG.[47] Brimonidine significantly improved retinal vascular
autoregulation in NTG patients; however, short-term alterations in visual function could not be
demonstrated
https://www.ncbi.nlm.nih.gov/pmc/articles/PMC4926570/

Against Atropine: The presentation of OP poisoning depends upon whether the poisoning is mild, moderate
or severe. The symptoms are basically those of excessive acetylcholine activity. (Only one eye in this
question)
Mild
• Small or pinpoint pupils.

• Painful, blurred vision.


• Runny nose and eyes.
• Excess saliva.
• Eyes looking 'glassy'.
• Headache.
• Nausea.
• Mild muscle weakness.
• Localised muscle twitching.
• Mild agitation.

In normal pressure glaucoma, the main purpose and advice is to keep the IOP as low as possible as
there are no evident facts are available to know the cause of eye damage even the IOP is normal.

Only for Study purpose / Only Notes / AMC MCQ


Only Study Notes / Only for Study purposes / Personal notes/ AMC MCQ

GLAUCOMA SCREENING IMPORTANT


130- Scenario of acute closure angle
glaucoma in old age, haloes around
light, red sclera, IOP 25 mmHg, asked
long term management?
A. Trabeculectomy
B. Peripheral iridotomy
C. Acetazolamide
D. Pilocarpine

Angle-closure Glaucoma:
Patients with anatomic narrow angles
without acute angle closure are
typically asymptomatic in both the
primary and secondary forms.
Similarly, primary and secondary
chronic angle closure patients often
experience no symptoms unless they
develop end-stage glaucoma, in which
case they may complain of decreased
vision or reduced peripheral vision.
Acute angle closure, on the other
hand, usually presents with dramatic
symptoms from the quick rise in
intraocular pressure. Patients
complain of blurred vision,
rainbows, halos around lights, or
even transient loss of vision. They
often have intense pain that may be
localized to the eye, may follow the
trigeminal distribution, or may be
described as diffuse discomfort.
Nausea and vomiting are common.
Subacute or intermittent angle closure
attacks are brief episodes of angle
closure that resolve spontaneously.
Patients experience the above
symptoms of acute angle closure, but
on a milder scale. They will typically
experience some blurring of the vision
or halos with mild to moderate eye pain, brow ache, or headache. These attacks are often resolved by
entering a well lit room which may cause miosis or sleep as sleep-induced miosis ameliorates the lesser
degree of pupillary block in these patients.
Acute angle closure glaucoma
The role of medical therapy in acute angle closure attacks is to lower IOP, reduce pain, and clear corneal
edema in preparation for iridotomy. The medications below can be used, provided the patient has no
condition contraindicating them:
Topical
• Beta blockers
• Selective alpha agonists
• Carbonic anhydrase inhibitors

Only for Study purpose / Only Notes / AMC MCQ


Only Study Notes / Only for Study purposes / Personal notes/ AMC MCQ

• Miotics (e.g., pilocarpine 2%) may help break an early angle-closure attack, but may be ineffective if
the iris is already ischemic. High-concentration miotics (e.g., pilocarpine 4%) should be avoided
because of the potential for forward displacement iris-lens diaphragm.
• Prostaglandin analogues – unreliable effect in acute attack because of slow onset of action
• Hyperosmolar agent (e.g. 5% sodium chloride) – assists in clearing corneal edema
• Prednisolone 1% - decreases inflammation
Systemic
Carbonic anhydrase inhibitors – oral acetazolamide’s maximum IOP reduction is reached in 2-4 hours and
lasts for 6-8 hours. Intravenous acetazolamide drops the IOP within 2 minutes with a peak effect noted by
10-15 minutes. In acute situations, a single dose of 500 mg acetazolamide should be given orally if the
patient is not vomiting. Regular acetazolamide is preferred over the sustained-release sequel form because
of quicker onset of action. If the patient is vomiting, acetazolamide can be given intravenously.
Osmotic agents
Mannitol can decrease the IOP 30 mm Hg or more within 30 minutes of administration. The recommended
intravenous dose is 0.5-1.5 g/kg body weight as a 15% or 20% solution, delivered at 3 to 5 mL/minute. Frail
patients with cardiac or conditions may develop circulatory overload, pulmonary edema, congestive heart
failure, and electrolyte imbalance. A rapid reduction in cerebral volume may result in subdural hematomas
from vein rupture between the sagittal sinus and cortical surface. Therefore, patients receiving IV mannitol
should be monitored in a hospital setting.
Oral osmotic agents:
Glycerin: 1 to 1.5 g/kg body weight of a 50% solution. Onset of pressure reduction is typically 10 to 30
minutes. Avoid in diabetics because the increased caloric load can cause ketoacidosis.
Isosorbide is commercially available as a 45% (45 g/100 mL) solution (Ismotic; Alcon Surgical). The
recommended dose is 1 to 1.5 g/kg body weight. Its effect is similar to glycerin’s but is safe for use in
diabetics because it is not metabolized.
Although less common, oral agents can also cause subdural hematomas. Headache and gastrointestinal
upset are common adverse reactions.

Paracentesis Can be perfomed in an acute setting. Technically, it can be difficult to perform on a phakic
eye in pain with a shallow chamber, and there is a risk of permanent damage to the cornea, lens, and iris.
Devastating complications such as endophthalmitis and choroidal hemorrhage from a rapid pressure drop
may occur. Also the effects are typically short-term, because, as the ciliary body begins to form aqueous
again, the IOP will inevitably rise. This procedure can be used in cases of extreme IOP elevation to “buy
time” until medications take effect or iridotomy can be performed.
Laser Iridotomy Should be performed as soon as possible in the affected eye and in the contralteral eye to
avoid an attack of acute angle closure glaucoma in the future.
Chronic angle closure glaucoma
Very few studies exist to address medical therapy in chronic angle closure glaucoma after laser iridotomy.
In cases where elevated IOP becomes an issue, aqueous suppressants are helpful in reducing IOP.[32]
Prostaglandin analogues have been shown to be effective in lowering IOP, even in angles that are partially
closed.[32] [33] Evidence is not conclusive, however, regarding their effectiveness in cases of 360° degrees
of synechial closure.[34] The role of PI and other surgical interventions are described below.
Peripheral iridotomy
See Primary prevention section for information regarding prophylactic LPI for narrow angles.
Chronic open-angle glaucoma is a painless condition which causes damage to the optic nerve at the
back of your eye and can affect your vision. The person affected is not aware they have it: it is detected by
an optician or eye doctor. It is usually caused by an increase in pressure within your eye
Medical therapy
Medical therapy may be topical or systemic. However, it is important to appreciate that topical medication
may cause significant systemic side effects, especially beta-blockers.
Topical therapy
• Alpha-agonists (apraclonidine, brimonidine)

Only for Study purpose / Only Notes / AMC MCQ


Only Study Notes / Only for Study purposes / Personal notes/ AMC MCQ

• Beta-blockers (timolol,
betaxolol, carteolol,
levobunolol,etc.)
• Carbonic anhydrase inhibitors
(brinzolamide, dorzolamide)
• Miotics (pilocarpine,etc.)
• Prostaglandins (latanoprost,
bimatoprost, travoprost, etc.)
The most efficacious class of drops
is the prostaglandin analogues. They
are also the most convenient drops
with once nightly dosage. Side
effects are mainly local and include
hyperemia, lengthening of lashes,
darkening of iris color and peri-ocular
skin pigmentation. These are
reversible on stopping the
medication. The second most
efficacious class of drops is the beta-
blockers. Topical alpha-agonists,
carbonic anhydrase inhibitors and
miotics are much less efficacious,
rarely achieving a drop in IOP of
greater than 3 mmHg.
Surgery
• Non-medical interventions for
POAG include:
• Laser trabeculoplasty (see
Laser Trabeculoplasty: ALT
vs SLT)
• Trabeculectomy +/- augmentation (see Trabeculectomy)
• Non-penetrating drainage surgery
• Shunt procedures (see section Glaucoma Drainage Implant Surgery)
• Cyclodestructive procedures (cyclodiode, cyclocryotherapy)

Only for Study purpose / Only Notes / AMC MCQ


Only Study Notes / Only for Study purposes / Personal notes/ AMC MCQ

Let me explain...in open-angle chronic glaucoma if patients' symptoms are not severe we do laser
trabeculoplasty where just make a way for the aqueous flow in severe cases we have to remove some
tissue to make a way forqueous it's called a trabeculectomy.

131- 67-year-old man presents to the emergency room with eye pain. He notes that his right eye has
become progressively more painful over the past day and that his vision has been worsening. He had
cataract surgery in the same eye 2 weeks ago. He has a past medical history of diabetes, hypertension,
obesity, and erectile dysfunction. He is currently sexually active and engages in anal, oral, and vaginal
intercourse. His temperature is 100°F (37.8°C), blood pressure is 164/104 mmHg, pulse is 85/min,
respirations are 12/min, and oxygen saturation is 98% on room air. Visual acuity is 20/400 in the affected
eye. Physical exam is performed as seen in Figure
A. Which of the following is the most appropriate
treatment for this patient?
1Intravitreal ceftazidime and amikacin
2Intravitreal vancomycin and ceftazidime
3IV ceftriaxone and topical erythromycin
4IV vancomycin and amikacin
5IV vancomycin and piperacillin-tazobactam

This patient is presenting with blurry vision and a


hypopyon after cataract surgery which is
concerning for endophthalmitis which should be treated with intravitreal vancomycin and ceftazidime.
Infectious endophthalmitis is a severe complication of cataract surgery. Patients may present up to 6 weeks
following surgery with a red, painful eye, and on slit-lamp examination, a collection of pus may be seen
filling the anterior chamber. The
most common pathogens are
gram-positive agents such as
streptococcal species and
staphylococcal species. Physical
exam may demonstrate a
hypopyon and decreased visual
acuity. Management involves a
vitreous tap for culture and
injection of intravitreal antibiotics
with appropriate combinations
being vancomycin and either
ceftazidime or amikacin. Vitrectomy
(removal of the vitreous humor) is only indicated in
refractory or severe cases.
Figure A shows a hypopyon, an accumulation of pus in
the anterior chamber which is seen in endopthalmitis.
Incorrect Answers:
Answer 1: Intravitreal ceftazidime and amikacin are
incorrect as the appropriate antibiotics are vancomycin
and either ceftazidime or amikacin.
Answer 3: IV ceftriaxone and topical erythromycin may
be appropriate management of gonnococcal
conjunctivitis which presents with a purulent discharge
from the eye. It would not present with blurry vision and
a hypopyon.

Only for Study purpose / Only Notes / AMC MCQ


Only Study Notes / Only for Study purposes / Personal notes/ AMC MCQ

Answer 4: IV vancomycin and amikacin are inappropriate as this combination of antibiotics must be given
intravitreally.

Answer 5: IV vancomycin and piperacillin-tazobactam are an inappropriate combination of antibiotics and is


an inappropriate route of administration. This is a broad-spectrum regimen that could be appropriate in
septic shock.
Bullet Summary:
The treatment of endophthalmitis is intravitreal vancomycin and ceftazidime (or amikacin).

132- Another clinical scenario mentioning painful red eye but not much increase in lacrimation and eye lids
are not sticky (most likely trying to rule out purulent
cause). Pic given showing marked conjunctival
vessel injection. Asked for next appropriate step
Almost similar picture
a. Topical Steroid
b. Topical Acyclovir
c. Topical Chloramphenicol

133- A patient presents with very painful red eye


with watery discharge. Normal disc on fundoscopy
Eye muscle movements were all normal No picture
given Investigation asked
A)Ct
B)Tonometry
C)Gonioscopy
D)slit lamp examination
- Foreign body
- classic cases of scleritis present with intense
ocular pain, photophobia, and a deep-red or purplish
scleral hue. In addition, in episcleritis, there is no
edema or thinning of the sclera. Such changes in
the sclera are readily visible with the aid of a slit-
lamp biomicroscope

134- A 6-year-old boy is brought to the pediatrician


complaining of itchy eyes. The mother states that
she has noted that he has been tearing and that
both of his eyes have been red for the past 4 days.
The patient denies any pain but has had a runny
nose for the past week. The mother states that he
has not had any sick contacts, and he has been
home from school for summer vacation. She notices

Only for Study purpose / Only Notes / AMC MCQ


Only Study Notes / Only for Study purposes / Personal notes/ AMC MCQ

that his symptoms seem to improve in the evening after he showers. His temperature is 97.0°F (36.1°C),
blood pressure is 100/66 mmHg, pulse is 90/min, respirations are 22/min, and oxygen saturation is 98% on
room air. Physical exam is notable for the finding in Figure A. Which of the following is the most likely
diagnosis?

1Allergic conjunctivitis
2Bacterial conjunctivitis
3Corneal abrasion
4Endophthalmitis
5Viral conjunctivitis

This patient is suffering from allergic conjunctivitis


given his pruritus, edema, and hyperemia of his eyes
without sick contacts and with improvement of
symptoms with showers (allergen removal).
Allergic conjunctivitis is a common condition which is caused by sensitivity to pollen, dander, mold, or
another allergen. It is part of a systemic reaction to an allergen, most commonly, pollen. Patients present
with intense itching of both eyes, as well as redness, tearing, and edema. Many patients have other allergic
symptoms as well, such as allergic rhinitis, asthma, and eczema. The treatment is allergen avoidance,
topical olopatadine, and oral antihistamines (such as cetirizine).

Figure A shows the eye of a patient with allergic conjunctivitis with hyperemia of the conjunctiva, swelling of
the eyelids and conjunctiva, and a watery discharge.
Incorrect Answers:
Answer 2: Bacterial conjunctivitis presents with a purulent discharge from the eye and is treated with topical
antibiotics.
Answer 3: Corneal abrasion presents after trauma with unilateral eye pain. A fluorescein stain will
demonstrate increased uptake in the lesion. The treatment is topical erythromycin.
Answer 4: Endophthalmitis presents after eye surgery with pain, decreased vision, erythema, and a
hypopyon in the eye. Management includes aspiration for culture and intravitreal antibiotics.
Answer 5: Viral conjunctivitis is a common condition that is very contagious. Unlike allergic conjunctivitis, it
begins unilaterally and then progresses to the other eye. It presents with a watery discharge, hyperemia of
the conjunctiva, and eyelid swelling. Although possible, allergic conjunctivitis is the more likely cause in this
patient, as he presented with bilateral symptoms, has no sick contacts, and has symptoms that improve
with showers (which removes the allergen).
Bullet Summary:
Allergic conjunctivitis presents with bilateral pruritus, hyperemia, and tearing of the eye and is worsened
with allergen exposure.

Only for Study purpose / Only Notes / AMC MCQ


Only Study Notes / Only for Study purposes / Personal notes/ AMC MCQ

135- A 63-year-old man with chronic constipation presents


to his primary care physician complaining of blood in his
eye. He noticed that his eye was red when he awoke this
morning. He denies any eye trauma, itching, burning, pain,
or change in vision. His eye is shown in Figure A. What is
the most appropriate next step?
1Prescribe atropine drops
2Refer to an ophthalmologist
3Treat constipation, no treatment for eye condition is
necessary
4Prescribe antibiotic drops
5Patch the eye to prevent further damage
This patient presents with a subconjunctival hemorrhage
following straining. The condition is benign and does not
require further workup or treatment.

Subconjunctival hemorrhage is due to rupturing of the fragile subconjunctival vessels leading to a painless
hemorrhage. Subconjunctival hemorrhage is a benign process and has many causes, including eye
trauma, straining from coughing, vomiting, or constipation, coagulation disorders, and hypertension.
Patients are often unaware of the inciting cause. There is no need for treatment, and it typically resolves in
weeks.

Cronau et al. describe the causes and treatment of red eye, including subconjunctival hemorrhage. While
conjunctivitis is the most common cause of red eye, physicians should be aware of other causes, including
subconjunctival hemorrhage, corneal abrasions, foreign body, blepharitis, glaucoma, and chemical burns.
They state that while no treatment is needed for subconjunctival hemorrhage, warm compresses and
lubricants may be used for symptomatic relief. Furthermore, subconjunctival hemorrhage is painless, and
thus the presence of pain should prompt further workup in order to identify the cause.

Pokhrel et al. describe the recognition and treatment of ocular emergencies. Common ocular emergencies
include penetrating injuries, central retinal artery occlusion, retinal detachment, acute angle-closure
glaucoma, and chemical burns. They state that all ocular emergencies should be seen by either the
emergency department or an ophthalmologist. Of note, treatment of chemical burns should be immediate
and not delayed by the evaluation of visual acuity

136- A 62-year-old man presents to the ED complaining of severe eye pain that started a few hours ago.
The patient reports that he fell asleep while watching TV on the couch and woke up with right-sided eye
pain and blurry vision. His wife drove him to the emergency room. His wife reports that since they arrived
the patient has also been complaining of intense nausea. The patient denies fever, headache, or visual
floaters. He has a history of hypertension, hyperlipidemia, type II diabetes mellitus, and osteoarthritis. He
takes aspirin, lisinopril, metformin, atorvastatin, and over-the-counter ibuprofen. His temperature is 99°F
(37.2°C), blood pressure is 135/82 mmHg, and pulse is 78/min. On physical examination, the right eye is
firm with an injected conjunctiva and a mildly cloudy cornea. The pupil is dilated at 6 mm and is non-
reactive to light. Ocular eye movements are intact. Vision is 20/200 in the right eye and 20/40 in the left
eye. The left eye exam is unremarkable. Which of the following is the most appropriate initial treatment?
1Intravenous acetazolamide
2Iridotomy
3Retinal photocoagulation
4Topical epinephrine: Topical epinephrine is used in treatment of open angle glaucoma, but is
contraindicated in closure angle as narrow angles and any degree of pupillary dilation can provoke an
acute attack of angle-closure glaucoma
5Topical prednisolone

Only for Study purpose / Only Notes / AMC MCQ


Only Study Notes / Only for Study purposes / Personal notes/ AMC MCQ

Diagnosis and reason

For an acute primary angle-closure attack, initial management involves prompt administration of
pressure-lowering eye drops. A possible regimen would be one drop each, one minute apart, of
[22]:

●0.5% timolol maleate;


●1% apraclonidine; and
●2% pilocarpine, especially immediately prior to laser peripheral iridotomy [2]

We also suggest giving the patient 500 mg of oral or intravenous (IV) acetazolamide.
Gonioscopy — Gonioscopy is the gold-standard method of diagnosing angle-closure glaucoma. This
technique involves using a special lens for the slit lamp, which allows the ophthalmologist to visualize the
angle

Epinephrine is reported to reduce intraocular pressure by decreasing aqueous formation and increasing
outflow facility. Because epinephrine is devoid of accommodative and miotic effects, is administered twice a
day, and is accompanied by relatively minor complications, it can be used as the initial therapy for primary
open angle glaucoma, as an additive to current therapy, and as part of maximal medical therapy for
glaucoma. Epinephrine is contraindicated for angle-closure glaucoma and should be used with caution in
patients with narrow angles. Although there is a potential for serious systemic side effects, most of the
recognized side effects are not serious and are reversible with drug discontinuation. The development of
dipivalyl epinephrine (DPE) as a molecule with greater ocular penetration promises to maintain this drug's
therapeutic effect with a reduction in external and extraocular side effects.
The use of ophthalmic preparations of epinephrine, including dipivefrin (a prodrug of epinephrine),
is contraindicated in patients with narrow-angle glaucoma or anatomically narrow angles. ... In patients
with narrow angles, any degree of pupillary dilation can provoke an acute attack of angle-closure
glaucoma

This patient is acutely presenting with a firm, red eye, severe eye pain, blurry vision, and a fixed dilated
pupil, which is consistent with acute angle-closure glaucoma. Part of initial management for acute angle-
closure glaucoma is acetazolamide.

Acute angle-closure glaucoma presents with acute onset pain and blurry vision with occasional
nausea/vomiting. The episode often occurs following pupillary dilation such as while sleeping or watching a
movie. Physical examination shows a fixed-dilated pupil, corneal clouding/edema, conjunctival injection,
and a firm eyeball. Initial treatment should be aimed at decreasing pressure such as with topical beta-
blockers and alpha-2-agonists and oral diuretics (or IV if nausea is present) like acetazolamide (a carbonic
anhydrase inhibitor that decreases aqueous humor secretion).

Incorrect Answers:
Answer 2: Iridotomy is the definitive treatment for acute angle-closure glaucoma. However, in the acute
phase, iridotomy should be delayed until corneal edema resolves.

Answer 3: Retinal photocoagulation is indicated in proliferative diabetic retinopathy. Patients present with
gradual and painless vision loss. Dot-blot retinal hemorrhages and neovascularization would be seen on
ocular exam.

Answer 4: Topical epinephrine is contraindicated in acute angle-closure glaucoma. This is because alpha-
1-agonists cause mydriasis that worsen acute angle-closure glaucoma.

Only for Study purpose / Only Notes / AMC MCQ


Only Study Notes / Only for Study purposes / Personal notes/ AMC MCQ

Answer 5: Topical prednisolone is not efficacious during an acute angle-closure attack. It is useful in
managing inflammation once the acute attack resolves.

137 – Endocarditis Roth Spots!

Roth Spots are defined as a white centered retinal hemorrhage and are associated with multiple
systemic illnesses, most commonly bacterial endocarditis. Originally described by Moritz Roth in 1872
while at the University of Basel, Roth spots were first seen in individuals with bacteremia secondary to
subacute bacterial
endocarditis.[1][2] The retinal findings
Roth made in 1872 were described
as round, oval or flame-shaped
hemorrhages with a central white
spot. Although originally described
by Roth, Roth spots earned their
name only later by the (biologist)
Litten in 1878. Litten made more
detailed observations of these
entities and claimed they appeared
in approximately 80% of cases of
subacute bacterial endocarditis, thus
cementing the hallmark
association.[3][2] However they can be
seen in a wide variety of conditions,
and more recent research has found
that only 2% of patients with
endocarditis had Roth spots present
on their retina.[4] Though Roth
spots are usually thought of as

Only for Study purpose / Only Notes / AMC MCQ


Only Study Notes / Only for Study purposes / Personal notes/ AMC MCQ

pathognomonic for bacterial endocarditis, they can occur in a number of conditions

138- red eye, decreased visual acuity, normal pupil response


1. Iritis(provide reason)
2. keratitis(provide reason)
3. angle-closure glaucoma(provide reason)
• For the picture: Conjunctival examination with 360°
perilimbal injection that decreases around the
limbus – conjunctivitis
• Here Keratitis
• The opposite conjunctival examination with 360°
perilimbal injection that increases towards the
limbus - ant uveitis (+ irregular pupil, and decrease vision.
• Keratitis doesn't normally have decrease vision, but herpetic keratitis can present with decreased
vision, normal pupil and red eye
• Uveitis - Miotic pupil (constrictive non-reactive) - painful eye - visual impairment

• Glaucoma - fixed pupil Mydriasis (irregular dilated) - painful eye - visual impairment

139- Spot Diagnosis: Hypopyon (Uveitis or endophthalmitis)

Only for Study purpose / Only Notes / AMC MCQ


Only Study Notes / Only for Study purposes / Personal notes/ AMC MCQ

Acanthamoeba keratitis is a rare


disease in which amoebae invade the
cornea of the eye, and affects
roughly 1.2 to 3 million people each
year

Only for Study purpose / Only Notes / AMC MCQ


Only Study Notes / Only for Study purposes / Personal notes/ AMC MCQ

Only for Study purpose / Only Notes / AMC MCQ


Only Study Notes / Only for Study purposes / Personal notes/ AMC MCQ

Early manifestations in the cornea can be seen as punctate keratopathy, pseudodendrites, and epithelial or
subepithelial corneal deposits.[10] These features can lead an examiner to confuse AK with a viral keratitis,
such as that caused by Herpes zoster virus or Herpes Simplex Virus
One of Differential diagnosis of Acanthamoeba keratitis is herpetic keratitis see here HERPETIC

140 - A 22 year old man has had an acute, painful, red right eye with blurring of vision for one day. He
had a similar episode one year ago and has had episodic back pain and stiffness relieved by exercise
and diclofenac for four years. what is the most likely cause of his red eye?
A Chorioretinitis
B Conjunctivitis
C Episcleritis
D Iritis
E Keratitis

D - Iritis (Ant Uveitis) related to prob ankylosing spondylitis (back pain and similar episode last year)
Iritis, also known as anterior uveitis, is a condition sometimes associated with AS where the front part of the
eye becomes red and swollen. It usually only affects 1 eye, rather than both.

141- exudates for diabetic


retinopathy

Only for Study purpose / Only Notes / AMC MCQ


Only Study Notes / Only for Study purposes / Personal notes/ AMC MCQ

142- in right eye improved after some time.


Cause?
a. CRAO
b. Carotid Disease

c. Retinal detachment

143 -

Retinoblastoma (Rb) is a rare form of cancer that


rapidly develops from the immature cells of
a retina, the light-detecting tissue of the eye. It is
the most common primary malignant intraocular
cancer in children, and it is almost exclusively
found in young children.
The most common and obvious sign of
retinoblastoma is an abnormal appearance of the
retina as viewed through the pupil, the medical
term for which is leukocoria, also known as
amaurotic cat's eye reflex.[1] Other signs and
symptoms include deterioration of vision, a red
and irritated eye with glaucoma, and faltering
growth or delayed development. Some children
with retinoblastoma can develop a squint,[3] commonly referred to as "cross-eyed" or "wall-eyed" (strabismus).
Retinoblastoma presents with advanced disease in developing countries and eye enlargement is a common
finding.
Depending on the position of the tumors, they may be visible during a simple eye exam using
an ophthalmoscope to look through the pupil. A positive diagnosis is usually made only with an examination
under anesthetic (EUA). A white eye reflection is not always a positive indication of retinoblastoma and can be
caused by light being reflected badly or by other conditions such as Coats' disease.

Only for Study purpose / Only Notes / AMC MCQ


Only Study Notes / Only for Study purposes / Personal notes/ AMC MCQ

The presence of the photographic fault red eye in only one eye and not in the other may be a sign of
retinoblastoma. A clearer sign is "white eye" or "cat's eye" (leukocoria).
Differential diagnosis
1. Persistent hyperplastic primary vitreous is a congenital developmental anomaly of the eye resulting from
failure of the embryological, primary vitreous, and hyaloid vasculature to regress, whereby the eye is
shorter, develops a cataract, and may present with whitening of the pupil.
2. Coats disease is a typically unilateral disease characterised by abnormal development of blood vessels
behind the retina, leading to blood vessel abnormalities in the retina and retinal detachment to mimic
retinoblastoma.
3. Toxocariasis is a parasitic disease of the eye associated with exposure to infected puppies, which
causes a retinal lesion leading to retinal detachment.
4. Retinopathy of prematurity is associated with low-birth-weight infants who receive supplemental oxygen
in the period immediately after birth, and it involves damage to the retinal tissue and may lead to retinal
detachment.

MRI pattern of retinoblastoma with optic nerve involvement (sagittal enhanced T1-weighted sequence)
If the eye examination is abnormal, further testing may include imaging studies, such as computerized
tomography (CT), magnetic resonance imaging (MRI), and ultrasound.[15] CT and MRI can help define the
structure abnormalities and reveal any calcium depositions. Ultrasound can help define the height and
thickness of the tumor. Bone marrow examination or lumbar puncture may also be done to determine any
metastases to bones or the brain.

144- Picture of very red eye with subconjunctival haemorrhage with a trauma history of hitting the eye with
a baseball abt asking for next step:
A- CT
B- Warm dressing
C- Antibiotic
D- Local anesthetic

Subconjunctival haemorrhage is treated with warm


dressing if spontaneous
If previous trauma possible orbital fracture and head
injury in an old patient always CT

Only for Study purpose / Only Notes / AMC MCQ


Only Study Notes / Only for Study purposes / Personal notes/ AMC MCQ

Please see the criteria for CT from uptodate - Features that should prompt neuroimaging include the
following:
• GCS ≤14
• Suspected open or depressed skull fracture (scalp lacerations or hematomas, or bony step-off of the
skull)
• Any sign of basilar skull fracture (hemotympanum, periorbital bruising, retroauricular bruising, otorrhea,
rhinorrhea)
• Two or more episodes of vomiting
• New neurologic deficit (eg, cranial nerve, motor, sensory, coordination, gait, cognition)
• Presence of a bleeding diathesis or use of an anticoagulant medication
• Seizure
• Age 60 years or older

Only for Study purpose / Only Notes / AMC MCQ


Only Study Notes / Only for Study purposes / Personal notes/ AMC MCQ

• Retrograde amnesia of a 30-minute or


longer period of time before the
traumatic episode
• Worrisome mechanism of head injury
(eg, pedestrian struck by motor
vehicle, occupant ejected from motor
vehicle, fall from 3 feet [approximately
1 meter] or more, fall down 5 stairs or
more)
• Intoxication, headache, or abnormal
behavior (such as excessive agitation,
inconsolability, uncooperativeness, odd
affect, violent behavior) are less well-
validated criteria for neuroimaging, but
some clinicians would obtain a
noncontrast head computed tomography
(CT) in patients with one or more of
these features

145- A 35-year-old HIV-positive male is complaining of deterioration of his vision over the past week. He
initially experienced eye pain and mild conjunctivitis, followed by rapid progressive visual loss.
Examination of his eyes reveals marked keratitis. Funduscopy shows widespread, pale, peripheral
retinal lesions and central necrosis of the retina. Which of the following is the most likely causative
organism of this patient’s condition?
A. Pseudomonas
B. Cytomegalovirus
C. Herpes simplex
D. Candida albicans
E. Epstein Barr virus

In HIV patients, both HSV and VZV can


cause severe, acute retinal necrosis
associated with pain, keratitis, uveitis, and
funduscopic findings of peripheral pale
lesions and central retinal necrosis. In
contrast, CMV retinitis is painless, not
usually associated with keratitis or
conjunctivitis, and characterized by
funduscopic findings of hemorrhages and
fluffy or granular lesions around the retinal vessels. CMV retinitis is the most common serious ocular
complication of HIV positive patients.

Only for Study purpose / Only Notes / AMC MCQ


Only Study Notes / Only for Study purposes / Personal notes/ AMC MCQ

Progressive outer retinal necrosis


Progressive outer retinal necrosis (PORN) is another variant of herpetic retinitis in AIDS patients and is
nearly always caused by VZV though HSV-1 has been reported to cause PORN syndrome.
widespread, pale, peripheral retinal lesions and central necrosis of the
retina

146- . a pic of eye with conjunctival injection, very red eye. . .headache with photophobia
A:uveitis
B:episcleritis
C:conjuctivitis
D:glaucoma
Glaucoma red eye headache
could be due to increased iop
too and photophobia presents

147-

Treatment

Only for Study purpose / Only Notes / AMC MCQ


Only Study Notes / Only for Study purposes / Personal notes/ AMC MCQ

High-dose vitamin A is the treatment for all


individuals with xerophthalmia and for infants or
children with severe malnutrition or measles.

Improvement of Bitot’s spots is seen within 2


weeks of high-dose vitamin A therapy. However,
the retinal mani-festations of vitamin A deficiency
are slower to respond to treatment, with night
blindness and dark adaptation problems often
persisting for 4 weeks.

https://www.rch.org.au/immigranthealth/clinical/Vitamin_A/?fbclid=IwAR0yoFiSPIMiqvBaw07BfSmkylLMF7
QVXWjFkMkgod0b70Zdv_k0gR-aRbs
Vitamin A
• Background
Vitamin A is a fat soluble vitamin that is required for vision, immune function, growth and maintenance of
epithelial cells.[1] Infants accumulate stores in the 3rd trimester of pregnancy and rely on breast milk for
supply. Vitamin A is measured in retinol equivalents . Foods contain both retinol and vitamin A
precursors (carotenoids), which can be converted to vitamin A.
1 retinol equivalent 1 mcg retinol 6 mcg beta-carotene 12 mcg other 3.33 IU vitamin A
(RE) provitamin A carotenoids (from retinol)
WHO recommends supplementation in high risk* children and women post-partum to prevent
vitamin A deficiency.[6, 7] Vitamin A deficiency is associated with visual impairment and increased
mortality in children, particularly from measles and diarrhoea. Treatment with vitamin A has been shown to
reduce mortality in children with measles infection.[1, 2] Vitamin A used as primary prevention has been
shown to reduce all-cause mortality in babies aged < 6 months,[3, 4] and in infants > 6 months of age.[5]
• Infants < 6 months: 50,000 IU at 6, 10 and 14 weeks
• Infants 6 - 12 months: 100,000 IU every 4 - 6 months
• Children > 12 months of age: 200,000 IU every 4 - 6 months
• Post-partum women: 200,000 IU x 2 doses within 6 weeks of delivery, at least 24 hours
apart
• Additional guidelines are available for treatment of xerophthalmia.
Note: *High risk is defined as: measles, diarrhoea, respiratory disease, chicken pox, severe protein calorie
malnutrition or living in vicinity of children with clinical Vitamin A deficiency.
Prevalence of vitamin A deficiency

Only for Study purpose / Only Notes / AMC MCQ


Only Study Notes / Only for Study purposes / Personal notes/ AMC MCQ

Low vitamin A was found in 19% - 38% of African refugee children attending RCH immigrant health clinic in
the early 2000s (noting many of these children had spent time in refugee camps) [8,9] and 3% of Karen
refugee children in a community based sample in Victoria.[10] No cases of xerophthalmia have ever been
identified within our service.
Assessment
• Consider vitamin A deficiency in children from Africa and South Asia , especially if they
have had restricted food access pre-arrival, or have poor nutrition. Maps of prevalence are
available.
• Eye signs of vitamin A deficiency: night blindness, Bitot's spots (grey-white deposits on the
bulbar conjunctiva adjacent to the cornea) or xerophthalmia (dryness, corneal ulceration,
keratomalacia).
• Children with eye signs should be referred for urgent ophthalmology
assessment .
Screening
• Screening for low Vitamin A is part of the initial refugee health assessment for
children where food access has been restricted. Screening is probably unnecessary in
children after initial settlement if they have good nutritional status and an age appropriate
diet
• The laboratory should report age-specific ranges for serum Vitamin A. A serum retinol < 0.7
micromol/l is significantly low for children of all ages
• Serum retinol may be transiently low during acute infections
• Vitamin A is light sensitive and blood samples for testing must be protected from
light, otherwise levels will be (falsely) low
Management
• Promote breast feeding of infants and encourage Vitamin A rich foods for infants and
children
• Vitamin A rich foods include yellow fruits and vegetables, butter/margarine, cheese,
eggs and offal (liver, kidney)
• Vitamin A is destroyed by prolonged cooking, and the vitamin A content of foods can
be reduced by drying or freezing
• Low vitamin A should be treated (Table 1). High dose capsules (50,000 IU) containing an
oil-based solution (and gelatin) are available, these do not require a prescription, although
they may need to be ordered in the community setting.
Table 1: Treatment of low vitamin A
Vitamin A level Age Vitamin A dose Follow-up

0.35 - 0.7 micromol/L(or <6 50,000 IU stat Repeat dose at 6 months if


low for age) months 100,000 IU stat risk factors persist
6 - 12 200,000 IU stat
months
> 12
months

< 0.35 micromol/L(or eye <6 50,000 IU oral daily Follow-up levels and repeat
signs) months for 2 days dose at 2 - 4 weeks
6 - 12 100,000 IU oral
months daily for 2 days
> 12 200,000 IU daily for
months 2 days
• Australian recommended daily intakes [11] are shown below in Table 2.
Toxicity
• Vitamin A toxicity may follow doses of 20 times the RDI in children, although high dose
supplementation in children with measles is not associated with acute toxicity/adverse
effects.[1]
• Side effects include irritability, vomiting and bulging fontanelle in 1 - 2%, which generally
resolves within 48 hours of ingestion.[12]

• Vomiting is more common in those aged < 6 months and is usually mild and self-
limited.[6]

Only for Study purpose / Only Notes / AMC MCQ


Only Study Notes / Only for Study purposes / Personal notes/ AMC MCQ

•Massive overdose can cause rough skin, dry hair, hepatomegaly, and raised ESR,
serum calcium and ALP.
• High dose Vitamin A should not be given to women of child-bearing age who may be
pregnant. Doses should not exceed 10,000 units a day or 25,000 units a week.[13] Most
commercial cod-liver oil capsules contain less than this amount.
Table 2. Australian Recommended Dietary Intake Vitamin A by life stage and gender
Infants Gender AI

0 - 6 months All 250 mcg/day of retinol (as retinyl esters)

7 - 12 months 430 mcg/day of retinol equivalents

Children and adolescents Gender EAR as RE RDI as RE

1 - 3 years All 210 mcg/day 300 mcg/day

4 - 8 years 275 mcg/day 400 mcg/day

9 - 13 years Girls 420 mcg/day 600 mcg/day

14 - 18 years 485 mcg/day 700 mcg/day

9 - 13 years Boys 445 mcg/day 600 mcg/day

14 - 18 years 630 mcg/day 900 mcg/day


Note: The Adequate Intake (AI) is used when no EAR or RDI is available and is based on
observed/experiemental intakes. The Estimated Average Requirement (EAR) reflects the estimated median
requirements, the Recommended Daily Intake (RDI) meets or exceeds the requirements for 97.5% of the
population. RE = retinol equivalents.

148- A 67-year-old woman presents to the ophthalmologist with complaints of worsening visual loss. She
states that her vision is blurry. Driving has become difficult, particularly at night, as she experiences
substantial glare and sees halos around lights. On physical examination, there is absence of a red
reflex. What is the most likely cause of this patient’s visual loss?
1Age-related macular degeneration

2Cataracts

3Open-angle glaucoma

4Refractive error

5Retinoblastoma

Halos are a common symptom of cataracts and so is glare, giving the sense that normal lights are bright
Absent red reflex halls and progressive loss of vision with old age and difficult driving at night all towards
cataract. There is no headache or problem with the pupils mentioned so ruled out option 3 .and age related
mD will not affect the red reflex
This elderly patient with painless, blurry vision, and an absent red reflex on physical exam most likely has
cataracts.

Only for Study purpose / Only Notes / AMC MCQ


Only Study Notes / Only for Study purposes / Personal notes/ AMC MCQ

Cataracts are opacifications of the lens of the eye, resulting in slow and progressive loss of visual acuity.
Risk factors for cataracts include age, smoking, glucocorticoids, UV exposure, and diabetes. Patients with
cataracts experience a painless, progressive, and gradual loss of visual acuity and typically have blurry
vision with complaints of glares and halos around lights. On physical exam, the lens appears cloudy, and
there may be the absence of a red reflex (Illustration A)

Incorrect Answers:
Answer 1: Age-related macular degeneration (ARMD) is a cause of visual loss in the elderly population due
to degeneration of the retina and neovascular proliferation. ARMD would not cause the absence of a red
reflex.
Answer 4: Refractive error is a common cause of blurry vision that can be easily corrected with glasses or
contact lenses, but would not produce the absence of a red reflex
Answer 5: Retinoblastoma is the most common intraocular malignancy in children and is thus unlikely in
this elderly patient. An abnormal red reflex is the most common presentation of retinoblastoma.

Bullet Summary:
Cataracts present with blurry vision and absence of a red reflex in severe cases

Only for Study purpose / Only Notes / AMC MCQ


Only Study Notes / Only for Study purposes / Personal notes/ AMC MCQ

149- No purulent discharge, no itchiness. No impairment of vision. Which eyedrop will you give?

A. Hypromellose [eye drop]


B. Ketotifen
C. Sodium cromoglycate
D. Prednisolone
E. Chloramphenicol

• No itchiness rule out allergic cause


• No discharge rule out conjunctivitis
• No impaired vision rule out glaucoma, uveitis, scleritis (painful loss of vision)

150- Old lady picture of jaundiced eyes – she looked scary, presented with left facial nerve plays , dribbling
and some ear discharge CLEAr
Next –
1. ct
2. MRI
3. Valcyclovie if VVZ related
4. prednisolone (if etiology HSV or Hep B,A,E,C)

Acute liver failure (ALF) can be due to numerous causes and result in fatality or necessitate liver
transplantation if left untreated. Possible etiologies of ALF include ischemia, venous obstruction,
medications, toxins, autoimmune hepatitis, metabolic and infectious causes including hepatitis A-E,
varicella-zoster virus (VZV), cytomegalovirus (CMV), herpes simplex virus (HSV), Epstein-Barr virus (EBV),
and adenovirus with VZV being the most rarely reported.
https://www.hindawi.com/journals/crihep/2018/1269340/
Diagnostic tests — Electrodiagnostic studies help determine the prognosis, and imaging studies can
define potential surgical causes of facial palsy. However, these tests are not necessary in all patients.
Patients with a typical lesion that is incomplete and recovers do not need further study.
Imaging studies — As mentioned above, imaging is warranted if the physical signs are atypical, there is
slow progression beyond three weeks, or if there is no improvement at four months. History of a facial
twitch or spasm that precedes facial weakness suggests nerve irritation from tumor and should also prompt
imaging
Other investigations — Serologic testing for Lyme disease is recommended for adults with acute-onset
facial palsy when there is the possibility of exposure in Lyme-endemic areas during the spring through
autumn seasons
The mainstay of pharmacologic therapy for acute idiopathic facial nerve palsy (Bell's palsy) or facial nerve
palsy of suspected viral etiology is early short-term oral glucocorticoid treatment. In severe acute cases,
combining antiviral therapy with glucocorticoids may improve outcomes. Eye care is important for patients
with incomplete eye closure
Glucocorticoid and antiviral therapy — We recommend early treatment with oral glucocorticoids for all
patients with idiopathic facial nerve palsy (Bell's palsy) or facial nerve palsy of suspected viral etiology,

Management Early therapy with aciclovir or related agents has been shown to reduce both early-
and late-onset pain, especially in patients over 65 years. Post-herpetic neuralgia requires aggressive
analgesia, along with agents such as amitriptyline 25–100 mg daily, gabapentin (commencing at 300 mg
daily and building slowly to 300 mg twice daily or more) or pregabalin (commencing at 75 mg twice daily
and building up to 100 mg or 200 mg 3 times daily if tolerated). Capsaicin cream (0.075%) may be helpful.
Although controversial, glucocorticoids have not been demonstrated to reduce post-herpetic neuralgia to
date.

Only for Study purpose / Only Notes / AMC MCQ


Only Study Notes / Only for Study purposes / Personal notes/ AMC MCQ

* House-Brackmann classification of facial nerve dysfunction (refer to UpToDate text for complete description):

1. Normal

2. Mild dysfunction

3. Moderate dysfunction

4. Moderately severe dysfunction

5. Severe dysfunction

6. Total paralysis

¶ Acyclovir (400 mg 5 times daily for 10 days) is an alternative to valacyclovir but is less convenient and
has inferior bioavailability.

Only for Study purpose / Only Notes / AMC MCQ


Only Study Notes / Only for Study purposes / Personal notes/ AMC MCQ

Only for Study purpose / Only Notes / AMC MCQ


Only Study Notes / Only for Study purposes / Personal notes/ AMC MCQ

Extra info
Varicella-zoster virus (VZV) infection causes two clinically distinct forms of disease [1]. Primary infection with
VZV results in varicella, also known as chickenpox, characterized by vesicular lesions in different stages of
development on the face, trunk, and extremities. Herpes zoster, also known as shingles, results from
reactivation of endogenous latent VZV infection within the sensory ganglia. This clinical form of the disease
is characterized by a painful, unilateral vesicular eruption, which usually occurs in a restricted dermatomal
distribution

The management of herpes zoster includes:

●Antiviral therapy to hasten healing of cutaneous lesions and to decrease the duration and
severity of acute neuritis. Whether antiviral therapy decreases the risk of post-herpetic
neuralgia (PHN) is less clear. (See 'Antiviral therapy' below.)
●Analgesia for patients with moderate to severe acute neuritis. (See 'Analgesia for acute
neuritis' below.)

The treatment of established PHN is discussed elsewhere. (See "Postherpetic neuralgia", section
on 'Treatment'.)

Only for Study purpose / Only Notes / AMC MCQ


Only Study Notes / Only for Study purposes / Personal notes/ AMC MCQ

Antiviral therapy — The goals of antiviral therapy are to [3,4]:


●Lessen the severity and duration of pain associated with acute neuritis
●Promote more rapid healing of skin lesions
●Prevent new lesion formation
●Decrease viral shedding to reduce the risk of transmission
●Prevent PHN

Several lines of evidence suggest that antiviral therapy hastens resolution of cutaneous lesions
and the acute neuritis of herpes zoster [5-7]. However, it is unclear if antiviral therapy prevents
PHN because of conflicting study results, which are due in part to different methodologies of pain
assessment, definitions of PHN, and length of follow-up [5-8].

≤72 hours after onset — We recommend antiviral therapy for patients with uncomplicated herpes
zoster who present within 72 hours of clinical symptoms. Antiviral therapy should be initiated within
this time frame to maximize the potential benefits of treatment

We prefer valacyclovir or famciclovir compared with acyclovir given the need for less frequent
dosing. Small comparative trials do not support the efficacy of one over the other [11-13].

The doses used to treat herpes zoster are as follows:

●Valacyclovir: 1000 mg three times daily for seven days


●Famciclovir: 500 mg three times daily for seven days
●Acyclovir: 800 mg five times daily for seven days

Acyclovir and its analogues are dependent upon renal function for clearance and dose adjustment
is needed in moderate to severe renal insufficiency.

Herpes zoster ophthalmicus — Herpes zoster ophthalmicus, a serious sight-threatening condition,


has been linked to varicella zoster virus (VZV) reactivation within the trigeminal ganglion [26,27].
Patients can develop conjunctivitis, episcleritis, keratitis, and/or iritis

Early diagnosis and treatment is critical to prevent progressive corneal involvement and potential
loss of vision [28].

●The standard approach to herpes zoster ophthalmicus includes oral antiviral therapy
(acyclovir, valacyclovir, or famciclovir) to limit VZV replication, and the use adjunctive topical
steroid drops to reduce the inflammatory response and control immune-associated keratitis
and iritis [26-29].
●Intravenous acyclovir (10 mg/kg three times daily for seven days) should be administered if
the patient is immunocompromised or requires hospitalization for sight-threatening disease.
Ramsay Hunt syndrome — The major otologic complication of VZV reactivation is the Ramsay Hunt
syndrome, which typically includes the triad of ipsilateral facial paralysis, ear pain, and vesicles in the
auditory canal and auricle.

For most patients, we administer valacyclovir (1 g three times per day for 7 to 10 days)
and prednisone (1 mg/kg for five days, without a taper). In severe cases (eg, vertigo, tinnitus, or
hearing loss), IV therapy can be initiated, and the patient can then be transitioned to an oral
antiviral agent when the lesions begin to crust.

However, there are few data to guide treatment decisions about management of this complication.
Although a systematic review found no evidence that antiviral agents have a beneficial effect on
outcomes in Ramsay Hunt syndrome [36], a subsequent retrospective review of 101 patients
found that patients who received acyclovir and glucocorticoids recovered significantly more than
those who had only one or no pharmacologic treatment

Only for Study purpose / Only Notes / AMC MCQ


Only Study Notes / Only for Study purposes / Personal notes/ AMC MCQ

151- An 82-year-old man presents to the emergency department complaining of vision loss in his left eye.
He states that it suddenly appeared as if a curtain was coming down over his left eye. It resolved after
five minutes, and his vision has returned to normal. He has a history of coronary artery disease and type 2
diabetes. What is the most likely cause of this patient's presentation?
1Sclerosis and narrowing of retinal vessels
2Deposition of retinal metabolism by-products
3Increased intraocular pressure due to a defect in the drainage of aqueous humor
4lnflammation of the optic nerve
5Cholesterol plaque embolization
Answer 1: Sclerosis and narrowing of retinal vessels is seen in hypertensive retinopathy, not amaurosis
fugax.
Answer 2; Deposition of retinal metabolism byproducts, drusen, occurs in dry age-related macular
degeneration, not amaurosis fugax.
Answer 3: Increased intraocular pressure due to decreased drainage of aqueous humor is seen in acute
angle-closure glaucoma, which is a rare cause of amaurosis fugax but is not the most likely etiology.
Answer 4: Inflammation of the optic nerve is seen in optic neuritis, which can rarely cause amaurosis fugax
but is not the most likely etiology.
This patient experienced amaurosis fugax, a painless loss of vision in one eye. The most common cause of
amaurosis fugax is a cholesterol plaque emboli from a carotid artery plaque.

Blockage of the central retinal artery leads to sudden, painless, monocular vision loss due to retinal
hypoxia. Vision loss is classically described as a curtain coming down over one eye. Amaurosis fugax
occurs if the clot passes and the vision loss is transient. If the clot cannot pass, central retinal artery
occlusion occurs. A fundus exam may reveal optic disk pallor, a cherry-red macula, and retinal edema. If it
does not resolve spontaneously, treatment is recommended within an hour of the occlusion. Treatment
involves surgical decompression, but, if unavailable, digital massage of the globe and CO2 rebreathing
should be initiated in an attempt to pass the clot.

Pokhrel et al. describe the recognition and treatment of ocular emergencies. Common ocular emergencies
include penetrating injuries, central retinal artery occlusion, retinal detachment, acute angle-closure
glaucoma, and chemical burns. They state that risk factors for central retina artery occlusion include age
greater than 70, diabetes, endarteritis, atherosclerosis, glaucoma, hypercholesteremia, and hypertension.
Following presentation and treatment, patients with central retinal artery occlusion should receive carotid
ultrasonography in order to identify possible carotid plaques or stenosis.

Liozon et al. discuss the relationship between central retinal artery occlusion and giant cell arteritis.
Fourteen to twenty percent of patients with giant cell arteritis will develop permanent vision loss from either
anterior ischemic optic neuropathy or retinal artery occlusion. Vision loss can be prevented by early
identification and treatment of giant cell arteritis with glucocorticoids. Amaurosis fugax may also be a sign
of giant cell arteritis.

152- Patient presented with acute glaucoma what is the first thing you should give:
1. Oral acetazolamide
2. Topical timolol
3. Topical pilocarpine
4. topical acetazolamide
5. Topical atropine

Only for Study purpose / Only Notes / AMC MCQ


Only Study Notes / Only for Study purposes / Personal notes/ AMC MCQ

153- ou r in a rural area and 4 cases of trachoma come to you then you find out extra 20 how can you treat
acute indexes?
a-handwashing
b-azithromycin
c-doxycyclin
d-don’tdoanything
e-penicillin

Antibiotic selection — We suggest azithromycin (single dose 20 mg/kg orally, maximum 1 g) for community-
wide distribution. Azithromycin is preferred since it is administered as a single oral dose; it is acceptable for
use in children and for pregnant women
Topical tetracycline (1% eye ointment twice daily for six weeks) can be used an alternative, but adherence
rates are likely to be very low

154 - It was something like this.. 5 kids in a community found with Trachoma...20 household contacts were
related to these kids.. it was a small community of total 240 ppl. How to manage this outbreak?
1- treat kids
2- treat kids and household
3- treat whole community
4- prophylactic abx for household
5- prophylactic abx for community
Contro still

155 -

Ans B

Only for Study purpose / Only Notes / AMC MCQ


Only Study Notes / Only for Study purposes / Personal notes/ AMC MCQ

Only for Study purpose / Only Notes / AMC MCQ


Only Study Notes / Only for Study purposes / Personal notes/ AMC MCQ

156-

Ans” A
Right direct pupillary reflex is the right pupil's response to light entering the right eye, the ipsilateral
eye. Right consensual pupillary reflex is the right pupil's indirect response to light entering the left eye,
the contralateral eye.

Direct response (pupil illuminated).


The direct response is impaired in
lesions of the ipsilateral optic nerve,
the pretectal area, the ipsilateral
parasympathetics traveling in CN III,
or the pupillary constrictor muscle of
the iris.

Consensual response (contralateral


pupil illuminated). The consensual
response is impaired in lesions of the
contralateral optic nerve, the pretectal
area, the ipsilateral parasympathetics
traveling in CN III, or the pupillary
constrictor muscle.

Accommodation (response to looking


at something moving toward the eye).
Accommodation is impaired in lesions
of the ipsilateral optic nerve, the
ipsilateral parasympathetics traveling
in CN III, or the pupillary constrictor
muscle, or in bilateral lesions of the
pathways from the optic tracts to the
visual cortex. Accommodation is
spared in lesions of the pretectal
area.

Only for Study purpose / Only Notes / AMC MCQ


Only Study Notes / Only for Study purposes / Personal notes/ AMC MCQ

Only for Study purpose / Only Notes / AMC MCQ


Only Study Notes / Only for Study purposes / Personal notes/ AMC MCQ

157-
Ans D

Only for Study purpose / Only Notes / AMC MCQ


Only Study Notes / Only for Study purposes / Personal notes/ AMC MCQ

Only for Study purpose / Only Notes / AMC MCQ


Only Study Notes / Only for Study purposes / Personal notes/ AMC MCQ

158 -

Only for Study purpose / Only Notes / AMC MCQ


Only Study Notes / Only for Study purposes / Personal notes/ AMC MCQ

Only for Study purpose / Only Notes / AMC MCQ


Only Study Notes / Only for Study purposes / Personal notes/ AMC MCQ

159 -A 15-year-old male presents to his primary care doctor for "eye pain." He states that he has noticed
his left eye hurting more over the past four days. He normally wears disposable contact lenses daily but
could not wear them because of the heightened sensation of something foreign in his eye. He has noticed
occasional sensitivity to light. He denies any headaches, changes in vision, or pain with eye movement. He
further denies any frank ocular or oral discharge but feels like something might pop if he squeezes hard
enough. An image of his left eye is seen in Figure A. The right eye is completely healthy appearing. What is
the most likely diagnosis?
1
Canaliculitis
2
Chalazion
3
Dacryocystitis
4
Hordeolum
5
Preseptal cellulitis

This vignette and image reveal a


patient with eyelid redness,
swelling, tenderness. The diagnosis
is hordeolum.

A hordeolum, or stye, is inflammation of the external sebaceous gland of the eyelid caused most commonly
by Staphylococcus aureus. Symptoms and signs for the first 1-2 days are indistinguishable from a
chalazion, with redness, edema, and pain localized to the conjunctiva. Tenderness continues through 2 to 4
days during which the hordeolum may rupture with pustular discharge. Recurrence is common.

Figure A shows an image of a hordeolum. Note the faint hyperemia and contained swelling suggesting
obstruction of a gland.

Incorrect Answers:
Answer 1: Canaliculitis is inflammation of the punctum and canaliculus, caused most commonly by
Actinomyces israelii, often from a retained punctal plug.

Answer 2: Chalazion is non-infectious obstruction of a meibomian gland that causes extravasation of


irritating lipid material in the eyelid soft tissues with focal secondary granulomatous inflammation. The
nodule that forms is typically nontender with decreased inflammation (less redness and edema) after the
first 1-2 days.

Answer 3: Dacryocystitis is inflammation of the lacrimal sac. The punctum and canaliculus are normal.

Answer 5: Preseptal cellulitis is inflammation of the eyelid and skin surrounding the eye anterior to the
orbital septum.

Bullet Summary
Hordeolum is inflammation of the external sebaceous gland over the upper or lower eyelid due to S.
aureus.

Only for Study purpose / Only Notes / AMC MCQ


Only Study Notes / Only for Study purposes / Personal notes/ AMC MCQ

160- A 3-year-old female patient is brought to the clinic by her parents. They are concerned about their
daughter's right eye (Figure A). What is appropriate management for this patient?
1
Close follow-up without intervention
2
Corrective surgery
3
Patching of left eye
4
Counsel parent on caring for child with vision loss
5
Patching of right eye

This patient has amblyopia, which can be appropriately


managed with total occlusion (patching) of the normal
left eye. Correction of refractive error should be performed before initiating amblyopia treatment.

Amblyopia is defined as decreased vision in one or both eyes not due to any obvious physical deformities.
It does not resolve upon correction of any refractive error. Decreased visual acuity in one or both eyes can
lead to visual cortex suppression and lifelong irreversible vision impairment.

Bradfield discusses the types and treatment of amblyopia. There are five main types of amblyopia: 1.
strabismic, 2. visual stimuli deprivation, 3. anisometropic (refractive error in both eyes), 4. ametropic
(severe symmetric refractive error in both eyes), 5. meridional (astigmatism of both eyes). Treatment
should include correction of refractory error first, followed by either patching of normal eye or optic
penalization with atropine. Blurred contact lenses is an alternative treatment.

Taylor et al. discuss the efficacy of various interventions for strabismic amblyopia. This study found that
occlusion (i.e. patching) along with refractory error correction is more effective than refractory error
correction alone. Near activities (i.e. doing homework, coloring) may also offer additional benefit.

Figure A demonstrates a female infant with esotropia of the right eye.

Incorrect Answers:
Answer 1: Lack of intervention may result in decreased vision or vision loss, permanently.
Answer 2: Surgery can be performed, only after amblyopia is reversed.
Answer 4: Vision loss is not unavoidable in this patient; proper treatment can prevent sequelae from
amblyopia.
Answer 5: Patching of the abnormal eye would be inappropriate.

Only for Study purpose / Only Notes / AMC MCQ


Only Study Notes / Only for Study purposes / Personal notes/ AMC MCQ

Very simply, Strabismus, the medical term for "crossed-eye", is a problem with eye alignment, in which
both eyes do not look at the same place at the same time. Amblyopia, the medical term for "lazy-eye", is a
problem with visual acuity, or eyesight. Many people make the mistake of saying that a person who has a
crossed or turned eye (strabismus) has a "lazy-eye," but lazy-eye (amblyopia) and strabismus are not the
same condition.

Treatment of strabismus usually involves glasses, eye muscle exercises and/or surgery.

Only for Study purpose / Only Notes / AMC MCQ


Only Study Notes / Only for Study purposes / Personal notes/ AMC MCQ

One conventional treatment for


amblyopia has been to place an
adhesive patch on the good eye

Strabismus (crossed eyes or eyes that turn out) is a misalignment caused when one or more of the six
muscles controlling the eyes fail to work properly. It prevents the eyes from working together, initially
causing vision impairment, double vision or faulty depth perception. Strabismus occurs in two to five
percent of all children. Unable to outgrow it, children may lose vision in one eye, which then succumbs to
amblyopia if the root cause is not treated in early childhood
Amblyopia, also called lazy eye, is a disorder of sight in which the brain fails to process inputs from
one eye and over time favors the other eye.[1] It results in decreased vision in an eye that otherwise typically
appears normal.[1] Amblyopia is the most common cause of decreased vision in a single eye among children and
younger adults.[

Strabismus can occur due to muscle dysfunction, farsightedness, problems in the brain, trauma or
infections.[3] Risk factors include premature birth, cerebral palsy and a family history of the condition.[3] Types
include esotropia, where the eyes are crossed ("cross eyed"); exotropia, where the eyes diverge ("lazy eyed" or
"wall eyed"); and hypertropia where they are vertically misaligne
Strabismus is a condition in which the eyes do not properly align with each other when looking at an
object.[2] The eye that is focused on an object can alternate.[3] The condition may be present occasionally or
constantly.[3] If present during a large part of childhood, it may result in amblyopia or loss of depth perception.[3] If
onset is during adulthood, it is more likely to result in double vision.

161- A 37-year-old machinist presents to his primary care physician with eye problems. The patient states
that he has had a mass in his eye that has persisted for the past month. The patient has a past medical
history of blepharitis treated
with eye cleansing and
squamous cell carcinoma of
the skin treated with Mohs
surgery. His temperature is
99.5°F (37.5°C), blood
pressure is 157/102 mmHg,
pulse is 90/min, respirations
are 17/min, and oxygen
saturation is 98% on room air.
Physical exam is notable for a
firm and rubbery nodule
palpable inside the patient's
left eyelid. Physical exam does

Only for Study purpose / Only Notes / AMC MCQ


Only Study Notes / Only for Study purposes / Personal notes/ AMC MCQ

not elicit any pain. Which of the following is the most likely diagnosis?
1
Chalazion
2
Foreign body
3
Hordeolum
4
Ingrown eyelash follicle
5
Meibomian cell carcinoma

his patient is presenting with symptoms suggestive of a chalazion.

Chalazion presents with a hard and painless lid nodule. This pathology occurs secondary to granulomatous
inflammation of the Meibomian gland. The lesion can be treated with simple excision; however, recurrent
lesions are concerning for Meibomian cell carcinoma. The most important differential diagnosis to consider
in these patients is a hordeolum which presents with a firm and painful lid nodule (in contrast to painless).

Incorrect Answers:
Answer 2: Foreign body is a possible diagnosis given this patient's occupation as a machinist; however, this
would typically present with pain over the patient's cornea with a sudden onset of symptoms.

Answer 3: Hordeolum presents with a hard and painful lid nodule in contrast to a hard and painless lid
nodule.

Answer 4: Ingrown eyelash follicle could present with a firm lid


nodule; however, it would be located near the margin of the
eyelid and would likely be inflamed and tender.

Answer 5: Meibomian cell carcinoma is a possible diagnosis but


is epidemiologically less common than a chalazion. This
diagnosis would be more likely if the lesion was recurrent
despite excision.

Bullet Summary:
Chalazion presents with a firm and painless eyelid nodule.

Only for Study purpose / Only Notes / AMC MCQ


Only Study Notes / Only for Study purposes / Personal notes/ AMC MCQ

one thing we should keep in mind if they gave a history of skin cancer and treated by
radiation which is a strong cause of sebaceous gland carcinoma and specifically mentioned it's
painless lesion...we must think about MGC.

162 - A 47-year-old man presents to his ophthalmologist for abnormal vision. The patient was
discharged from the hospital 2 weeks ago after a motor vehicle accident where he was given 1L of
blood. He also underwent several surgical procedures including enucleation of the right globe and
exploratory abdominal laparotomy to stop internal bleeding. Currently, he states that he has noticed
decreased vision in his left eye. The patient has a past medical history of poorly controlled diabetes,
asthma, and atopic dermatitis. His temperature is 98.9°F (37.2°C), blood pressure is 167/108 mmHg, pulse
is 90/min, respirations are 14/min, and oxygen saturation is 97% on room air. Physical exam is notable for

Only for Study purpose / Only Notes / AMC MCQ


Only Study Notes / Only for Study purposes / Personal notes/ AMC MCQ

20/200 vision in the patient's left eye, which is worse than usual. The patient's left eye appears red and
irritated. Which of the following is the most likely diagnosis?
1
Anterior uveitis
2
Diabetic retinopathy
3
Normal post-operative visual adaptations
4
Seasonal allergies
5
Sympathetic ophthalmia
This patient is presenting with blurry vision and a red eye after trauma to the contralateral eye requiring
enucleation, suggesting a diagnosis of sympathetic ophthalmia.

Sympathetic ophthalmia is a granulomatous uveitis that typically occurs after trauma to one globe. The
exposed antigens of the injured globe lead to autoimmune injury to the other globe. Patients usually
present with blurry vision, a red eye, and decreased vision. Prevention is treatment/enucleation of the
injured globe (to stop antigen exposure and protect the other globe) within 2 weeks (though it does not
have a 100% success rate in preventing the condition), and treatment of the acute condition includes
corticosteroids and immunomodulators to protect the other eye.
Sympathetic ophthalmia is currently thought to be an autoimmune inflammatory response toward
ocular antigens, specifically a delayed hypersensitivity to melanin-containing structures from the outer segments
of the photoreceptor layer of the retina. The immune system, which normally is not exposed to ocular proteins, is
introduced to the contents of the eye following traumatic injury

Incorrect Answers:
Answer 1: Anterior uveitis presents very similarly to sympathetic ophthalmia; however, in the setting of
previous trauma and enucleation, sympathetic ophthalmia is a more descriptive diagnosis.

Answer 2: Diabetic retinopathy would be more associated with a chronic loss of vision and retinal
abnormalities in a poorly controlled diabetic.

Answer 3: Normal post-operative visual adaptations would not include symptoms of worsening vision and a
red eye.

Answer 4: Seasonal allergies would present with bilateral red/teary eyes that are seasonal. This diagnosis
is not associated with poor vision.

Bullet Summary:
Sympathetic ophthalmia occurs after trauma to one eye where the exposed antigens of the injured eye lead
to the autoimmune injury to the other eye causing blurry vision, a red eye, and decreased vision.

Only for Study purpose / Only Notes / AMC MCQ


Only Study Notes / Only for Study purposes / Personal notes/ AMC MCQ

163-A 22-year-old man presents to his primary care physician with itchy eyes. The patient recently moved
to the northern US and states that he has experienced bilateral itching and watering of his eyes. He states
that it is very distracting and is making it hard for him to work. The patient denies any recent sick contacts
and lives alone. He works as a machinist and is currently struggling with his finances. The patient has a
past medical history of asthma, and his current medications include albuterol, fluticasone, fish oil, and whey
protein supplements. His temperature is 99.5°F (37.5°C), blood pressure is 100/65 mmHg, pulse is 60/min,
respirations are 15/min, and oxygen saturation is 99% on room air. Cardiopulmonary exam is within normal
limits. You notice the bilateral findings seen in Figure A. Otoscopy reveals bilateral clear tympanic
membranes. The rest of the patient's physical exam is within normal limits. Which of the following is the
most likely
diagnosis?
1
Viral
conjunctivitis
2
Bacterial
conjunctivitis
3
Allergic
conjunctivitis
4
Corneal
abrasion
5
Foreign body

This patient is
presenting with bilateral itching and watering of his eyes with the absence of other symptoms/exposures
suggesting allergic conjunctivitis as the most likely diagnosis.

Allergic conjunctivitis occurs when an individual is exposed to an antigen that leads to mast cell
degranulation. Patients can present with bilateral tearing, pruritis, and hyperemia. The best treatment for
allergic conjunctivitis is allergen avoidance as well as non-sedating anti-histamines such as loratadine or
fexofenadine.

Figure A demonstrates the physical exam findings of hyperemia and tearing which are commonly seen in
allergic conjunctivitis.

Incorrect Answers:
Answer 1: Viral conjunctivitis presents similarly to allergic conjunctivitis with bilateral tearing, pruritis, and
hyperemia. However, given this patient's history of atopy, recent migration to a new location, lack of other
physical exam findings, and absence of systemic symptoms, allergic conjunctivitis is a more likely
diagnosis. Viral conjunctivitis is a strong possibility and should be on the differential.

Answer 2: Bacterial conjunctivitis presents with pain and a purulent discharge from the eye, in contrast to
this patient's watery discharge and hyperemia.

Answer 4: Corneal abrasion presents with pain in the eye with or without visual disturbance. This patient's
career as a machinist puts him at risk for corneal abrasion; however, his bilateral allergic symptoms
suggest a diagnosis of allergic conjunctivitis.

Answer 5: Foreign body in the eye presents with eye pain and a chief complaint of a sensation of an object

Only for Study purpose / Only Notes / AMC MCQ


Only Study Notes / Only for Study purposes / Personal notes/ AMC MCQ

in the eye.

Bullet Summary:
Allergic conjunctivitis presents with bilateral tearing, pruritis, and hyperemia.

164- A 70-year-old woman presents to an ophthalmology clinic for bilaterally blurred vision. Her symptoms
have progressed gradually over several years, but she recently became concerned when she noticed that
she could no longer read her newspaper without the help of a magnifying glass. She also mentions that her
eyes have been “playing tricks on her,” pointing out that the edges of the examination table appear curved,
when she knows they are usually straight. Physical exam is unremarkable. Fundoscopic exam (shown in
Figure A) is similar in both eyes. Which of the following is the most likely diagnosis?
1
Atrophic macular degeneration
2
Cataracts
3
Exudative macular degeneration
4
Open angle closure glaucoma
5
Retinal vein occlusion
This elderly patient with gradually
worsening bilateral vision loss,
metamorphosia, and fundoscopic exam
revealing drusen in the macula, likely has
macular degeneration (MD) -
specifically, atrophic ("dry") MD.

MD is a common cause of vision loss in the elderly. In MD, the macula (the central portion of the retina)
deteriorates, causing painless blurring/loss central vision, while peripheral vision is spared. A discrete
central spot of vision loss (central scotoma) may be present. Distortion of lines may occur early on (straight
lines appear curved, known as metamorphosia). There are two forms of MD: atrophic ("dry") MD and
exudative ("wet") MD. Atrophic MD is the most common, and presents with mild symptoms that progress

Only for Study purpose / Only Notes / AMC MCQ


Only Study Notes / Only for Study purposes / Personal notes/ AMC MCQ

gradually. Fundoscopic exam shows drusen (extracellular matrix deposits in the macula). Exudative MD is
caused by sub-retinal neovascularization. Its presentation is typically more acute and severe, with blood
vessel proliferation and hemorrhage seen on fundoscopic exam.
Amsler GRID

Figure A demonstrates a fundoscopic image of atrophic (dry) macular degeneration with drusen (yellow
spots in the macula).

Incorrect Answers:
Answer 2: Cataracts present with gradual loss of entire visual field (central and peripheral) and difficulty
with night vision. Lens opacification may be appreciated on physical exam.

Answer 3: Exudative macular degeneration presents with rapid, painless loss of central vision. Fundoscopic
exam shows proliferation of blood vessels and hemorrhage.

Answer 4: Open angle closure glaucoma presents with gradual, painless loss of peripheral vision.

Answer 5: Retinal vein occlusion resents with sudden, unilateral loss of entire visual field (central and
peripheral).

Bullet Summary:
Atrophic (“dry”) macular degeneration presents with gradual, painless loss of central vision with yellow
drusen deposits seen on fundoscopic exam

165-A 27-year-old woman presents to the emergency department with eye pain. She states that for the
past 2 days she has experienced severe eye pain and discharge from her eye. The patient has a past
medical history of type I diabetes and is currently taking insulin. Her temperature is 102°F (38.9°C), blood
pressure is 127/68 mmHg, pulse is 100/min, respirations are 17/min, and oxygen saturation is 98% on
room air. Physical exam is notable for the finding in Figure A. Examination of cranial nerve III elicits
pain. Which of the following is the most likely diagnosis?
1
Infection anterior to the orbital septum
2
Infection posterior to the orbital septum
3
Maxillary osteomyelitis
4
Meibomian gland infection
5
Thrombus within the cavernous sinus
This patient is presenting with erythema and edema of the orbit with
pain with extraocular movements suggesting a diagnosis of orbital
cellulitis (infection posterior to the orbital septum).

Orbital cellulitis presents similar to periorbital cellulitis with


inflammation, erythema, and edema of the eye. Key differentiating
factors between these 2 pathologies are that orbital cellulitis
presents with impairment of cranial III, pain with extraocular movements, proptosis, and vision impairment,
whereas periorbital cellulitis does not. The extent of the infection should be assessed with a CT scan of the
head and IV antibiotics should be started.

Only for Study purpose / Only Notes / AMC MCQ


Only Study Notes / Only for Study purposes / Personal notes/ AMC MCQ

Incorrect Answers:
Answer 1: Infection
anterior to the orbital
septum is periorbital
cellulitis and would
not present with pain
with extraocular
movements.

Answer 3: Maxillary
osteomyelitis would
be less likely to
present with orbital
symptoms and
periorbital edema.

Answer 4: Meibomian gland infection or a stye would present with symptoms limited to the eyelid.

Answer 5: Thrombus within the cavernous sinus describes cavernous sinus thrombosis which would
present with severe symptoms such as pain, diplopia, and headache with a much more concerning clinical
picture.

Bullet Summary:
Orbital cellulitis (infection posterior to the orbital septum) presents with inflammation, erythema, pain with
extraocular movements, proptosis, and vision impairment.

166- A 52-year-old female presents to the emergency room with severe right eye pain and vision loss. The
symptoms started a couple hours ago while she was out to a late dinner. She denies ever feeling these
symptoms before. In addition to the pain and blurry vision, she reports excessive tearing. She denies
discharge from her eye. The patient’s past medical history is significant for hypertension and rheumatoid
arthritis. Her medications include hydrochlorothiazide and sulfasalazine. She has a 40 pack-year smoking
history. She works as a pre-school teacher at a local school. Figure A shows physical examination findings
of both eyes. The right pupil is less reactive to light in comparison to the left pupil. Figure B shows a
fundoscopic exam of the right eye. Fundoscopy of the left eye is unremarkable. Which of the following is
the most accurate test to diagnose the patient’s condition
1
Culture and gram stain

2
Fluoroscein stain

Only for Study purpose / Only Notes / AMC MCQ


Only Study Notes / Only for Study purposes / Personal notes/ AMC MCQ

3
HLA-B27 test

4
Gonioscopy

5
Tonometry

This patient is presenting with sudden-


onset eye pain and optic disc cupping,
suggesting the diagnosis of acute
angle-closure glaucoma. The most
accurate test is gonioscopy.

Acute angle-closure glaucoma


presents as acute-onset, severe, eye pain and blurred vision. It may be associated with a same-side
orbitofrontal headache, conjunctival injection, excessive lacrimation, nausea, and vomiting. An acute
episode usually occurs following pupillary dilation such as being in a dark movie theater or at nighttime.
Examination of the eye reveals a red eye that is rock-hard and a dilated pupil (Figure A) that is non-reactive
to light. Fundoscopic exam will show increased optic cup-to-disc ratio (>0.4) (Figure B). Tonometry will
show increased intraocular pressure, and is the best initial test. Gonioscopy allows angle evaluation, and is
the diagnostic gold-standard.

Figure A shows redness and a mid-sized pupil of the right eye, which can be seen with acute angle closure
glaucoma. Figure B shows a fundoscopic exam with inferior temporal disc rim thinning, evident by the color
change and vessel distortion, consistent with glaucoma.

Incorrect Answers:
Answer 1: Culture and Gram stain would be the appropriate diagnostic test for bacterial conjunctivitis.
Bacterial conjunctivitis presents as a red eye with purulent discharge, usually with minimal vision changes,
pupillary changes or pain.

Answer 2: Fluoroscein stain can be used for corneal surface abnormalities, such as corneal abrasions or
herpes simplex keratoconjunctivitis. Herpes simplex keratoconjunctivis presents as a painful, tearing eye
and is often precipitated by the use of steroids. It would not be associated with an increased optic disc ratio.

Answer 3: HLA-B27 test may be positive in uveitis caused by autoimmune-disorders, such as ankylosing
spondylitis, juvenile rheumatoid arthritis, inflammatory bowel disease, sarcoidosis, and psoriasis. Uveitis
presents as a painful, red eye, severe photophobia, decreased vision and miotic pupils.

Answer 5: Tonometry can be used to show increased intraocular pressure, and is the best initial test for
acute angle-closure glaucoma. However, it is not the most accurate diagnostic test.

Bullet Summary:
The best initial test for acute angle-closure glaucoma is ocular tonometry, while the most accurate test is
gonioscopy.

167- A 71-year-old female arrives to your clinic complaining of sudden vision loss. She reports that she had
been in her usual state of health, until waking up this morning unable to see out of her right eye. She

Only for Study purpose / Only Notes / AMC MCQ


Only Study Notes / Only for Study purposes / Personal notes/ AMC MCQ

denies pain, but reports her right eye “feels funny.” The patient’s medical history is significant for a previous
myocardial infarction, hypertension, and osteoporosis. Her current medications include aspirin, metoprolol,
rovustatin, lisinopril, and alendronate. The patient’s temperature is 98°F (36.7°C), blood pressure is 145/86
mmHg, pulse is 62/min, and respirations are 12/min with an oxygen saturation of 98% O2 on room air. On
physical exam, the patients pupils are symmetrical in size and equally reactive to light with accommodation.
Fundoscopic exam of the right eye is shown in Figure A. Left optic fundus is unremarkable. Of the
following, what is the most likely diagnosis?
1
Retinal detachment

2
Retinal vein occlusion

3
Central retinal artery occlusion

4
Open-angle glaucoma

5
Papilledema
This patient is presenting with
sudden, painless, unilateral vision
loss with a fundoscopic exam
positive for venous dilation and
retinal hemorrhage, suggesting the diagnosis of retinal vein occlusion.

Retinal vein occlusion can occur in either central or branch retinal veins and is often caused by
compression from nearby arterial atherosclerosis. It presents as acute onset, painless vision loss, normally
in one eye. On fundoscopic exam (see Figure A), venous engorgement, retinal hemorrhage, cotton wool
exudates, and edema/swelling of the optic disk can be seen. There is no definitive treatment. Retinal vein
occlusion may resolve spontaneously or progress to permanent vision loss.

Figure A is fundoscopy showing engorged retinal


veins, optic disk swelling, and widespread retinal
hemorrhages, characteristic of central retinal vein
occlusion. Illustration A is fundoscopy showing a
crinkled appearing retina, characteristic of retinal
detachment.

Incorrect Answers:
Answer 1: Retinal detachment presents as
monocular loss of vision, often described as a
“curtain pulled down,” and preceded by “flashers”
and “floaters.” On fundoscopic exam (Illustration A),
crinkling of the retinal tissue and abrupt changes in
vessel direction can be seen.

Answer 3: Central retinal artery occlusion can present as sudden onset, unilateral vision loss. However,
fundoscopic exam will show a “cherry-red” spot at the fovea and a “fuzzy” retina with attenuated vessels.
Embolic sources may include carotid artery atherosclerosis or cardiac vegetations.

Only for Study purpose / Only Notes / AMC MCQ


Only Study Notes / Only for Study purposes / Personal notes/ AMC MCQ

Answer 4: Acute closed-angle glaucoma presents as sudden, painful, vision loss caused by increased
intraocular pressure. The eye will appear red and often feel “hard.” The optic disc will show characteristic
cupping.

Answer 5: Papilledema may present as an enlargement in blind spot and is usually bilateral, as it is caused
by increased intracranial pressure. On fundoscopic exam, there will be bilateral optic disk swelling, seen as
an elevated optic disk and blurred margins.

Bullet Summary:
Retinal vein occlusion presents with a sudden, monoocular, painless loss of vision with optic disk swelling,
dilated veins, cotton wool spots, and retinal hemorrhages.

168-A 63-year-old man presents to the ophthalmologist with the complaint of sudden, painless visual loss in
his right eye. He notes that in the previous weeks, he started to notice new floaters and shadows in his field
of vision. He underwent cataract surgery in both eyes last year. A dilated ophthalmologic examination is
performed and reveals the retinal findings seen in Figure A. Which of the following conditions is the most
likely cause of the development of this patient’s visual loss?
1
Central retinal artery occlusion
2
Closed-angle glaucoma
3
Diabetic retinopathy
4
Open-angle glaucoma
5
Recent cataract surgery

This patient with the complaint of painless visual


loss, new-onset floaters and shadows, and a fundus
examination showing hemorrhage is likely suffering
from vitreous hemorrhage, which is commonly
caused by diabetic retinopathy.

Vitreous hemorrhage is the extravasation of blood into the vitreous cavity, generally via 2 mechanisms: 1)
hemorrhage from pathological structures, such as retinal neovascularization seen in proliferative diabetic
retinopathy, or 2) rupture of normal blood vessels through trauma, such as from a globe injury or a retinal
tear. It commonly presents a sudden, painless visual loss. Patients may describe their vision having a “red
hue” and the presence of new-onset floaters and shadows. Patients with poorly-controlled diabetes who
develop proliferative diabetic retinopathy are at high risk for vitreous hemorrhage.

Figure A shows the retina of a diabetic patient with vitreous hemorrhage.

Incorrect Answers:
Answer 1: Central retinal artery occlusion presents with sudden, painless visual loss but does not cause
vitreous hemorrhage.

Answer 2: Closed-angle glaucoma presents with painful visual loss with a hard, red eye and a pupil that is
dilated and non-reactive to light. Closed-angle glaucoma does not cause vitreous hemorrhage.

Only for Study purpose / Only Notes / AMC MCQ


Only Study Notes / Only for Study purposes / Personal notes/ AMC MCQ

Answer 4: Open-angle glaucoma presents with gradual loss of peripheral vision, with cupping of the optic
nerve head seen on funduscopic examination. Open-angle glaucoma does not cause vitreous hemorrhage.

Answer 5: Recent cataract surgery is not a risk factor for vitreous hemorrhage, as the surgery is performed
in the anterior chamber of the eye.

Bullet Summary:
A common cause of vitreous hemorrhage is proliferative diabetic retinopathy.

169- A 32-year-old construction worker presents to the emergency room with severe right eye pain. He
states that the pain started two days ago while he was drilling. He has been experiencing severe pain,
redness, and watering of the right eye since then. His eye is shown in Figure A. Bacterial culture will most
likely reveal which organism?
1
Streptococcus viridans
2
Coagulase-negative staphylococcus
3
Neisseria gonorrhoeae
4
Staphylococcus aureus
5
Streptococcus epidermidis

This patient presents with a corneal foreign body. The


most common organsim cultured from corneal foreign
bodies is coagulase negative staphylococcus.

A corneal foreign body is an eye trauma due to the


introduction of a particle into the corneal epithelium or
stroma. Intraocular foreign bodies are common and often
caused by the operation of drills, hammers, or other
tools. The presence of a foreign body may lead to an inflammatory response, causing vasodilation and
swelling of the surrounding lid and conjunctiva. Intraocular foreign bodies should be removed surgically by
an ophthalmologist. Systemic and topical antibiotic treatment is recommended.

Peate reviews work-related ocular injuries. Common work-related eye illnesses and injuries include blunt
trauma, conjunctivitis, corneal abrasions, foreign bodies, and chemical burns. Ocular foreign bodies often
co-occur with corneal abrasions. While superficial foreign bodies may be removed with a wet cotton swab,
deeper foreign bodies require removal under a slit lamp or by an ophthalmologist. Rust rings are due to rust
residue from the oxidation of a ferrous foreign body.

Wipperman and Dorsh describe the diagnosis and treatment of corneal abrasions. Corneal abrasions are
due to damage of the cornea and can present with pain, tearing, photosensitivity, and a foreign body
sensation. Diagnosis can be confirmed by fluorescein staining of the cornea, which will show the abrasion
as green under cobalt blue light. Treatment includes pain control with topical NSAIDs or oral analgesics
and topical antibiotics.

Figure A shows a patient with an iron fragment lodged in the cornea.

Incorrect answers:

Only for Study purpose / Only Notes / AMC MCQ


Only Study Notes / Only for Study purposes / Personal notes/ AMC MCQ

Answers 1, 3, 4, and 5: The most common organisms cultured from corneal foreign bodies are coagulase
negative staphylococcus.

170- A 22-year-old woman with no significant past medical history presents with right eye pain, blurry
vision, and watery discharge. She had an uncomplicated urinary tract infection for which she has been
receiving TMP-SMX. She denies any problems with her left eye. On physical examination, both pupils are
equal, round, and reactive to light and accommodation. Intraocular pressure is normal and equal bilaterally.
Visual fields are full. However, visual acuity in the right eye is 20/60 compared to 20/20 in the left eye. Slit
lamp exam of the left eye reveals the following seen in Figure A. Which of the following is true?
1
The treatment is with glucocorticoid drops and the diagnosis is Map-Dot-Fingerprint dystrophy
2
Laboratory and serologic testing should
be used to confirm the diagnosis
3
The patient should be admitted for
intravenous antibiotics
4
The diagnosis is Stevens-Johnson
syndrome and the offending drug
should be stopped
5
The patient may be treated with oral
acyclovir
Herpes simplex virus keratitis presents
as an infection of the superficial cornea
with diffuse dendritic lesions on the
corneal epithelium. The treatment is with oral acyclovir.

In herpes simplex virus keratitis, patients may present with pain, blurry vision, and watery discharge. Ocular
manifestations of HSV-1 are not limited to keratitis but may also be conjunctivitis, blepharitis, retinitis, and
scleritis so the remainder of the eye should be examined. Corneal pathology, however, is the most often.
The treatment of HSV keratitis is dependent upon whether the episode of disease is caused by active viral
replication or immune response to past infection. Oral and topical antivirals are equally effective, but oral
agents are more convenient to use.

Usatine and Tinitgan discuss nongenital herpes simplex virus. HSV-1 is usually transmitted during
childhood during nonsexual contact. Most of these infections involve the oral mucosa or lips (herpes
labialis). Other nonoral herpes simplex virus type 1 infections include herpetic keratitis, herpetic whitlow,
herpes gladiatorum, and herpetic sycosis of the beard area. In cases of herpatic keratitis where treatment is
delayed, scarring of the cornea may occur.

Rowe et al. discuss HSV-1 ocular infection. More often HSV-1 ocular infections result from reactivation of
virus that originally established a latent infection in the TG following a non-ocular route of infection.
Reactivation of latent virus in the ophthalmic branch of the TG can result in shedding at the corneal surface.
HSV-1 corneal lesions can either be restricted to the corneal epithelium, or have stromal involvement with
or without damage to the overlying epithelium.

Figure A depicts the dendritic pattern in HSV keratitis on exam. Illustration A shows the classic appearance
of Map-Dot-Fingerprint dystrophy. Illustration B shows orbital cellulitis. Illustration C shows a keratinized
and vascularized cornea as a result of Stevens-Johnson Syndrome.

Only for Study purpose / Only Notes / AMC MCQ


Only Study Notes / Only for Study purposes / Personal notes/ AMC MCQ

Incorrect Answers:
Answer 1: Applying topical glucocorticoid drops without the use of antiviral medication could worsen the
infection and threaten vision.
Map-dot-fingerprint dystrophy occurs when the epithelium’s basement membrane develops abnormally
causing the epithelial corneal cells to not properly adhere to it. This results in recurrent epithelial erosions.
Sometimes on exam, the irregular basement membrane will form concentric lines in the central cornea that
resemble small fingerprints.
Answer 2: Laboratory testing is not needed to confirm the diagnosis. The diagnosis is most often made
clinically and in this case there is ample clinical evidence to suggest a dose of HSV keratitis. Further
serologic testing is often not helpful because of the high prevalence of exposure in the population.
Answer 3: Intravenous antibiotics would be indicated in orbital cellulitis.
Answer 4: Stevens-Johnson syndrome is marked by a drug reaction where the cornea may become
keratinized and vascularized leading to blindness. It is not
characterized by a dendritic pattern on corneal exami

Only for Study purpose / Only Notes / AMC MCQ


Only Study Notes / Only for Study purposes / Personal notes/ AMC MCQ

171- A 32-year-old man presents to the emergency department with a chief complaint of severe bleeding
from his eye. The patient states that this morning he woke up with a large amount of blood in his eye and
immediately came to the emergency department. He cannot recall the events that led up to this
presentation. He states that he went out last night and endorses using cocaine and alcohol. He cannot
recall any episodes of trauma or any other events. The patient has a past medical history of alcohol abuse,
pancreatitis, and suicidal ideation. His temperature is 99.5°F (37.5°C), blood pressure is 187/108 mmHg,
pulse is 100/min, respirations are 17/min, and oxygen saturation is 96% on room air. On physical exam,
you note a healthy young man and the finding seen in Figure A. Cardiopulmonary exam is within normal
limits. Cranial nerves II-XII are grossly intact and the patient's gait is stable. Which of the following is the
best next step in management?
1
CT scan
2
MRI
3
Observation
4
Tonometry
5
Ultrasound

This patient is presenting with a subconjunctival hemorrhage for which the next best step in management is
observation.

Subconjunctival hemorrhages present with bright red blood underneath the conjunctiva and over the sclera.
They are typically precipitated by trauma or forceful straining (such as bowel movements and vomiting) but
can also occur spontaneously. In patients who present with intact vision and neurologic function, the best
next step in management is observation as these hemorrhages spontaneously resolve.

Figure A demonstrates the classic findings of a subconjunctival hemorrhage with bleeding localized over a
portion of the sclera underneath the conjunctiva.

Incorrect Answers:
Answers 1-2: CT scan or MRI of the head would be unnecessary for a simple conjunctival hemorrhage.
These tests may be indicated if their is cranial nerve dysfunction or penetrating trauma that needs to be
further evaluated.

Answer 4: Tonometry is typically indicated for conditions such as glaucoma or if you suspect an open
globe. A simple subconjunctival hemorrhage is not an indication for tonometry. Tonometry may be indicated
if you suspect an open globe secondary to severe trauma which also may present with a subconjunctival
hemorrhage.

Answer 5: Ultrasound can evaluate the structure of the eye and search for foreign objects; however, it
would not be indicated for a simple subconjunctival hemorrhage.

Bullet Summary:
Simple subconjunctival hemorrhages resolve on their own and require no workup.

Only for Study purpose / Only Notes / AMC MCQ


Only Study Notes / Only for Study purposes / Personal notes/ AMC MCQ

172- A 22-year-old man presents to the emergency department with eye pain. His symptoms started
several days ago and have not been improving. The patient smokes cigarettes occasionally and is not
sexually active. His temperature is 98.0°F (36.7°C), blood pressure is 120/78 mmHg, pulse is 80/min,
respirations are 17/min, and oxygen saturation is 98% on room air. Physical exam with fluorescein is seen
in Figure A. Which of the following is the best next step in management?
1
Ceftriaxone
2
Difluprednate
3
Topical erythromycin
4
c
5
Warm compresses
This patient is presenting with
dendritic ulcers on fluorescein
stain suggesting a diagnosis
of herpes simplex virus (HSV)
keratitis which can be treated
with trifluridine eye drops.

HSV keratitis typically


presents with vision obstruction, eye pain, redness, photophobia, blurred vision, and tearing. There are
typically no physical exam findings other than impaired visual acuity and some redness of the eye. The best
diagnostic test for HSV keratitis is the fluorescein stain which will demonstrate corneal vesicles and
dendritic ulcers upon fluorescein stain of the eye. The treatment of HSV keratitis is acyclovir, trifluridine, or
other topical antiviral medications. It is important to note that an oral agent AND a topical agent must be
given for sufficient management. If the agent is acyclovir-resistant then foscarnet can be used.

Figure/Illustration A demonstrates a fluorescein stain with dendritic ulcers suggesting a diagnosis of HSV
keratitis with the classic pattern seen on stain outlined in red.

Incorrect Answers:
Answer 1: Ceftriaxone is the appropriate treatment of gonococcal conjunctivitis which presents with
purulent discharge from the eye that is typically more profuse or copious than chlamydia conjunctivitis. In
newborns, gonococcal conjunctivitis presents earlier than chlamydial conjunctivitis and with more severe
symptoms.

Answer 2: Difluprednate is a topical steroid that could be used in uveitis. Steroids must be used with
extreme caution when treating the eye. If given to a patient with HSV keratitis it could worsen the infection
and cause blindness.

Answer 3: Topical erythromycin can be used for bacterial conjunctivitis in adults and presents with
mucopurulent discharge from the eyes. An antibiotic would not treat a viral etiology of symptoms.

Answer 5: Warm compresses and symptomatic treatment in addition to washing linens is the appropriate
management of viral conjunctivitis which presents with conjunctival erythema and a thinner and less
purulent discharge from the eye. Viral conjunctivitis is very contagious and typically spreads to the other
eye but has a self-limited presentation. It is a very different diagnosis than HSV keratitis which is a viral
infection due to HSV that requires treatment.

Only for Study purpose / Only Notes / AMC MCQ


Only Study Notes / Only for Study purposes / Personal notes/ AMC MCQ

Bullet Summary
The treatment for herpes simplex virus keratitis is oral acyclovir and topical agents such as trifluridine or
vidarabine.

173-A 32-year-old physician is cleaning his pool when he splashes the hydrochloric acid in his left eye. He
feels immediate pain and burning. His eye starts to tear profusely, and he can barely open it. His medical
history is significant for psoriasis. He is farsighted and has glasses for reading and computer work. He uses
topical calcipotriene and topical triamcinolone as needed. His only surgery was a tonsillectomy as a child.
He is married and has one son who is healthy. His mother has Graves disease. He drinks a glass of wine
with dinner but denies tobacco or recreational drug use. Which of the following is the best initial step in
management?
1
Apply topical bacitracin
2
Call the patient’s ophthalmologist
3
Go to the emergency department immediately
4
Irrigate with alkali solution
5
Irrigate with tap water
The patient's left eye was exposed to acid. The best initial step in management is to irrigate the eye with a
neutral solution, such as tap water.

Acute chemical injuries of the eye are medical emergencies. Acidic chemicals, such as pool cleaning
solution or car battery fluid, can denature proteins in the cornea causing damage. Alkali chemicals, such as
household cleaning solutions and fertilizers, cause not only direct surface damage but may also penetrate
the cell membranes to cause injury to deeper globe structures. The first step in the management of
chemical exposures of the eye is irrigation with saline or tap water. Timely irrigation to flush out the
chemical and attempt to neutralize the pH of the eye is the most important step in treatment and in
preventing further damage.

Incorrect Answers:
Answer 1: Applying topical bacitracin may be part of the eventual treatment for a chemical injury of the eye.
However, irrigation of the eye should occur first followed by an evaluation by a physician and/or
ophthalmologist. An antibiotic ointment is usually used to prevent bacterial superinfection.

Answer 2: Calling the patient’s ophthalmologist would delay irrigating the eye and flushing out the chemical,
which should be the first immediate step in management.

Answer 3: Going to the emergency department, even if immediately after the exposure, would still delay
irrigation of the eye, which is the most important initial step in management.

Answer 4: Irrigating with alkali solution should not occur. Even though the patient had an acid exposure,
irrigating with an alkali chemical would cause additional damage.

Bullet Summary:
Chemical exposures to the eye should be treated immediately with tap water or saline irrigation without
delay, even before contacting or going to a medical professional.

Only for Study purpose / Only Notes / AMC MCQ


Only Study Notes / Only for Study purposes / Personal notes/ AMC MCQ

174-A 6-month-old male infant is brought to the clinic because the mother thinks that her child is "cross-
eyed." The mother's prenatal history was unremarkable. The child was delivered vaginally without
complications. His nutrition has been stellar, and his well-child visits documented appropriate reflexes and
milestones. The mother reports that the infant's eyes are constantly crossed, and she is concerned that her
child will need surgery. On physical exam, there is no white reflex seen with the ophthalmoscope in either
eye. The infant's eyes are shown in Figure A. What is the best next step in management?
1
Right medial rectus muscle elongation

2
Right lateral rectus muscle resection

3
Occlusion of the right eye

4
Occlusion of the left eye

5
Reassurance

This infant presents with evidence of strabismus. To minimize progression to amblyopia, the unaffected left
eye should be occluded.

Strabismus is ocular misalignment in one or both eyes, and it is seen in 3-5% of all children. Cases often
present with parental concern about a wandering eye, crossing eyes, or poor vision. If untreated, amblyopia
may occur, where the affected eye's visual acuity decreases due to lack of neural stimulation of that eye
and increased dependence on the unaffected eye. To "recruit," and encourage use of the affected eye, the
unaffected eye is patched or its vision decreased with the use of cycloplegic eye drops.

Figure A shows an infant with strabismus. The right eye is directed medially, otherwise called esotropia.

Incorrect Answers:
Answers 1-2: Right medial rectus muscle elongation or right lateral rectus muscle resection are invasive
procedures that may be necessary if visual acuity does not sufficiently improve in the right eye with non-
invasive interventions.

Answer 3: Occlusion of the right eye would worsen neurodevelopment of the right optic nerve and its
associated cortices, exacerbating the onset of amblyopia.

Answer 5: Reassurance is inappropriate management of strabismus as it could lead to decreased visual


acuity in the affected eye.

Bullet Summary:
Strabismus can lead to amblyopia and should be managed by occluding the unaffected eye or by
pharmacologically decreasing the unaffected eye's visual acuity in order to stimulate use of the affected eye

Only for Study purpose / Only Notes / AMC MCQ


Only Study Notes / Only for Study purposes / Personal notes/ AMC MCQ

As an alternative to patching the stronger eye can be


“penalized” by using atropine drops once a day to this eye.
These drops weaken the focusing mechanism of the eye so
reducing the close up vision to such an extent that the child’s
brain “chooses” the image form the amblyopic eye rather than
the blurred image from the stronger eye.
Penalisation of the better seeing eye with atropine drops or
ointment has been demonstrated to be as effective as
patching for moderate amblyopia (20/40 to 20/100; 6/12 to
6/30). It has also been used to successfully treat severe
amblyopia.

Only for Study purpose / Only Notes / AMC MCQ


Only Study Notes / Only for Study purposes / Personal notes/ AMC MCQ

175- A 6-year-old immigrant from West Africa is being evaluated after recently arriving to the United States.
He is observed to be rubbing and scratching at his eye. On physical exam, the eyelid is everted and the
finding is shown in Figure A. This condition is most likely to cause blindness through which mechanism?
1
Viral infection of the cornea
2
Bacterial infection of the cornea
3
Fungal infection of the cornea
4
Benign growth from conjunctiva onto the cornea
5
Neovascularization of the cornea

The patient in this vignette most likely has a trachoma, which does not actually infect the eye itself but
rather causes physical irritation of the cornea resulting in neovascularization and ultimately blindness.

Trachoma is an ocular manifestation of Chlamydia trachomatis serotypes A-C. Infection leads to eyelid
irritation, followed by neovascularization that results in the typical appearance of small, white nodules on
the everted eyelid. The gold standard of diagnosis is a Giemsa stain performed on conjunctival scrapings.
Treatment involves either topical tetracycline or oral azithromycin. More severe cases with corneal scarring
and eyelid inversion may be appropriate for surgical treatment.

Mishori et al. review the diagnosis and treatment of trachoma. Ocular infections with C. trachomatis can
manifest as neonatal conjunctivitis, adult conjunctivitis, or trachoma. Trachoma is caused by chronic
infection that leads to eyelid scarring and inversion; often, this results in misplacement of the eyelashes
such that they consistently scratch the cornea.

Sommer et al. review challenges in providing ophthalmological care in the developing world. They note that
trachoma is the leading cause of blindness in the world, affecting roughly 40 million people and causing
blindness in nearly 8 million people. They suggest mass-treatment distribution to areas with endemic
trachoma.

Figure A displays an example of a trachmoa. Note the small white nodules on the everted eyelid.

Incorrect Answers:
Answer 1: This is the mechanism of herpes simplex keratoconjunctivitis.
Answer 2: C. trachomatis infects the eyelid but does not directly infect the cornea.
Answer 3: Fungal infection is not the mechanism of trachoma formation.
Answer 4: This is the mechanism of pterygium formation.

176- A 77-year-old woman presents with pain and decreased vision in her right eye. She states that it
started yesterday and has been rapidly worsening. Her temperature is 99.0°F (37.2°C), blood pressure is
133/81 mmHg, pulse is 88/min, respirations are 15/min, and oxygen saturation is 98% on room air. Physical
exam is notable for the finding in Figure A. Which of the following is most likely to be found in this patient's
history?

Only for Study purpose / Only Notes / AMC MCQ


Only Study Notes / Only for Study purposes / Personal notes/ AMC MCQ

1
Atopic dermatitis
2
Blunt trauma
3
Cataract surgery
4
Chronic steroid use
5
Recent viral illness
This patient is presenting with pain
and decreased vision with a
hypopyon on physical exam which is
concerning for endophthalmitis.
Recent cataract surgery is a risk
factor for this condition.

Symptoms of endophthalmitis include eye pain, vision loss, swollen eyelids, and conjunctivitis. Physical
exam reveals the presence of a hypopyon, which is a layer of leukocytes in the anterior chamber. Initial
management involves aspiration for culture and intravitreal antibiotics (vancomycin and ceftazidime or
amikacin). Prognosis depends on the cause and severity of endophthalmitis; a good indicator of outcome is
visual acuity. Risk factors for endophthalmitis include anything that penetrates the eye that could inoculate
bacteria including recent surgery or penetrating trauma.

Figure/Illustration A shows an example of endophthalmitis with a hypopyon present (blue arrow).

Incorrect Answers:
Answer 1: Atopic dermatitis and eczema could be found in the history of a patient with allergic conjunctivitis
which presents with itchy and watery eyes with edema. Symptoms are bilateral and worsened with allergen
exposure.

Answer 2: Blunt trauma may cause a hyphema to form in the eye which would present with a collection of
blood in the inferior portion of the eye in the anterior chamber.

Answer 4: Chronic steroid use is a risk factor for cataracts which presents with bilateral and progressive
loss of vision (worse at night) with a cloudy lens seen on slit-lamp exam.

Answer 5: Recent viral illness presents with unilateral conjunctival injection and watery/itchy eyes with
symptoms that typically spread from one eye to the other in addition to other symptoms of a viral infection
such as a runny nose and cough.

Bullet Summary:
Penetrating trauma and cataract surgery are risk factors for endophthalmitis.

Only for Study purpose / Only Notes / AMC MCQ


Only Study Notes / Only for Study purposes / Personal notes/ AMC MCQ

177- A 64-year-old male presents to the emergency room with a severe headache and nausea. He reports
that he had been rocking his grandson to sleep that night when the symptoms began. He states the pain is
constant, on his right-side, and predominantly surrounds his eye and upper forehead. He had one episode
of vomiting. The patient also reports difficulty seeing out of his right eye, which he attributes to the
excessive tearing. The patient’s past medical history is significant for hypertension. His medications include
hydrochlorothiazide. The patient’s right eye is shown in Figure A. Upon physical examination, the right pupil
is minimally responsive to light and the globe feels firm. A right-sided carotid bruit is appreciated. Of the
following, which is the best
prophylaxis for this patient’s
condition
1
Acetazolamide
2
Amitriptyline
3
Clopidogrel
4
Epinephrine
5
Verapamil

This patient is presenting with


sudden-onset, unilateral,
orbitofrontal headache, a
dilated pupil with poor light
response, vision loss, and a
hard ocular globe, suggesting the diagnosis of acute angle-closure glaucoma. Long-term management of
angle-closure glaucoma may involve multiple pharmacologic agents, such as acetazolamide.

Examination of the eye in a patient with acute-closure glaucoma will reveal a red eye that is rock-hard, and
a dilated pupil which is non-reactive to light. Fundoscopic exam will show an increased optic cup-to-disk
ratio (>0.4), and tonometry will show increased intraocular pressure. Gonioscopy is the diagnostic gold-
standard. Acute treatment and long-term management involves beta-blockers, alpha-2-agonists and
carboanhydrase inhibitors, such as acetazolamide, to decrease intraocular pressure. The definitive
treatment is iridotomy.

Figure A shows an injected eye with a


mid-dilated pupil. Illustration A shows a
normal optic disk on the left and an
increased cup-to-disk ratio on the
right.

Incorrect Answers:
Answer 2: Amitriptyline can be used as
prophylaxis for migraines. Migraines
present as unilateral, pulsating
headaches, which may be associated
with nausea or photophobia.

Answer 3: Clopidogrel may be used as


conservative treatment for carotid
atherosclerosis, which can be an embolic source for central retinal artery occlusion (CRAO). CRAO

Only for Study purpose / Only Notes / AMC MCQ


Only Study Notes / Only for Study purposes / Personal notes/ AMC MCQ

presents as acute, painless, monocular vision loss.

Answer 4: Epinephrine is contraindicated in the treatment of angle-closure glaucoma as it increases


pupillary dilation. It is used in management of open angle glaucoma.

Answer 5: Verapamil is used as prophylaxis for cluster headaches. Cluster headaches present as
unilateral, repetitive, brief headaches associated with severe peri-orbital pain, lacrimation, rhinorrhea, and
Horner syndrome (miosis, ptosis, and anhidrosis).

Bullet Summary:
Pharmacologic management of a cute angle-closure glaucoma involves beta-blockers, alpha-2 agonists,
and carbohydrase inhibitors, such as acetazolamide.

178- A 59-year-old woman presents with sudden onset ocular pain. She reports she was visiting the
planetarium when the pain started, and she is now seeing halos around lights. She has vomited several
times from the pain. Her temperature is 97.6°F (36.4°C), blood pressure is 154/91 mmHg, pulse is 89/min,
respirations are 16/min, and oxygen saturation is 98% on room air. Physical examination is notable for the
finding in Figure A. Which of the following is the most effective treatment for this patient?
1
Iridotomy
2
Methylprednisolone
3
Pilocarpine drops
4
Retinal reattachment
5
Timolol drops
This patient is presenting with acute eye pain after spending time in the darkness as well as is seeing visual
halos, has impaired vision, and has a fixed and dilated pupil which is concerning for acute angle-closure
glaucoma. Definitive treatment is a laser peripheral iridotomy.

Acute angle-closure glaucoma occurs when the natural drainage system for aqueous outflow is obstructed.
The angle refers to the area between the iris and cornea, through which fluid must flow to escape via the
trabecular meshwork. When this angle is closed, intraocular pressure rises rapidly, causing compression of
the vessels that oxygenate the retina. In a short time, these cells may become ischemic resulting in retinal
ganglion cell death and optic nerve atrophy. Patients present with ocular pain, halos, a red eye, elevated
intraocular pressures, decreased vision, and a fixed, dilated pupil. Initial medical treatment involves timolol,
acetazolamide, dorzolamide, brimonidine, and pilocarpine. Definitive treatment is via laser peripheral
iridotomy which involves opening a small hole in the iris with a laser which can re-establish proper aqueous
outflow.

Figure A depicts the classic appearance of acute angle-closure glaucoma with a fixed, dilated pupil.

Incorrect Answers:
Answer 2: Methylprednisolone is a potent IV steroid which can be given for a flare of multiple sclerosis
which may present with optic neuritis. Optic neuritis presents with sudden-onset, painful vision loss in a
young woman with other neurologic complaints (such as paresthesias or weakness).

Answer 3: Pilocarpine drops act on the muscarinic receptors and cause miosis which opens the angle;
however, this is not definitive treatment of this condition.

Only for Study purpose / Only Notes / AMC MCQ


Only Study Notes / Only for Study purposes / Personal notes/ AMC MCQ

Answer 4: Retinal reattachment is indicated in the management of retinal detachment which presents with
vision loss with a "curtain coming down" over the visual field. Ultrasound can further characterize the
detached retina.

Answer 5: Timolol drops block beta-receptors, reduce aqueous humor production, and are appropriate
initial medical treatment but are not definitive therapy.

Bullet Summary:
Iridotomy is the definitive treatment of acute angle-closure glaucoma.

179-A 30-year-old male presents to your primary care clinic complaining of a "red eye". He denies pain,
drainage, eye or eyelid surgery, or recent trauma. His medical history is significant for asthma and GERD;
his only medication is ranitidine. He drinks 2-3 alcoholic beverages weekly, and denies smoking or illegal
drug use. He reports recently beginning a weightlifting program. His vital signs are as follows: HR 63, BP
119/72, RR 9, and SpO2 100% on room air. Physical exam is unremarkable, with the exception of the
findings seen in Figure A. What is the proper treatment for this condition? A
1
Watchful waiting
2
Topical antihistamine
3
Topical acyclovir
4
Repeated washing of eyelid with soap and water
5
Topical acetazolamide
This patient has a subconjunctival hemorrhage.
This is a self-limited condition that typically
resolves within 2 weeks.

Subconjunctival hemorrhage is caused by the


rupture of conjunctival or episcleral blood vessels
into the subconjunctival space. Bleeding is
typically painless, and exam reveals a sharply
demarcated area of bleeding on the sclera. Recent surgery, trauma, drainage, and significant pain are
warning signs that warrant a broader differential diagnosis.

Cronau et al. discuss the diagnosis and management of the patient with a red eye. They note that common
causes include conjunctivitis, blepharitis, corneal abrasion, foreign body, subconjunctival hemorrhage,
keratitis, iritis, glaucoma, chemical burn, and scleritis.

Sahinoglu-Keskek et al. present a retrospective observational study of 50 patients with subconjunctival


hemorrhage. They note that the most commonly associated condition in spontaneous hemorrhage was
hypertension, and that spontaneous hemorrhage appeared equally often in the temporal and nasal portions
of the eye.

Figure A shows a patient with subconjunctival hemorrhage.

Incorrect Answers:
Answer 2: Topical antihistamine drops would be appropriate for allergic conjunctivitis.
Answer 3: Topical acyclovir drops are used to treat herpes conjunctivitis.

Only for Study purpose / Only Notes / AMC MCQ


Only Study Notes / Only for Study purposes / Personal notes/ AMC MCQ

Answer 4: Washing with soap and water can help blepharitis.


Answer 5: Topical acetazolamide is a treatment for acute closed-angle glaucoma.

180-A 60-year-old female presents to the emergency department with severe pain and vision loss in her left
eye. She states that the pain and vision loss seemed to start when she was driving through a dark tunnel.
She has never experienced symptoms like this before and has no prior history of ophthalmologic problems.
Her left eye is demonstrated in Figure A, is poorly responsive to light, and feels hard to the touch. What is
the most likely cause of this patient's presentation?
1
Open angle glaucoma

2
Closed angle glaucoma

3
Cataract

4
Central retinal artery occlusion

5
Central retinal vein occlusion

This patient presents with


acute closed angle glaucoma
characterized by unilateral
vision loss, eye pain and
redness, mydriasis, and poor responsiveness to light.

Acute closed angle glaucoma is caused by enlargement or forward movement of the lens against the
central iris, leading to obstruction of the normal aqueous flow through the pupil (Illustration A). This causes
fluid buildup behind the iris, pushing the peripheral iris against the cornea and impeding flow through the
trabecular network. When this occurs acutely, it presents with unilateral severe eye pain and vision loss. It
is often accompanied by halos around light, eye hardness, and frontal headaches. The treatment is
pilocarpine, an acetylcholine agonist.

Pokhrel and Loftus review the diagnosis and management of ocular emergencies. They state all ocular
emergencies, including a penetrating globe injury, retinal detachment, central retinal artery occlusion, acute
angle-closure glaucoma, and chemical burns, should be referred immediately to the emergency department
or an ophthalmologist. Furthermore, they state that all patients with eye problems should be tested for
visual acuity and ocular movements. Confrontation visual field examination, pupillary examination, and
direct ophthalmoscopy of both eyes also should be performed.

Sng et al. review the mechanism of acute angle closure glaucoma. They state that more shallow anterior
chamber depth was the main anterior segment biometric parameter associated with acute primary angle
closure during the attack. Furthermore, they state anatomic changes in the anterior segment explained only
about one third of the variance in acute primary angle closure occurrence, and the role of nonanatomic
factors require further investigation.

Figure A demonstrates conjunctival injection and mydriasis which is common in acute closed angle
glaucoma. Illustration A demonstrates the pathophysiology of acute closed angle glaucoma. Illustration B
demonstrates the characteristic opacity seen in cataracts. Illustration C demonstrates retinal whitening with

Only for Study purpose / Only Notes / AMC MCQ


Only Study Notes / Only for Study purposes / Personal notes/ AMC MCQ

a cherry-red spot as is commonly seen in central retinal artery occlusion. Illustration D demonstrates the
"blood and thunder" image that would be seen on funduscopic exam in central retinal vein occlusion.

Incorrect Answers:
Answer 1: Open angle glaucoma is characterized by peripheral, then central PAINLESS vision loss, and is
associated with increasing age.
Answer 3: Cataracts are painless opacifications of the lens seen with advanced age. The opacification
would be clearly evident on physical exam as seen in Illustration B.
Answer 4: Central retinal artery occlusion (CRAO) is characterized by sudden, painless blindness and a
pale retina with a cherry-red macula on funduscopic evaluation as seen in Illustration C.
Answer 5:
Central
retinal vein
occlusion
can present
similarly to
CRAO, but a
"blood and
thunder"
image would
be seen on
funduscopic
exam as

shown in Illustration D.

Only for Study purpose / Only Notes / AMC MCQ


Only Study Notes / Only for Study purposes / Personal notes/ AMC MCQ

181- A 55-year-old African American man presents to his primary care physician for a general check-up.
The patient states that he feels well aside from the occasional headache. He also reports trouble
maintaining an erection with his new girlfriend. The patient has a past medical history of diabetes,
hypertension, and vascular claudication. His temperature is 99.1°F (37.3°C), blood pressure is 187/108
mmHg, pulse is 90/min, respirations are 15/min, and oxygen saturation is 99% on room air. Physical exam
is notable for narrowing of the patient's visual field. The rest of his neurological exam is within normal limits.
Ophthalmoscope exam reveals the finding in Figure A. Which of the following is the most likely
diagnosis? Review Topic | Tested Concept
QID: 210625
FIGURES:
A
1
Closed-angle glaucoma
2
Migraine headache with aura
3
Normal age related changes
4
Open-angle glaucoma
5
Pituitary mass

This patient is presenting with


narrowing of his visual field and
cupping of his optic disc
suggesting a diagnosis of open-
angle glaucoma.

Open-angle glaucoma typically


presents in patients of African
American descent with a history of glaucoma and diabetes. Early on, the patient is asymptomatic but if
untreated glaucoma can lead to a loss of peripheral vision or tunnel vision. Ophthalmoscope exam will
demonstrate cupping of the optic disc. Timolol eye drops are the best initial management.

Figure A demonstrates cupping of the optic disc that would be seen in glaucoma.

Incorrect Answers:
Answer 1: Closed-angle glaucoma presents with sudden blurred vision, severe eye pain, nausea/vomiting,
and a red eye with a fixed and dilated pupil.

Answer 2: Migraine headache with aura could present with headaches and visual abnormalities; however,
the visual abnormalities would typically occur prior to an episode and would involve spots or waves in the
visual field rather than a narrowing of it.

Answer 3: Normal age related changes would not cause narrowing of the visual fields or the optic disc
cupping seen in this patient.

Answer 5: Pituitary mass could present with headaches, bitemporal hemianopsia, and symptoms of
increased prolactin like erectile dysfunction; however, the cupping seen in this patient is more suggestive of
a diagnosis of open angle glaucoma.

Only for Study purpose / Only Notes / AMC MCQ


Only Study Notes / Only for Study purposes / Personal notes/ AMC MCQ

Bullet Summary:
Open-angle glaucoma presents with gradual narrowing of the visual field and cupping of the optic disc.

182-A 57-year-old woman presents with a cough, hemoptysis, fever, chills, and weight loss that has
persisted since she returned from her trip to Thailand. She admits to having sexual contact with several
prostitutes. The patient is generally healthy and is not taking any medications. She has a family history of
glaucoma, diabetes mellitus, factor V Leiden, and an ischemic stroke. Her temperature is 101°F (38.3°C),
blood pressure is 125/84 mmHg, pulse is 99/min, respirations are 19/min, and oxygen saturation is 95% on
room air. Physical exam is notable only for some coarse breath sounds. A chest radiograph is performed as
seen in Figure A. Treatment for this patient's condition is started. The patient returns to clinic with
decreased visual acuity mainly in her right eye. She also reports pain in both eyes with movement. On
physical examination, when a penlight is shined into the right eye, there is no pupillary constriction in either
eye. The rest of her neurological exam is unremarkable. Which of the following is the most appropriate next
step in management?
1
Change antibiotic therapy

2
CT head

3
MRI head

4
Timolol eye drops

5
Ultrasound eye

183-This patient initially


presented with travel to
an endemic area for
tuberculosis (TB) and the
diagnosis is highly
suggested by her
symptoms (cough, night
sweats, and weight loss)
in conjunction with the cavitary lesion on chest radiography. Appropriate treatment is typically RIPE therapy
which includes ethambutol which can cause optic neuritis (painful loss of vision) which, when present,
requires a different treatment regimen.

Tuberculosis should be suspected in any patient with travel to an endemic area who presents with fever,
cough, malaise, and night sweats. The diagnosis of tuberculosis can be made with a positive chest
radiograph and sputum acid fast staining. The treatment of tuberculosis is RIPE therapy:

1. Rifampin
2. Isoniazid
3. Pyrazinamide
4. Ethambutol

Only for Study purpose / Only Notes / AMC MCQ


Only Study Notes / Only for Study purposes / Personal notes/ AMC MCQ

Ethambutol carries with it the risk of elevating liver enzymes and can cause optic neuritis. Optic neuritis is
an acute inflammatory demyelinating injury to the optic nerve. The classic presentation of optic neuritis
involves painful, monocular visual loss evolving within hours to a few days. The most common signs on
physical examination are an afferent pupillary defect and color desaturation. Optic nerve atrophy can be
visible on MRI. Often, these problems persist after treatment, even if vision is recovered. In the case of
optic neuritis from ethambutol toxicity, an alternative treatment regimen should be chosen.

Figure/Illustration A is a chest radiograph demonstrating a cavitary lesion in the right upper lobe (red
circle).

Incorrect Answers:
Answer 2: CT head would be the best initial step in management for a suspected stroke which may present
with sudden onset focal neurologic deficits typically in an older patient with other medical comorbidities.

Answer 3: MRI head could be performed to confirm the diagnosis of optic neuritis. However, such classic
symptoms of optic neuritis in the face of ethambutol (an agent that can cause such a condition), warrants
discontinuing the offending agent first to avoid further irreversible damage.

Answer 4: Timolol eye drops would be an appropriate treatment in glaucoma which would present with a
rock hard eye with a fixed and dilated pupil. The diagnosis of acute glaucoma can be made with tonometry.

Answer 5: Ultrasound of the eye would be appropriate in the workup of a retinal detachment which presents
with a "curtain falling down over the eye" loss of vision. It would demonstrate a free flap (the retina).

Bullet Summary:
Ethambutol is used to treat tuberculosis and can cause optic neuritis.

184-A 3-year-old boy is brought to the pediatrician by his mother for "irritated eyes." The mother states that
2 days ago she noticed that the patient kept rubbing his eyes. Yesterday, his eyes appeared slightly red.
Then this morning he woke up crying because he couldn’t open his eyes. The patient has atopic dermatitis
and uses topical triamcinolone as needed. He had surgery for eustachian tube dysfunction 6 months ago.
The patient’s temperature is 101.3°F (38.5°C), blood pressure is 102/60 mmHg, and pulse is 110/min. On
physical exam, extraocular eye movements are intact and the pupils are equal and reactive to light. Figure
A shows the patient’s eyes as they appear grossly. Which of the following is the most likely cause of the
patient’s symptoms?
1
Adenovirus
2
Allergen
3
Chemical irritant
4
Corneal abrasion
5
Streptococccus pneumoniae

The patient is presenting with fever and


purulent discharge of the eyes, which is
suspicious for bacterial conjunctivitis. One cause of bacterial conjunctivitis is Streptococccus
pneumonia.

Only for Study purpose / Only Notes / AMC MCQ


Only Study Notes / Only for Study purposes / Personal notes/ AMC MCQ

Bacterial conjunctivitis is an infection of the conjunctiva of the eye. The most common causes are
Staphylococcus aureus, Streptococcus pneumonia, and Haemophilus influenza. Other causes
include Moraxella lacunata, and the sexually transmitted organisms Neisseria gonorrhoae and
Chlamydia trachomatis. Patients will present with eye redness, irritation, light sensitivity, and
purulent discharge that classically causes crusting or “gluing” of the eyelids upon waking. Patients
may have an associated fever. Physical exam will show evidence of purulent discharge and
erythema but an otherwise normal ocular examination.

Figure A shows mildly erythematous conjunctiva with purulent discharge, consistent with bacterial
conjunctivitis.

Incorrect Answers:
Answer 1: Adenovirus is a common cause of viral conjunctivitis. Patients will present similarly to
bacterial conjunctivitis except the discharge is watery rather than purulent. Patients may also have
signs of a concurrent upper respiratory infection.

Answer 2: Allergens such as pollen can cause allergic conjunctivitis. Patients will present with
watery, markedly itchy eyes bilaterally.

Answer 3: Chemical irritants can cause immediate corneal damage that often results in pain and
photophobia. They may also cause defects in vision. Purulent discharge is not associated with
chemical injury.

Answer 4: Corneal abrasions are often secondary to trauma, dry eyes, or an intraocular foreign
body. Patients will present with severe eye pain, photophobia, and a foreign body sensation.
Purulent discharge is not associated with a corneal abrasion.

Bullet Summary:
Bacterial conjunctivitis will present with grossly purulent exudate, irritation, erythema, and fever.

1
Highlight or note the key tested concept in the explanation to advance to 60%

185-A 55-year-old man comes into your office for an eye exam. The patient states he has had some trouble
seeing over the last few weeks. He denies a history of pain. He has a history of HIV. He states that he was
diagnosed with HIV about 20 years ago due to his intravenous drug usage. He has not used any drugs in
the past 10 years, but states he
also does not take his
medications. His last CD4+
count (6 months ago) was 42.
His vitals are normal and Figure
A shows his fundoscopic exam.
On eye exam, the patient has
both some central and
peripheral vision loss. What is
the most likely diagnosis?
1
Retinal vein occlusion
2
Retinal detachment
3

Only for Study purpose / Only Notes / AMC MCQ


Only Study Notes / Only for Study purposes / Personal notes/ AMC MCQ

Acute angle closure glaucoma


4
CMV retinitis
5
HSV retinitis

Patient with HIV and a CD4+ count less than 50 along with painless vision loss over a period of a few
weeks is most likely suffering from cytomegalovirus (CMV) retinitis. He should be treated with
valgancyclovir.

Cytomegalovirus (CMV) is a prevalent pathogen, with proposed infection rates 40 to 100%. CMV is a
member of the Herpesviridae family, which includes the Epstein-Barr virus (EBV), herpes simplex virus,
varicellazoster virus, and herpesvirus 6, 7, and 8. Like those viruses, infection with CMV is lifelong with the
virus remaining latent within the host and reactivating when the immune system is compromised (such as in
HIV, organ transplant, chemotherapy). It is especially dangerous in the immunocompromised and pregnant
women as congenital infection can often be fatal.

Taylor discusses CMV and its role in HIV patients. Taylor states that before HAART therapy, CMV retinitis
was the second most common opportunistic infection in HIV patients and that 21 to 45% of the patients
presented with complications of CMV retinitis, an irreversible form of vision loss. They also recommend that
an ophthalmologist should do a dilated indirect funduscopic examination on patients with symptoms or a
CD4 count of less than 50 per mm3 every three to four months because only 10 percent of the retinal area
can be observed with the use of a direct ophthalmoscope.

Jabs et al. compared different treatment regimens for CMV retinitis. When they compared intraocular only
regimens with systemic therapy, they found that regimens containing systemic anti-CMV therapy were
associated with a 50% reduction in mortality, a 90% reduction in new visceral CMV disease, and an 80%
reduction in second eye disease (among those who only presented with uniocular disease) (P<0.01).

Figure A shows yellow-white patches of retinal opacification due to CMV retinitis. Illustration A shows
widespread retinal hemorrhage with dilated retinal blood vessels.Illustration B shows a detached superior
retina.

Incorrect Answers:
Answer 1: Retinal vein occlusion would present with painless vision loss, but would have dilated veins
along with hemorrhage on fundoscopic exam. (Illustration A)
Answer 2: Retinal detachment would present with painless vision loss in the setting of hypertension or
trauma, but would have a detached retina along with the patient complaining of "floaters" or "flashes." It
may also present with a dark curtain falling over the eye if the detachment is severe as illustrated.
(Illustration B)
Answer 3: Acute angle closure glaucoma would present with severe pain, increased intraocular pressure,
and a mid-dilated pupil.
Answer 5: HSV retinitis would present in a patient with a low CD4+ count (as above) , but would present
with painful vision loss instead of painless vision loss. HSV also may involve other parts of the eye such as
the retina.

186-A 74-year-old man with a history of type II diabetes and a 40 pack-year smoking history presents to the
emergency room complaining of sudden-onset, painless vision loss in his left eye. He describes the feeling
as if things went black in his left eye suddenly. His temperature is 98.0°F (36.7°C), blood pressure is
154/94 mmHg, pulse is 100/min, respirations are 17/min, and oxygen saturation is 98% on room air.
Neurological exam reveals cranial nerves III-XII as intact. The patient is unable to see out of his left eye but

Only for Study purpose / Only Notes / AMC MCQ


Only Study Notes / Only for Study purposes / Personal notes/ AMC MCQ

has normal vision in his right eye. The rest of his exam including strength, sensation, and gait are
unremarkable. An ECG is performed as seen in Figure A and an ultrasound of the eye is performed as
seen in Figure B. Fundoscopy is performed as seen in Figure C. A CT scan of the head is currently
pending. Which of the following is the most likely diagnosis?
1
Central retinal artery
occulsion
2
Central retinal vein
occlusion
3
Hemorrhagic stroke
4
Ischemic stroke
5
Retinal detachment
This patient is presenting with sudden monocular
vision loss, atrial fibrillation, a cherry red spot on the
macula, and a normal ocular ultrasound suggesting
a diagnosis of central retinal artery occlusion
(CRAO).

Central retinal artery occlusion presents acutely as


a painless, monocular loss of vision. The retina will
appear pale on fundoscopic exam with a cherry-red
macula. This occurs because the loss of perfusion
of the retina allows blood from the choroid to shine
through. Risk factors can include a hypercoagulable
state and atrial fibrillation. Treatment can involve
digital massage of the globe and CO2 rebreathing, which function to dilate retinal vessels in an attempt to
pass the clot into a distal segment of the vessel (though there is not concrete evidence to support this
practice).

Figure A is an ECG demonstrating atrial fibrillation with an irregularly, irregular rhythm with no P waves.
Figure B is an ultrasound of the eye that is
unremarkable and does not demonstrate a free
flap (thus, ruling out retinal detachment). Figure C
is the fundoscopic finding of a cherry red macula.

Incorrect Answers:
Answer 2: Central retinal vein occlusion presents
with the "blood and thunder" finding on fundoscopy
in addition to vision loss.

Answer 3: Hemorrhagic stroke would present with


focal neurological deficits that worsen as there is
continued bleeding. It is unlikely this patient is
having a stroke given his only neurological finding
is monocular vision loss.

Answer 4: Ischemic stroke would present with focal


neurological deficits that extend beyond just

Only for Study purpose / Only Notes / AMC MCQ


Only Study Notes / Only for Study purposes / Personal notes/ AMC MCQ

monocular vision loss. Though this diagnosis is possible, the fundoscopy findings and otherwise normal
neurological exam suggest against this.

Answer 5: Retinal detachment would present with vision loss described as "a curtain coming down over the
vision" with an ultrasound showing a free flap in the eye.

Bullet Summary:
Central retinal artery occlusion presents with a sudden monocular loss of vision.

187-An 82-year-old male with a history of atrial fibrillation presents to the emergency room complaining of
sudden-onset, painless vision loss of his right eye. He describes the onset as if a “curtain was pulled down”
over his right eye. A review of systems is significant for occasional palpitations and pre-syncope. The
patient’s medications include aspirin and carvedilol. He reports that he is a smoker with a 50 pack-year
history. The patient’s temperature is 99°F (37.2°C), blood pressure is 112/72 mmHg, pulse is 84/min, and
respirations are 14/min with an oxygen saturation of 97% O2 on room air. Physical exam reveals 20/800
vision in the right eye. Fundoscopic exam of the right eye is shown in Figure A. Physical and fundoscopic
examinations of the left eye are unremarkable. Which of the following is the best next step in
management?
1
Administer tissue plasma activator
2
Administer ranibizumab injection
3
Start retinal photocoagulation
4
Start timolol drops
5
Begin ocular massage and hyperbaric
oxygen

This patient with a history of atrial


fibrillation, is presenting with acute-
onset, painless, monocular vision loss
and a fundoscopic exam positive for
retinal whitening. This suggests a
diagnosis of central retinal artery occlusion. Acute management includes ocular massage and hyperbaric
oxygen.

Symptoms of central retinal artery occlusion (CRAO) include sudden, painless, complete, and normally
unilateral vision loss. Fundoscopic exam can show pallor of the optic disc, diffuse ischemic whitening of the
retina (Figure A), cherry-red macula (Illustration A), retinal edema, and constricted/bloodless retinal
arterioles. The center of the macula appears red because the photoreceptor layer at the fovea is perfused
by the posterior choroidal vasculature, not the central retinal artery. Treatment involves surgical
decompression of the retinal anterior chamber, ideally within one hour of the occlusion. If surgical
intervention is not readily available, the next best step in management involves ocular massage (to
dislodge the embolus) and either carbogen therapy or hyperbaric, high-flow oxygen.

Figure A is a fundoscopic photograph showing retinal pallor surrounding the superior temporal arcade.
Illustration A is a fundoscopy showing a cherry-red spot.

Incorrect Answers:

Only for Study purpose / Only Notes / AMC MCQ


Only Study Notes / Only for Study purposes / Personal notes/ AMC MCQ

Answer 1: Administration of tissue plasma activator has not been shown to be beneficial in central retinal
artery occlusion.

Answer 2: Ranibizumab is an anti-


VEGF antibody, which can be used to
treat wet age-related macular
degeneration (ARMD). Symptoms of
wet ARMD include acute visual
distortion and loss of central vision that
occurs over months. Fundoscopic
exam would show neovascularitzation.

Answer 3: Retinal photocoagulation is


used to treat diabetic retinopathy. In
diabetic retinopathy, symptoms of
painless vision loss is gradual. Retinal
exam would show retinal hemorrhage
and possibly neovascularization.

Answer 4: Timolol is a beta-blocker used to treat glaucoma. Assessment of the optic disk would show an
enlarged cup-to-disk ratio.

Bullet Summary:
Acute management of central retinal artery occlusion involves ocular massage and hyperbaric oxygen,
especially if surgical intervention is not readily available.

188-A 69-year-old male presents to your clinic


with complaints of seeing “wavy lines” in the
vision of his left eye. He is not sure when it
started, but does believe it has gotten
progressively worse. He denies pain,
photophobia, or other ocular symptoms. His past
medical history is significant for type II diabetes
mellitus and hypertension. His current
medications include metformin and lisinopril. On
examination, his visual acuity is 20/400 in the left
eye and 20/70 in the right eye. His pupils are 2
mm bilaterally and equally reactive to light with
accommodation. There are no extraocular motor
deficits. A fundoscopic exam of his left eye is
shown in Figure A. What is the best next step in
management?
1
No treatment indicated
2
Massage the globe and start hyperbaric oxygen therapy
3
Start multivitamin and antioxidant supplements
4
Start bevacizumab

Only for Study purpose / Only Notes / AMC MCQ


Only Study Notes / Only for Study purposes / Personal notes/ AMC MCQ

5
Start acetazolamide drops
This patient is presenting with visual distortion, bilateral decrease in vision, and a fundoscopic exam
positive for “drusen,” suggesting the diagnosis of dry age-related macular degeneration. The best initial
therapy is a multivitamin and antioxidant supplements.

Dry ARMD is non-neovascular/non-exudative, and presents as a gradual central vision loss (scotomas)
over decades. Wet ARMD involves choroidal neovascularization and can cause a more rapid loss of vision
caused by bleeding secondary to the new vessel formation. Both dry and wet ARMD can present with
metamorphopsia or visual distortions. Dry ARMD on fundoscopic exam will show drusen deposition (Figure
A) and areas of retinal pigmentation and de-pigmentation. Wet ARMD will show choroidal
neovascularization (Illustration A). Dry ARMD is treated with zinc and antioxidant vitamins to prevent
progression. Wet ARMD is additionally treated with vascular endothelial growth factor (VEGF) inhibitors
and laser photocoagulation therapy.

Figure A shows fundoscopy with yellow extracellular material deposits known as “drusen,” characteristic of
dry age-related macular degeneration. Ollustration A shows fundoscopy with choroidal neovascularization,
which can be seen in wet age-related macular degeneration.

Incorrect Answers:
Answer 1: No definitive treatment is indicated for
retinal vein occlusion. Retinal vein occlusion
presents as sudden, painless, monocular vision
loss. Fundoscopic exam would show retinal
hemorrhage and venous dilation.

Answer 2: Massaging the globe and hyperbaric


oxygen therapy has shown to be beneficial in
central retinal artery occlusion. Symptoms include
acute, painless, monocular vision loss.
Fundoscopic exam would show a cloudy retina
with attenuated vessels and a “cherry-red”
macula.

Answer 4: Bevacizumab is a VEGF inhibitor, which


can be used to treat wet ARMD and is also part of
treatment for proliferative diabetic retinopathy. Both wet ARMD and proliferative diabetic retinopathy will
show forms of neovascularization on fundoscopic exam.

Answer 5: Acetazolamide is a diuretic used to treat open-angle glaucoma by inhibiting carbonic anhydrase,
which then decreases aqueous humor synthesis. Open-angle glaucoma presents as painless, peripheral
vision loss, resulting in “tunnel vision.” Optic disc atrophy causes an increased cup-to-disk ratio on
examination.

Bullet Summary:
Dry-ARMD, characterized by "drusen" on fundoscopic exam, is treated with zinc and antioxidant vitamins to
prevent progression.

189- A 37-year-old man comes to the emergency department for severe eye pain. The patient reports that
he is a construction worker and was drilling metal beams when he suddenly felt a sharp pain in his right
eye. Since then, the vision in his right eye has seemed blurry and his eye “has not stopped tearing.” The

Only for Study purpose / Only Notes / AMC MCQ


Only Study Notes / Only for Study purposes / Personal notes/ AMC MCQ

patient’s medical history is significant for type II diabetes mellitus and hypertension. His medications
include metformin, captopril, and lovastatin. He has a mother with glaucoma, and both his brother and
father have coronary artery disease. Upon physical examination, there is conjunctival injection present in
the right eye with no obvious lacerations to the eyelids or defects in extraocular eye movements. Pupils are
equal and reactive to light. No afferent pupillary defect is appreciated. The unaffected eye has 20/20 visual
acuity. The patient refuses to participate in the visual acuity exam of the right eye due to pain. Which of the
following is the best initial step in management?
1
Fluorescein stain
2
Orbital magnetic resonance imaging
3
Patching
4
Tonometry
5
Topical corticosteroids
The patient is presenting with a suddenly painful eye following construction work, suggesting a potential
intraocular foreign body, which should initially be tested with fluorescein staining.

An intraocular foreign body can present with severe pain in the eye. High-velocity activities such as drilling
and hammering increase one’s risk. If there are no open globes, traumatic hyphemas, or obvious
penetrating objects, the best initial step in management following physical exam is fluorescein staining,
which will highlight any defects in the cornea. Management is based on the type of corneal abrasion but
may include topical antibiotics, cycloplegics, and/or foreign body removal with topical anesthetic.

Incorrect Answers:
Answer 2: Orbital magnetic resonance imaging (MRI) should be avoided if the intraocular foreign body may
contain metal. An orbital plain radiograph or computed tomography (CT) scan may be used, but fluorescein
would be the initial diagnostic step.

Answer 3: Pressure patching may decrease the pain of large corneal abrasions (> 50% of the corneal
surface), but data is limited. Both pressure patching and black fabric patching are not recommended for
small abrasions and are contraindicated for patients with recent contact lens wear as it increases the risk of
infection.

Answer 4: Tonometry is used to measure intraocular pressure when examining for glaucoma. Angle-
closure glaucoma can present with acute onset of pain and blurry vision, but the pupil would be fixed-
dilated with possible corneal clouding/edema.

Answer 5: Topical corticosteroids should be avoided in the management of intraocular foreign bodies and
corneal abrasions. Corticosteroids can increase the risk of secondary infection.

Bullet Summary:
An intraocular foreign body may present with severe eye pain often following high-velocity activities, and
initial diagnosis should include a physical exam and fluorescein staining.

190-A 3-week-old infant presents to the emergency department with thick white discharge from his eyes
that has persisted for the past 24 hours. The patient's birth was not complicated, and he was born at home
vaginally with a mid-wife supervising the birth. The patient has a documented allergy to penicillin which
caused anaphylaxis after it was given secondary to a maternal syphilis infection. His temperature is 97.6°F

Only for Study purpose / Only Notes / AMC MCQ


Only Study Notes / Only for Study purposes / Personal notes/ AMC MCQ

(36.4°C), blood pressure is 75/40 mm Hg, pulse is 130/min, respiratory rate is 24/min, and oxygen
saturation is 98% on room air. The patient is currently sleeping. Physical exam is notable for bilateral
purulent drainage from the eyes. Which of the following is a complication associated with the best treatment
for this patient?
1
Damage to the lacrimal ducts

2
Non-bilious projectile vomiting

3
Optimal therapy has no known side-effects

4
Possible anaphylaxis and urticaria

5
Sedation and increased sleepiness

This patient is presenting with purulent drainage from his eyes several weeks after a vaginal birth
suggesting a diagnosis of chlamydial conjunctivitis. Chlamydial conjunctivitis is best treated with oral
erythromycin which increases the risk of pyloric stenosis (which causes non-bilious projectile vomiting).

Bacterial conjunctivitis presents with purulent drainage from the eyes. In pediatric patients, the most likely
cause of conjunctivitis after birth is secondary to Neisseria gonorrhea or Chlamydia trachomatis. N.
gonorrhea typically causes a more severe presentation with profuse purulent drainage from the eyes that
occurs days after birth. In contrast, C. trachomatis presents with a less severe purulent discharge that
typically occurs weeks after birth. The treatment of choice for chlamydial conjunctivitis is oral erythromycin
which is typically administered at birth prophylactically. Erythromycin increases the risk of pyloric stenosis,
in particular, in male newborns.

Incorrect Answers:
Answer 1: Damage to the lacrimal ducts is a possibility in dacryocystitis if a surgical intervention is
indicated. Dacryocystitis presents with a unilateral and enlarged lacrimal duct with expressible pus.
Inappropriate surgical procedures or incision and drainage of the lacrimal ducts can permanently damage
them.

Answer 3: Optimal therapy has no known side-effects reflects the treatment for viral conjunctivitis for which
treatment is symptomatic only and includes warm compresses. Viral conjunctivitis presents with a unilateral
or bilateral watery or yellowish discharge from the eye that is generally not purulent.

Answer 4: Possible anaphylaxis and urticaria is a possible complication in a penicillin-allergic patient who
received cephalosporins. IV ceftriaxone would be the optimal treatment for gonococcal conjunctivitis which
presents early (days after birth) with a thick and profuse purulent discharge that is more severe than that of
chlamydial conjunctivitis. Penicillin and cephalosporins only have about a 10% cross-reactivity at most.

Answer 5: Sedation and increased sleepiness is a side-effect of diphenhydramine or less commonly


loratadine which could be indicated in allergic conjunctivitis. Allergic conjunctivitis occurs in a seasonal
fashion and presents with a bilateral watery discharge from the eyes with bilateral injection.

Bullet Summary:
Chlamydial conjunctivitis is treated with erythromycin which increases the risk for pyloric stenosis.

Only for Study purpose / Only Notes / AMC MCQ


Only Study Notes / Only for Study purposes / Personal notes/ AMC MCQ

191-A 37-year-old woman presents to the emergency department with a headache, photophobia, blurred
vision, lacrimation, and eye pain. She states that she cannot recall when it started, but her symptoms are
severely affecting her ability to work and function. She has a past medical history of obesity, diabetes, and
hypertension. Her temperature is 99°F (37.2°C), blood pressure is 147/98 mmHg, pulse is 100/min,
respirations are 16/min, and oxygen saturation is 97% on room air. Direct and consensual photophobia is
noted on physical exam as well as the finding seen in Figure A. Which of the following is the best treatment
for this patient? A
1
Acetazolamide
2
Acyclovir
3
Ibuprofen
4
Topical erythromycin
5
Topical steroids
This patient is presenting with a
red eye and consensual
photophobia suggesting a
diagnosis of anterior uveitis,
which is best managed with topical steroids.

Anterior uveitis is defined as inflammation of the anterior uveal tract including the iris and ciliary body. The
etiology is thought to be either genetic, autoimmune, or infectious. Patients will typically present with a
headache, photophobia, a red eye, lacrimation, and blurred vision. A unique physical exam finding in
anterior uveitis is both direct and consensual photophobia. The diagnosis can be supported with a slit lamp
exam which may demonstrate cells in the anterior chamber. Treatment of anterior uveitis includes
consultation with an ophthalmologist and administration of topical steroids. It is important that an infectious
etiology is not suspected as topical steroids can lead to permanent damage to the eye otherwise.

Figure A demonstrates the physical exam finding of a red eye which can be seen in anterior uveitis.

Incorrect Answers:
Answer 1: Acetazolamide is the preferred treatment of pseudotumor cerebri or idiopathic intracranial
hypertension which typically presents with a headache and blurred vision in an overweight woman. Other
treatments include a lumbar puncture which can confirm the diagnosis when an elevated intracranial
pressure is demonstrated and reduce intracranial pressure by removing cerebrospinal fluid. Acetazolamide
would also be indicated in acute glaucoma.

Answer 2: Acyclovir is appropriate management of herpes keratitis which presents with abnormal vision,
pain, redness, and photophobia. The diagnosis can be confirmed with a fluorescein stain, and physical
exam will demonstrate corneal vesicles and dendritic ulcers.

Answer 3: Ibuprofen is appropriate management of a migraine headache which would present with pulsatile
head pain that could be associated with photophobia or even visual abnormalities if associated with an
aura.

Answer 4: Topical erythromycin is appropriate treatment of bacterial conjunctivitis which presents with
purulent drainage from the eyes and crusting.

Only for Study purpose / Only Notes / AMC MCQ


Only Study Notes / Only for Study purposes / Personal notes/ AMC MCQ

Bullet Summary:
The best treatment for anterior uveitis is topical steroids.

192-A 25-year-old man presents to the emergency department with bilateral eye pain. The patient states it
has slowly been worsening over the past 48 hours. He admits to going out this past weekend and drinking
large amounts of alcohol and having unprotected sex but cannot recall a predisposing event. The patient's
vitals are within normal limits. Physical exam is notable for bilateral painful and red eyes with opacification
and ulceration of each cornea. The patient's contact lenses are removed and a slit lamp exam is performed
and shows bilateral corneal ulceration. Which of the following is the best treatment for this patient? Review
Topic | Tested Concept
QID: 211075
1
Acyclovir
2
Erythromycin ointment
3
Gatifloxacin eye drops
4
Intravitreal vancomycin and ceftazidime
5
Topical dexamethasone and refrain from wearing contacts
This patient is presenting with painful, red eyes with opacification and ulceration of each cornea in the
setting of contact lens use suggesting a diagnosis of contact lens-associated infectious keratitis. Treatment
for contact lens-associated infectious keratitis includes topical broad-spectrum antibiotics (such as
gatifloxacin eye drops).

Contact lens-associated infectious keratitis typically occurs in patients who wear contact lenses who do not
remove their contacts or clean them properly. The patient will present with a painful, red eye with
opacification and ulceration of the cornea. The infection is typically caused by gram-negative (though could
also be caused by gram-positive) organisms as well as by fungi and amebas. The most likely gram-
negative culprits are Pseudomonas aeruginosa and Serratia spp. Contact lens wearers should receive
topical antibiotic coverage that covers for Pseudomonas including agents such as ciprofloxacin or
tobramycin (or other aminoglycosides). The contact lens should be discarded and not replaced until the
infection has completely resolved.

Incorrect Answers:
Answer 1: Acyclovir is an appropriate management of herpes simplex virus keratitis which is sexually
transmitted. It presents with pain and alteration of the patient's vision. Herpes keratitis appears as a linear
branching epithelial defect with terminal bulbs that stain brightly with fluorescein dye during slit lamp
examination.

Answer 2: Erythromycin ointment is an appropriate management of bacterial conjunctivitis which presents


with purulent drainage from the eye. It can also be indicated if super glue gets in the eye as it dissolves
super glue. Erythromycin ointment does not offer broad enough coverage for contact lens-associated
keratitis and does not cover for P. aeruginosa.

Answer 4: Intravitreal vancomycin and ceftazidime is an appropriate management of endophthalmitis which


is an infection involving the deep structures of the globe and presents with pain, altered vision,
inflammation, and visible inflammatory material on physical exam. It occurs secondary to trauma or surgery
which inoculates bacteria to the area.

Answer 5: Topical dexamethasone and refrain from wearing contacts is an insufficient management of

Only for Study purpose / Only Notes / AMC MCQ


Only Study Notes / Only for Study purposes / Personal notes/ AMC MCQ

contact lens-associated infectious keratitis. Though topical steroids in conjunction with topical broad-
spectrum antibiotics can improve outcomes, topical steroids alone could lead to local immunosuppression
and worsening of the patient's condition - steroid alone might be appropriate if there was chronic corneal
inflammation without evidence or concern for infection.

Bullet Summary:
Contact lens-associated infectious keratitis is best treated with topical broad-spectrum antibiotics, topical
steroids, and refraining from wearing contact lenses

193-A 59-year-old man presents to his ophthalmologist for an annual eye exam. The patient reports feeling
well and has noticed some changes in his visual acuity. Medical history is significant for hypertension, type
II diabetes, and ankylosing spondylitis. He is currently taking lisinopril, metformin, and naproxen. Family
history is significant for glaucoma in his mother and father. His intraocular pressure is 17 mmHg in the left
eye and 18 mmHg in the right eye. The patient undergoes a slit-lamp examination and the findings are
shown in Figure A. Which of the following is the most likely diagnosis?
1
Central retinal vein occlusion
2
Diabetic retinopathy
3
Glaucoma
4
Hypertensive retinopathy
5
Nonarteritic anterior ischemic
optic neuropathy

This patient's ocular findings of


retinal hemorrhages and
neovascularization are
suggestive of proliferative
diabetic retinopathy.

Diabetic retinopathy (DR) is a microvascular complication of diabetes that can result in blindness. DR can
be divided into non-proliferative and proliferative. The retinal vessels in these patients are abnormally
permeable and can become occluded, resulting in ischemia and neovascularization. In non-proliferative
DR, one can see retinal hemorrhages, lipid exudates, microaneurysms, and nerve fiber layer infarcts
(cotton wool spots.) In proliferative DR, one can see neovascularization, retinal and vitreous hemorrhages,
and potential retinal detachment as a consequence of the neovascularization.

Figure A demonstrates neovascularization and retinal hemorrhages, suggestive of proliferative DR.

Incorrect Answers:
Answer 1: Central retinal vein occlusion classically presents with a "blood and thunder" fundus, where there
are excessive intraretinal hemorrhages and cotton wool spots.

Answer 3: Glaucoma presents with cupping of the optic disc. The intraocular pressure is typically elevated,
and peripheral vision is the first to be lost. Normal intraocular pressure is between 8-22 mmHg.

Answer 4: Hypertensive retinopathy presents with flame hemorrhages, arteriovenous nicking,


microaneurysms, and cotton-wool spots.

Only for Study purpose / Only Notes / AMC MCQ


Only Study Notes / Only for Study purposes / Personal notes/ AMC MCQ

Answer 5: Nonarteritic anterior ischemic optic neuropathy typically presents with optic disc edema,
peripapillary bleeding, and a small optic cup.

Bullet Summary:
In proliferative diabetic retinopathy, there is neovascularization, retinal and vitreous hemorrhages, and
potential retinal detachment as a consequence of the neovascularization.

Only for Study purpose / Only Notes / AMC MCQ

You might also like